Путь который избрал герой ошибочен неправильный ошибка

Зачетная работа по русскому языку 11 класс

Блок 1. Орфографические и синтаксические нормы языка

  1. Распределите слова по группам с учётом орфографических особенностей. Назовите орфограммы.

Безнравственный, бескорыстный, колются, ветреный, вовсе не интересный, скворчонок, строят, раскорчёванный (лес), навзничь, не вооружён, карманный, холщовый, по-медвежьи, презрение, прекрасный, приморский, выкачанная (нефть), тушёнка, серебряный, соломенный, чрезмерный, нехоженый.

  1. Перепишите текст, расставьте пропущенные буквы и знаки препинания, раскройте скобки.

Каждое новое пок…ление детей всегда созд…ёт снов… и снов… великое множество подобных глаголов (н…)зам…чая своего языкового новаторства. Активность оглагол…вания имён существительных сн…жае(т,ть)ся у них лиш(?) (по)мере того как они выход…т из дошкольного возр…ста. Как близко пр…мыкают созд…ваемые ими глагольные формы к тем формам которые созд…(н,нн)ы и созд…ю(т,ть)ся народом видно (на)пример из слова «ра…кулачить». (В)первые я услыхал это слово (пол)века назад даже раньше. Гай внуч…нок И.Е. Репина крепко …жал свой кулачишк… и сказал Ну(ка) ра…кулач(?) мои пальцы!

В те времена такого слова ещё (не)существовало в народе так как ра…кулач…вание ещё (н…)стало историческим фактом. Для того что(бы) ребёнок мог зара(н,нн)ее так сказать (на)перёд ск…нструировать то самое слово которое лет двадцать спустя было созд…(н,нн)о народными ма(с,сс)ами нужно что(бы) он в сов…ршенстве владел теми(же) пр…ёмами построения слов которые выр…ботал (в)течени… тыс…ч…летий народ.

(К.И. Чуковский. «От двух до пяти»)

  1. Поставьте существительное, стоящее в скобках, в форму родительного падежа множественного числа.

Купить килограмм (абрикос, ананас, апельсин); группа (грузин); десять (ватт, вольт); двести (грамм); много (килограмм, километр); группа (таджик, татарин, узбек, якут); вернуться  из далёких (ущелье); пара (носок); килограмм (мандарин); несколько чистых (полотенце); салат из (помидор); урожай (баклажан); много свободных (место); несколько (юноша)

  1. Укажите синтаксические средства выразительности, использованные В. Астафьевым в данном тексте (средство — номера предложений)

 (1)Растаял мокрый снег. (2)Осталось на стекле приклеившееся птичье пёрышко. (3)Смятое. (4)Тусклое и до боли сиротливое. (5)Может, птаха малая стучалась ночью клювом по стеклу, просилась в тепло, а я, тугой на ухо человек, не услышал её, не пустил. (6)И пёрышко это, как укор, белеет на стекле.

(7)Потом обсушило солнцем стекло. (8)Унесло куда-то пёрышко. (9)А тоска осталась. (10)Должно быть, не перезимовала птичка, не дотянула до тепла и весны, вот сердцу-то и неловко, печально. (11)Залетело, видать, в  меня пёрышко. (12)Прилипло к моему сердцу.

  1. Укажите случаи неверного, разнотипного оформления однородных частей или членов предложения. Исправьте ошибки.

1. Путь, который выбрал герой, ошибочен, неправильный.

2. Книга научила смелости, честности и не болеть.

3. Стихотворение «Анчар» было отдано в печать, но не пропущенное цензурой.

4. Желаю вам здоровья и не болеть.

5. Всем понравилась повесть, опубликованная в последнем номере журнала
и который рассказывает о современной молодёжи.

  1. Постройте схемы предложений с прямой речью, исправив допущенные ошибки. Расставьте знаки препинания.

1. Сергей сказал, что я вернусь на следующей неделе.

2. Я знаю проговорила она вас не убедишь.

3. Гроза будет сказал Федя посматривая на облака.

4. Врач спросил какая у вас была температура.

5. Ученик сказал, что я ещё не подготовился к ответу.

6. Тарас сказал, что завтра же мы едем в Запорожье, чего откладывать.

  1. Устраните ошибки, связанные с употреблением вводных слов и предложений.

1. Кажется, видно, работа ещё не закончена.

2. Но увы, к счастью, коньки были с ним.

3. Вошёл какой-то незнакомый мужчина, пожалуй, скромно, даже бедно, одетый.

4. Покупатель медлит, обдумывая, какой лучше выбрать подарок, вероятно.

5. Это был огромный дуб, с обломанными, видно, сучьями давно.

6. Девочки напряжённо вглядывались в даль моря: наверное, на горизонте появится лодка.

7. По словам рыбаков, ночью был шторм, а теперь штиль.

8. Книга – источник знаний, как видят многие.

  1. Укажите недочёты в построении предложений (нарушение связи между словами, смешение конструкции,  пропуск членов предложения и т.д.). Отредактируйте фразы.

1. Учащиеся нашей школы приняли участие и стали призёрами областного смотра фольклорных коллективов художественной самодеятельности.

2. Солома, впитывая в себя ценные питательные соки, выделяемые в силосе при молочнокислом брожении, охотно доедается скотом.

3. Где проходила наша экспедиция, то были глухие места.

4. Необходимо знания и навыки, полученные учащимися в школе, впоследствии могли бы быть применены ими в жизни.

5. На трикотажной фабрике установлено 200 новых станков, а также для выработки объёмной полиэфирной нити.

6. Ставшая популярной ипотечная программа банков, она успешно работает уже на протяжении двух лет.

7. После обретения независимости Киргизию покинуло свыше тысяч граждан.

  1. Установите соответствие между предложениями и их характеристикой с точки зрения соблюдения морфологических норм.

    А. Нехорошо присваивать чужие идеи.

    Б. Нечего меня задобривать.

    В. Позови меня, когда вода вытекет из ванны.

    Г. Не трожь этот кустик: он здесь к месту.

    Д. Спортсмен достигнул хороших результатов.

    1. Нарушены нормы образования личной формы глагола.

    2. Нарушены нормы образования формы прошедшего времени глагола.

    3. Нарушены нормы образования видовой пары глагола.

    4. Нарушены нормы образования формы повелительного наклонения глагола.

    5. Морфологические нормы не нарушены.

  2. Исправьте ошибки в употреблении падежных форм и предлогов.

1. За это мы поговорим с вами на следующей неделе.

2. Тренер уверен за своего воспитанника.

3. Преподаватель подчеркнул о необходимости выполнять домашнее задание.

4. Его новый образ жизни сводился лишь в отказе от курения.

5. Мы всей семьёй будем контролировать за ходом лечения бабушки.

6. Наш профессор только вчера вернулся с Киева, где он выступал с докладом на научной конференции.

7. В журнале «Новый мир» опубликована рецензия о новой книге Александра Солженицына.

8. По окончанию института вы станете высококвалифицированными специалистами.

Блок 2. Работа с текстом

Прочитайте предложения и выполните задания 1, 2, 3.

А. Им было легче, чем европейским коллегам, провести такие исследования: в США и Канаде водится монарх – классический образец перелётной бабочки.

Б. Американские энтомологи внесли немалый вклад в изучение перелётов бабочек.

В. Регулярно в строго определённое время, весной и осенью, эти красивые бабочки совершают дальние путешествия.

Г. Среди других бабочек нет пожалуй равных им по умению преодолевать на крыльях далёкие расстояния

  1. В каком предложении допущена пунктуационная ошибка: не выделено вводное слово?

  2. В какой последовательности нужно расположить предложения, чтобы получился текст?

  3. Укажите ошибку в синтаксической характеристике предложения.

1) Предложение А сложное, бессоюзное, с обособленным приложением.

2) Предложение Б простое, распространённое.

3) Предложение В простое, распространённое, осложнённое уточняющими обстоятельствами.

4) Предложение Г простое, распространённое, грамматическая основа бабочек нет.

Блок 3. Задание с развернутым ответом

Прочитайте текст и выполните задания к нему

(1)Кажется, я был в пятом классе, когда у нас появились сразу несколько новых молодых учителей, только что вышедших из университета. (2)Одним из первых появился Владимир Васильевич Игнатóвич – учитель химии. (3)Это был молодой человек, только что с университетской скамьи, с чуть заметными усиками, маленького роста, с пухлыми розовыми щеками, в золотых очках. (4)Говорил он голосом, в котором звучали тонкие, как будто детские, нотки. (5)В классе несколько робел, и лицо его часто заливал застенчивый румянец. (6)Новый учитель обращался с нами вежливо, преподавал старательно, заданное спрашивал редко, к отметкам выказывал пренебрежение, уроки объяснял, как профессор, читающий лекцию.

(7)Первым результатом его системы было то, что класс почти перестал учиться. (8)Вторым – то, что ему порой начали слегка грубить. (9)Бедный юноша, приступавший к нам с идеальными ожиданиями, вынужден был расплачиваться за общую систему, которая вносила грубость и цинизм. (10)Впрочем, это было недолго. (11)Однажды, когда класс шумел и Игнатóвич напрасно надрывал свой мягкий голосок, одному из нас показалось, будто он назвал нас стадом баранов. (12)Другие учителя очень часто называли нас стадом баранов, а порой и хуже. (13)Но то были другие. (14)Они были привычно грубы, а мы привычно покорны. (15)Игнатóвич же сам приохотил нас к другому обращению.

(16)Один из учеников, Заруцкий, очень хороший, в сущности, малый, но легко поддававшийся настроениям, встал среди шумевшего класса.

– (17)Господин учитель, – сказал он громко, весь красный и дерзкий. – (18)Вы, кажется, сказали, что мы стадо баранов. (19)Позвольте вам ответить, что… в таком случае…

(20)Класс вдруг затих так, что можно было слышать пролетевшую муху.

– (21)Что в таком случае… вы сами баран…

(22)Стеклянная колбочка, которую держал в руках Игнатович, звякнула о реторту. (23)Он весь покраснел, лицо его как-то беспомощно дрогнуло от обиды и гнева. (24)В первую минуту он растерялся, но затем ответил окрепшим голосом:

– Я этого не говорил… (25)Вы ошиблись…

(26)Простой ответ озадачил. (27)В классе поднялся ропот, значение которого сразу разобрать было трудно, и в ту же минуту прозвенел звонок. (28)Учитель вышел; Заруцкого окружили. (29)Он стоял среди товарищей, упрямо потупившись и чувствуя, что настроение класса не за него. (30)Сказать дерзость учителю, вообще говоря, считалось подвигом, и если бы он так же прямо назвал бараном одного из «старых», то совет бы его исключил, а ученики проводили бы его горячим сочувствием. (31)Теперь настроение было недоуменно-тяжёлое, неприятное…

– (32)Свинство, брат! – сказал кто-то.

– (33)Пусть жалуется в совет, – угрюмо ответил Заруцкий.

(34)Для него в этой жалобе был своего рода нравственный выход: это сразу поставило бы нового учителя в один ряд с учителями старыми и оправдало бы грубую выходку.

– (35)И пожалуется! – сказал кто-то.

– (36)Конечно! (37)Думаешь, спустит?

(38)Этот вопрос стал центром в разыгравшемся столкновении. (39)Прошло два дня, о жалобе ничего не было слышно. (40)Прошёл день совета… (41)Признаков жалобы не было.

(42)На следующий урок химии Игнатóвич явился несколько взволнованный; лицо его было серьёзно, глаза чаще потуплялись, и голос срывался. (43)Видно было, что он старается овладеть положением и не вполне уверен, что это ему удастся. (44)Сквозь серьёзность учителя проглядывала обида юноши, урок шёл среди тягостного напряжения. (45)Минут через десять Заруцкий, с потемневшим лицом, поднялся с места. (46)Казалось, что при этом на своих плечах он поднимает тяжесть, давление которой чувствовалось всем классом.

– (47)Господин учитель… – с усилием выговорил он среди общей тишины. (48)Веки у молодого учителя дрогнули под очками, лицо всё покраснело. (49)Напряжение в классе достигло высшего предела.

– (50)Я… прошлый раз… – начал Заруцкий глухо. (51)Затем, с внезапной резкостью, он закончил:

– Я извиняюсь.

(52)И сел с таким видом, точно сказал новую дерзость. (53)Лицо у Игнатóвича посветлело, хотя краска залила его до самых ушей. (54)Он сказал просто и свободно:

– Я говорил уже, господа, что баранами никого не называл.

(55)Инцидент был исчерпан. (56)В первый ещё раз такое столкновение разрешилось таким образом. (57)«Новый» учитель выдержал испытание. (58)Мы были довольны и им и – почти бессознательно – собою, потому что также в первый раз не воспользовались слабостью этого юноши, как воспользовались бы слабостью кого-нибудь из «старых». (59)Самый эпизод скоро изгладился из памяти, но какая-то ниточка своеобразной симпатии, завязавшейся между новым учителем и классом, осталась.

 (По В.Г. Короленко*)

 *Владимир Галактионович Короленко (1853–1921) – русский писатель, журналист, публицист, общественный деятель.

  1. Какие из высказываний соответствуют содержанию текста? Укажите номера ответов.

1) Стремясь перекричать класс и понимая, что ему не удаётся это сделать, учитель химии оскорбил учеников.

2) Учитель химии Владимир Васильевич Игнатóвич получил университетское образование.

3) Одноклассники Заруцкого не одобрили его поступок по отношению
к учителю химии.

4) Заруцкий принёс учителю свои извинения, потому что побоялся, что тот пожалуется в совет.

5) Разрешение конфликта привело к возникновению определённой симпатии во взаимоотношениях класса с учителем.

  1. Ознакомьтесь с информацией о тексте. Заполните пропуски.

 Примерный круг проблем

Авторская позиция

1. 

1. Уважительное отношение со стороны учителя создаёт условия для формирования в характере учеников лучших качеств: способности к объективной оценке своего поведения по отношению к другим людям, способности к совершению честного поступка не в связи со внешним давлением, а по велению собственной совести, потребности в самоуважении. Учитель может влиять на формирование характеров учеников самой своей личностью, свойственной ему манерой поведения, своей речью.

2. Проблема решения конфликта между учителем и учениками. (При каком условии последствия конфликта, недоразумения, возникшего между учителем и учениками, могут быть устранены?)

2. 

3. 

3. Человеку бывает непросто признать свою вину, но, для того чтобы восстановить справедливость, вернуть доверительные взаимоотношения и сохранить самоуважение, важно принести свои извинения тому человеку, которому была нанесена обида.

4. Проблема высокой требовательности по отношению к интеллигентному человеку. (Какие требования предъявляются к интеллигентному человеку?)

4. 

5. Проблема противостояния человека и сложившейся системы. (Может ли человек противостоять системе?)

5. Человек, имеющий твёрдые принципы, способен не только противостоять общепринятой и даже одобряемой системе отношений, но и добиваться успеха в этом противостоянии.

  1. Напишите комментарий к одной из проблем текста (укажите проблему)

Пояснительная записка

Практикум по русскому языку и
культуре речи предназначен для студентов, обучающихся в средних
профессиональных заведениях. Основной целью данного пособия является
формирование у студентов практических навыков.

Структура пособия позволяет
отработать на практике такие вопросы стилистики и культуры речи, как нормы
русского литературного языка, функциональные стили и их особенности, а также
орфографические, пунктуационные навыки.

Поэтому весь материал размещен в
виде трех блоков: стилистика, культура речи, правописание.

Задачей практикума является
формирование и развитие коммуникативной, языковой, лингвистической и
культуроведческой компетенции.

1. Стилистика

Задание
1

Исправьте ошибки в данных предложениях.

1) Сейчас диктор подтвердил о том, что шайбу забросил
Фирсов.

2) Что можно комментировать о том ударе?

3) Сегодня погода благоприятствует для хорошего темпа
лыжных гонок.

4) На конференции состоялся обмен мнений.

5) Такие встречи помогают наладить обмен передового
опыта.

6) Нам остается позавидовать о том, что у вас есть такой
музей.

7) Эта песня доставила для меня большое удовольствие.

Большая группа ученых была удостоена
правительственными наградами.

9) Солдаты, полные жгучей ненавистью к врагу, сражались
мужественно и бесстрашно.

10) Большая сила воли
характерна этому герою.

11) Вся деятельность врачей
направлена для уничтожения страданий человеческих.

12) Герои этого рассказа
предпочитают смерть, чем покорность и рабство.

13) Награда еще раз напоминает
о знании литературного творчества писателя.

14) Разведчик не раз побывал в
тылу врага.

15) Знание русской грамматики
помогает избежать многие ошибки.

16) Дипломная работа студента
показывает о его умении анализировать и обобщать материал.

17) Выводы комиссии
противоречат с мнением коллектива лаборатории.

18) Болезнь и связанные с ней
пропуски занятий повлияли на знаниях школьника.

Задание 2

I. Составить словосочетания, правильно используя связь
слов.

1) Упрекать (бездеятельность)

2) Ругать (беспечность)

3) Повлиять (решение)

4) Свидетельствовать (работа)

5) Информировать (ход выполнения задания)

6) Видеть (картина)

7) Интересоваться (искусство)

Любовь (родина)

9) Исследовать (территория)

10) Благодаря (помощь)

11) Согласно (предписание)

12) Вопреки (угроза)

13) Полон (бодрость)

14) Предложить (входить)

15) Превосходство (противник)

16) Ориентироваться (мир книг)

17) Информация (погода)

18) Чуждый (музыка)

II

Приплыть

Добраться (берег)

Достичь

Восхищение (подвиг
космонавта)

Уважение

Гордость

Препятствовать (движение

Тормозить)

Страдать

Сетовать (разлука)

Тосковать

Скучать (вы)

Касаться

Прикасаться (стол)

Оплатить (квартира)

Уплатить

Гордиться (успехи)

Радоваться

Жажда (деятельность)

Потребность

Стремление

Участие

Принадлежность
(организация)

Стремление

Убежденность (правота
дела)

Верящий

Сопротивляться

Бороться (враг)

Приговаривать (казнь)

Осудить

Внушать страх (противник)

Вселять страх

Задание 3

I Исправьте
ошибки, вызванные нарушением связи между подлежащим и сказуемым.

1) Вот это смелость, дерзость,
сила любви к Родине, стремление уничтожить несправедливость, особенно дорога
мне в поэте.

2) Копившаяся много лет
ненависть против крепостников вылилось в подлинно народное восстание.

3) Жесткость крепостников —
помещиков, поборы, голод приводит к тому, что народ поднимает восстание.

4) На свободу русского народа
пытались посягнуть множество опасных врагов.

5) Сборник рассказов Бабеля
«Конармия» произвели бурю в литературном мире.

6) Вся наша литература, в том
числе и Казахстан, развивались под влиянием Пушкина.

7) Много молодежи пришли на
вечер танцев.

Никто из пришедших не
поздоровались с ней.

II Согласуйте сказуемое с подлежащим, допишите окончания.

1) Ряд специалистов
направлен… на заводы Урала, часть инженеров командиру…тся в Казахстан.

2) Установлен… три новых
рекорда по тяжелой атлетике.

3) Больше половины поступивших
в институт был… хорошо подготовлен… по всем предметам

4) Интеллигенция, и особенно
лучшая ее часть — образцовые писатели, всегда считал.. .сь хранителями…
чистоты литературного языка.

5) Первые, кто приш… на
избирательный участок для голосования, были мои соседи, пенсионеры.

6) Печать, в часности
многотиражки и стенные газеты, систематически расширяя…т информационные
материалы.

7) Немало военных преступников
нашл… себе после войны приют в Испании.

За короткий срок
построен… и сдан… в эксплуатацию пятьдесят один промышленный объект.

9) Полмесяца прошл… с тех
пор, как начались экзамены.

Задание 4

I Раскройте скобки и поставьте глагол в прошедшем
времени.

1) В углу в свернутом виде (лежать)
плащ — палатка.

2) Библиотека — музей
(приобрести) неизданные рукописи писателя.

3) ФРГ (заключить) договор о
перемирии.

4) ЦСУ (выпустить) очередной
бюллетень.

5) Диван — кровать (стоять) у
правой стороны.

6) На самом берегу моря
(раскинуться) Батуми.

7) Музей — квартира Булгакова
(приобрести) его личные вещи.

Платье — халат (висеть) на
вешалке.

9) «Спартак» (выйти в) финал
соревнований.

II

1) Подгорела картошка. Что
(подгореть)?

2) Подгорел лук. Что
(подгореть)?

3) Подгорели картошка и лук.
Что (подгореть)?

4) Поехал брат. Кто (приехал)?

5) Приехала сестра. Кто
(приехать)?

6) Приехали брат и сестра. Кто
(приехать)?

7) Кто такая (приехать)?

Об этом нам (рассказывать)
не кто иной, как Светлана.

9) Никогда не забудут ужасов
войны те, кто ее (пережить).

10) Никто из сестер, даже
старшая, не (знать) о случившимся.

11) Журнал «Москва»
(опубликовать) повесть молодого писателя.

12) «Известия» (поместить)
подробную рецензию на премьеру в театре «Современник».

13) Команда «Динамо»
значительно (обновить) состав нападающих.

14) Издательство «Наука»
(выпустить) в свет серию книг об аномальных явлениях.

Задание 5

Исправьте ошибки, связанные с неправильным
употреблением деепричастного оборота (деепричастный оборот оставьте без
изменения).

1) Глядя на эту ветку сирени,
мне вспомнилась моя молодость.

2) Подъезжая к станции и глядя
на природу в окно, с меня слетела шляпа.

3) Слушая музыку Моцарта, нас
не покидает ощущение причастности к чуду.

4) Прочитав это письмо, мне
стало даже смешно.

5) Едва научившись ходить, у
него возникла потребность рисовать.

6) Думая о ней, мне становится
грустно.

7) На катке всегда весело и
интересно, хотя, покатавшись, боля ноги.

Приехав домой, мне стало
значительно лучше.

9) Используя этот крем, кожа
ваша станет нежнее.

10) Войдя в воду, у меня все
тело покрылось мурашками.

11) Взяв ее под руку, они пошли
вместе.

12) Живя в монастыре, Мцыри
преследовали мечты о свободе.

13) Взобравшись на крышу сарая,
Павке хорошо был виден сад Лещинских.

14) Жизнь должна изображаться в
книгах такой, как она есть, не приукрашивая ее.

15) Увидев Марину в новом
платье, у него дух захватывало от радости.

16) Услышав его голос, сердце у
меня забилось чаще.

17) Проходя по дачному поселку,
на меня пахнуло свежесть жасмина.

Задание 6

Исправьте ошибки в предложениях. Здесь нарушено
правило об однородности членов предложения.

I.

1) В отрядах Пугачева было много башкир, татар, чувашей и
уральских рабочих.

2) За последний год я прочитал несколько романов,
художественных произведений, повестей и рассказов.

3) Помещики, капиталисты и самодержавие жили в роскоши.

4) В майские дни толпы людей можно было видеть повсюду:
на улицах, площадях, бульварах, скверах.

5) Жизнь крестьян изображена в произведениях многих
русских писателей- классиков: Гоголь, Тургенев, Толстой, Чехов.

6) В нашем отделе можно купить канцелярские товары,
карандаши и бумагу.

7) Принимаются заказы на пошив мужского, дамского и
военного платья.

В качестве материала для романа «Война и мир» Толстой
использовал семейные архивы, мемуары, литературные источники и другие
официальные документы.

II.

1) Я люблю игру на скрипке и петь.

2) Она молода, умна, красивая.

3) Он мечтал о поездке в Москву, учиться, работать, быть
полезным людям.

4) В своем сочинении я хотела рассказать о великих людях
и чему учат их подвиги.

5) Если весной войти в глубь леса, то можно услышать, как
лопаются почки и радостное чириканье птиц.

6) Недорослями называют молодых людей, не получивших
образования и которые стремились к праздной жизни.

1) Летом ученики нашего класса не только побывали в
Новгороде и Пскове, но и в Михайловском.

2) «Записки охотника» Тургенева интересны тем, что в этом
произведении не только фигурирует представители поместного дворянства, но и
крепостные крестьяне.

3) Я очень люблю ловить рыбу и занимаюсь этим не только
летом, а также и зимой.

4) Дополнительный материал я не только брал из учебной
литературы, но и из газет и журналов.

5) Для правильного письма не сколько важно знание
формулировки правил, столько умение его применять.

6) Сведений о Дубровском не было и на второй и третий
день.

Задание 7

Перепишите предложения, числительные запишите словами.

1. Государственная казна пополнилась еще 678000
(рубль).

2. Забастовки произошли еще в 281 (город).

3. Стипендии были выплачены 1552 (студент).

4. Наши охотники за один сезон уничтожили 54 (волк).

5. Учитель проэкзаменовал 22 (ученик).

6. Письмо датировано 1831 (год).

7. Мне пришлось пожертвовать еще 16 (доллар).

8. К 2567 (наименование) в список приписали еще и
1033 (наименование).

9. В трех зрительных залах – по 70 (место).

10. Потери
составили более 843 (килограмм).

Задание 8

Исправьте ошибки, вызванные наличием лишнего слова.

1. Рождество я отмечал на
даче у коллеги по работе.

2. Близнецы были так похожи,
что даже родители с трудом различали их одного от другого.

3. Деепричастный оборот
всегда обособляется запятыми.

4. Специфическая особенность
художественной речи состоит в том, что в ней много образных слов и выражений.

5. Дайте сообщение в газету
о свободных вакансиях.

6. Очень замечательные
пироги печет Пульхерия Ивановна

Задание 9

Найдите в тексте стилистические ошибки, отредактируйте
его, перепишите в исправленном виде.

В произведениях Пушкина и
Лермонтова два главные героя играют важное значение в истории русской
литературы. Прочитав их., нам стало понятно, как благодаря реакции,
обрушившейся после разгрома декабрьского восстания декабристов и которая вынудила
молодежь отказаться от активной политической деятельности, они, чувствуя превосходство
перед другим, стали эгоистами поневоле почти что более сто пятьдесят лет назад.

Задание 10

Перепишите предложения, устранив двусмысленность,
связанную с неправильным употреблением личных местоимений.

1. В комнате, возле печки,
стояла девушка; она была жарко натоплена.

2. Отец умер, когда ему было
девять лет.

3. Народ не знает, против
кого надо бороться. Они не знают, кто их главный враг.

4. Боясь грозы, старуха
спрятала голову под подушку и держала ее там, пока она не кончилась.

5. Молодежь осталась
довольна концертами, они просили почаще устраивать такие встречи с артистами.

6. За дорогой на вершине
холма — хвойной лес, а на его склонах раскинулись луга.

7. Мальчик дал щенку
тушеного мяса, и он за несколько секунд съел его.

8. Эту селедку дала мне
продавщица Люба, ввиду жары она уже припахивала.

Задание 11

Отредактируйте отрывок, устранив неясности, вызванные
неправильным употреблением местоимений.

Охотник увидел барса, но он был не
из трусливых. Он положил ружье, привязал собаку, взял рюкзак и побежал за ним.
«Поймай живым,»- решил Мартын Павлович. Мартын Павлович накинул на голову барса
мешок и, когда увидел, что он не может его сбросить, он вмиг очутился на нем
верхом.

Задание 12

Исправьте речевые ошибки.

1) Узкая дорога была покрыта проваливающимся снегом под
ногами.

2) Мой спутник указал мне на высокую поднимающуюся гору
прямо против нас.

3) Туристы делились полученными впечатлениями о Кавказе
за время похода.

4) По не зависящим причинам от Центрального телевидения
мы не можем продолжить передачу.

5) Прошу вас ответить на волнующие вопрос не только меня,
но и многих других.

6) Мне было поручено уничтожить засевшего снайпера на
высоте «830».

7) Состав играющих хоккеистов за московский «Спартак»
сегодня несколько изменен.

Поэт обратился к самому сокровенному, созданным самим
народом.

9) Недорослями называют молодых людей, недостаточно воспитанными
родителями.

10) Прочитав эту книгу,
невольно хочется стать похожей на этого внешне ничем не отличающихся от других
людей мальчишку.

11) На краю горизонта тянется
серебряная цепь снеговых вершин, начинающихся Казбеком и кончающихся Эльбрусом.

12) Воздух был пропитан острым
запахом моря и жирными испареньями земли, незадолго до вечера смоченными
дождем.

13) Он оглядывается и видит
напряженное лицо сына, внимательно прислушивающееся к их разговору.

14) Мы любовались картиной
бушующего моря у наших ног.

15) По узкой тропинки мы
поднялись на площадку, извивающуюся между скал.

Задание 13

Отредактируйте текст.

Помните Остапа Бендера, отпуская
витиеватый комплимент жене «голубого воришки» Альхена, запутался в придаточных
предложениях и так и не смог закончить свою мысль? Неумение строить сложные
предложения — серьезный пробел в знаниях. Многие писатели специально используют
в речи некоторых персонажей корявые нагромождения придаточных предложений для
того, чтобы подчеркнуть бескультурье и невежество говорящего. Например, Ф. М.
Достоевский вкладывает в уста отрицательного персонажа Ставрогина (из романа
«Бесы») такую неприглядную фразу: «Я так был низок, что у меня дрогнуло сердце
от радости, что выдержал характер и дождался, что она вышла первая».

Задание 14

Найдите и исправьте ошибки, связанные с неправильным
употреблением синонимов.

1) В составах команд немало именитых спортсменов много
раз добивавшихся отменных результатов на соревнованиях.

2) Преподаватель обязан содействовать исправлению
допущенных студентами ошибок.

3) Его деятельность по распространению билетов была
отлична.

4) Выгода массового производства блюд промышленными
методами способствует их удешевлению.

5) Промышленный и постперестроечный период переживает
кризис.

6) Сейчас набирают спортивную команду, лучшие спортсмены
поедут в Москву.

7) Повышение ответственности на старших курсах приводит к
высоким результатам в учебе.

Задание 15

Исправьте стилистические недочеты в употребленных
существительных, прилагательных, числительных.

1) Во время перерыва в холле продолжают спорить. Пойдем
поближе к этим троим студентам, послушаем, что они говорят.

2) Бытовые условия в поселке оказались более
предпочтительнее, чем леспромхозе.

3) Эти двести метров, отделяющие оба института, учебный и
исследовательский, преодолеть не так уж трудно.

4) Вера оказалась в преглупейшем положении.

5) Мы с композитором заметили, что в обоих песнях припев
почти одинаковый.

6) Метод исследования правилен и соответствен сложности
самой проблемы.

7) Благодоря рационализации на новой линии сейчас занято
вместо семерых только пять человек.

Задание 16

Найдите и исправьте ошибки, связанные с нарушением
лексической сочетаемости и порядок слов.

I

1) Скрутившись клубком в своем гнезде, спит белка.

2) После катка я иду домой радостная, с веселыми
впечатлениями.

3) Роль книги в жизни человека велика: ведь он должен
пополнять свой кругозор

4) Побыв в кино, я обычно думаю, способна ил я на такие
поступки, которые делают герои фильма.

5) Успехами мы не блестим.

6) Задача, поставленная нами, достигнута.

7) На демонстрации Сизов приобщается к революционерам.

Воздух чист. Солнце греет ярче. Дыхание свежо.

II.

1) Группа студентов пришла слушать факультативный курс
лекций по сварке доцента Юрьева.

2) Всем членам кассы взаимопомощи просьба подтвердить
свои остатки.

3) Организовано общежитие для мужчин и женщин с железными
койками на сетках.

4) Столяр сделал эту этажерку из дуба с четырьмя ножками.

5) Общественность города широко готовится отметить юбилей
известного писателя.

6) Бабушка восьми лет отвезла внука на Кавказ.

7) Гоголь прекрасно описал о похождениях Чичикова.

Царское правительство устроило гонение за передовыми
людьми.

Задание 17

Отредактируйте отрывок. Необходимо не только исправить
стилистические ошибки, но и усилить стилистическую выразительность текста,
заменив отдельные слова более экспрессивными синонимами.

Павел расстегнул кобуру, взял блестящий
револьвер и пообедал домой. Вбежав в дом, мать возилась в кухне и не обратила
на него внимания. Он взял лежащую за сундуком тряпку и положил ее в карман.
Потом он вышел в дверь и пошел на дорогу, ведущей к старому кирпичному заводу.

Там в полуразрушенных обжигных
печах было много потаенных местечек. В одно из них он спрятал завернутый
револьвер в тряпку и завалил его кирпичами.

Задание 18 (контрольное)

Найдите недочеты и исправьте их.

1) Начинается с декабристов, русская интеллигенция повела
борьбу с царизмом против эксплуатации и угнетения.

2) Но их мысли были утопическими: они намеревались
изменить существующий строй без помощи народа.

3) У вас есть пример, когда дворянство с оружием в руках
выступало против царизма (восстание декабристов), но они потерпели поражение.

4) Но вот в конце XIX — начале XX в. на арене
революционной борьбы появилась новая интеллигенция, оружие которых был
марксизм.

5) Как раз роман Горького «Мать» и освещает тот период,
когда интеллигенция начинает сеять зерна мысли о необходимости революционной
борьбы среди рабочих.

6) Кто же такая интеллигенция?

7) Первая, кто встречается нам, — это Наташа.

В детстве ей ни в чем не отказывали, и будущее будет у
нее светлым.

9) Но, повзрослев, у нее появилась точка зрения на жизнь.

10) Она увидела вокруг, что кругом жестокость и произвол.

11) Больше Наташа не могла жить в родительском доме, и
отец выбрал ее с такими настроениями.

12) И она оказалась наедине с жизнью.

13) Но тем и отличается революционная интеллигенция: сила
воли и стойкий характер помогли ей справиться со всеми трудностями.

14) Сейчас мы видим ее как стойкого борца.

15) Возьмем, к примеру, Егора Ивановича, вспомним, когда
посадили Павла в тюрьму за историю с «болотной копейкой», он пришел к Ниловне.

16) Он пришел к Ниловне и сказал, что ей надо делать,
чтобы спасти сына и его товарищей.

17) Нужно было восстановить доставку листов на завод,
тогда полицейские отпустят Павла, так как без них продолжали листовки печатать,
значит, заключенные не виноваты.

18) Егор Иванович мог не приносить листовки матери, но
высокое чувство долга не позволяло ему ждать освобождение товарищей.

19) О прошлом Егора Ивановича мы знаем мало.

20) Но по тому моменту, как Егор тяжело дышит, можно
догадаться, какая судьба была у него в прошлом.

21) Жизнь политического заключенного, тянувшего лямку
нелегкой судьбы каторжника.

22) О его высоких идеалах говорит тот факт, когда он,
умирая, не мог докончить дело до конца

23) А Саша, которая любила Павла, вместе с Ниловной ходила
по деревням, селам и рабочим поселкам, чтобы вернуть правду мужику, который не
может найти дорогу в этой жизни.

24) Но не все рабочие верили интеллигентам, например,
Рыбин.

25) Он говорил Ниловне, что листовки печатают сами
господа, а эти воззвания направлены против господ.

26) Это он не мог
понять, что «господа», печатающие литовки, отказались от мирских благ и
отдались революции.

27) Вот такие это были люди. Они были и
высокообразованные: учителя, доктора, юристы.

28) В самом
начале своего революционного пути Павел много читал и ходил в театр: он брал
пример с этих людей.

29) И в заключении я хочу сказать, что это были добрые и
веселые люди, сохранившие свои качества в нелегкой борьбе за воплощение своих
идеалов в жизни.

Задание 19

Охарактеризуйте стили по
следующим параметрам, внося данные в таблицу.

Характеристика

Стили

Цель

использования

Сфера

использования

Жанры

Языковые

средства

Официально-деловой

Научный

Художественный

Публицистический

Разговорный

Задание 20

Прочитайте текст.

Были глаза: черные, прекрасные.
Взглянут — смотрят, и спрашивают.

И были глазенки: серые,
плутоватые- все шмыгают, ни на кого прямо не смотрят. Спросили глаза:

— Что вы бегаете? Чего ищете?

Забегали глазенки, засуетились,
говорят:

— Да так себе, понемножку, полегоньку, нельзя —
помилуйте, надо же — сами знаете. И были гляделки: тусклые, … Уставятся и
глядят. Спросили глаза:

— Что вы смотрите? Что видите? Скосились гляделки,
закричали:

— Да как вы смеете? Да кто вы? Да кто мы? Да мы вас!

Искали глаза глаз таких же
прекрасных, не нашли и сомкнулись.

Вопросы и задания.

1) Глаза — глазенки — гляделки.

С какой целью использует автор
синонимы в тексте?

2) Как меняют прилагательные и глаголы, оказавшись рядом
с этими синонимами?

3) Что будут представлять собой люди — обладатели глаз,
глазенок, гляделок?

4) Определите разницу между этими словами в лексическом и
стилистическом планах.

5) Глазищи — буркалы — очи.

Дайте стилистическую характеристику этих слов.

II. КУЛЬТУРА РЕЧИ.

Задание 1.

Поставьте ударение в словах.

Алфавит, агент, баловать, диалог,
договор, документ, звонит, изображение, индустрия, искра, каталог, квартал,
километр, коклюш, комбайнер, лыжня, металлургия, мизерный, намерение,
обеспечение, облегчить, начать, предложить, усугубить, партер, револьвер,
свекла, силос, соболезновать, феномен, фундамент, шофер.

Задание 2.

Образуйте множественное число от существительных,
придерживаясь правильного литературного стиля.

Директор, доктор, инспектор,
профессор, инженер, шофер, фельдшер, редактор, отпуск, якорь, трактор, договор,
выбор, цех, фронт, лектор, катер, торт, порт, слесарь, суп, бухгалтер,
библиотекарь, аптекарь, раз, почерк, дно, (кастрюли).

Задание 3.

Исправьте ошибки в предложениях, отредактируйте стиль.

1) Сегодня холодно, поэтому я одела шубу.

2) В магазине завезли импортные моющие обои.

3) К праздничному столу купили пять кило банан.

4) Врачиха выписала мне лекарство.

5) В Индии живут индейки, а в Корее — корейки.

6) Я познакомился с ужасно красивой девушкой.

7) Библиотеку перенесли в более худшее помещение.

В институт я не пошла, так как была заболевшей.

9) Речь этого депутата произвела негативный эффект на
аудиторию.

10) Становьте сюда свой чемодан.

11) В личной жизни я потерпел полное фиаско.

12) На концерте мы видели всех знаменитых артистов, ихние
наряды были потрясные.

13) Отсюдова было хорошо видно море.

14) Я предъявил оплоченный чек.

15) Мальчик добился пересиления чувства страха.

16) Красноречие юноши произвело на девушку надлежащий
эффект.

17) Работу нужно закончить к пятому апрелю.

18) Ребенок держался за коляску обоими руками.

Задание 5.

Раскройте скобки.

1) Мать велела мне купить несколько (простыни).

2) Я не нашла в магазинах ни (апельсины), ни (мандарины),
ни (баклажаны), зато купила 2 (килограммы) (помидоры) и (килограммы) (яблоки).

3) В углу стояло несколько (кочерга).

4) Рим пал от нашествия (варвары).

5) Сегодня мне нужно забрать братишку из (ясли).

6) Отец привез из Самарканда несколько (дыня).

7) В далекий поход по северным землям мы взяли две пары
резиновых (сапоги).

Отряд (солдаты) и группа (партизаны) встретились в
назначенном месте.

9) Я оставался в лесу долго от синих (сумерки) до полных
(потемки).

10) Я встала, сняв ребенка с (колени).

11) В саду много (яблони), (вишни), (смородина).

12) Сколько же (платья) тебе нужно?

13) Я купила весь садовый инвентарь, кроме (грабли).

14) Над болотом кружила пара (цапли).

15) Сколько (доли) от наследства ты получил?

16) В нашем городе много (грузины), (туркмены), (узбеки),
(цыгане), (осетины), (башкиры).

Задание 6.

Близкие по звучанию, но разные по значению слова
называются паронимами. Для того, чтобы проверить, можете ли вы правильно
использовать слова, выберите одно из двух, данных в скобках.

1) Чтобы чаще бывать в театре, я купил (абонент —
абонемент).

2) У девушки были влажные, (чувствительные — чувственные)
губы.

3) В этот день я услышала много (обидных — обидчивых)
слов.

4) Любой (поступок — проступок) заслуживает осуждения.

5) Все вокруг привлекательно, и (близкие — ближние), и
(далекие — дальние) холмы

6) Студент быстро (усвоил — освоил) материал.

7) Молодой рабочий (усвоил — освоил) профессию токаря.

Мальчик отвечал на вопросы с каким-то (виноватым —
виновным) видом.

9) Мать приготовила (сытый — сытный) завтрак.

10) В новых кварталах города находятся самые (высокие —
высотные) дома.

11) (Болотистый — болотный), ил является отличным
удобрением.

12) Один берег реки был луговой, а другой — (глинистый —
глиняный).

13) (Соседний — соседский) кот важно прошел на кухню.

14) После десятимесячного возраста ребенка перевели на
(искусное — искусственное) вскармливание

15) Мы живем в одном доме, но на (разных — различных)
этажах.

16) В жаркий день приятно походить по (теневым — тенистым)
аллеям.

17) В квартире нужно было (провести — произвести) ремонт.

18) На поляне мы увидели высокий (земельный — земляной)
холм.

19) Эти фотографии сохранились в семье потому, что моя
бабушка была очень (бережная — бережливая)

20) Давайте найдем в себе (скрытые — скрытные) резервы и
доберемся до вершины

21) Мой брат — человек очень (практический — практичный),
он не выбрасывает старые веши

22)  С (особым — особенным) удовлетворением учитель отметил
успехи слабого ученика.

23)  Ему была (предоставлена — представлена) возможность
поехать на юг.

24)  Знатоки во всем, он все же был (невежей — невеждой) в
живописи.

25)  Помещики жестоко обращались со своими (придворными —
дворовыми).

Задание 7.

Раскройте скобки, выбрав нужную букву, объясните свой
выбор.

Иструхляв(е, и)ли солому — пни
иструхляв(е, и)ли

враги кровав (е, и) ли руки —
нарывы кровав (е, и) ли

реки обезвод (е, и) ли в засуху —
обезвод (е, и) ли сырой участок

обезденеж (е. и) л после отпуска —
обезденж (е, и) л родителей

обезлес (е, и) ли область —
участки обезлес (е, и) ли от вырубок

обезнож (е. и) л от страха —
паралич обезнож (е, и) л его

обессил (е, и) л в схватке —
обессил (е, и) л противника

пашня обесплод (е, и) ла —
нерадивость обесплод (е, и) ла пашню

штормовка обледен (е, и) ла на
морозе — вьюга обледен (е, и) ла руки

оподл (е, и) ть душой — оподл (е,
и) ть мысли

ведро проржаве (е, и) ло —
коррозия проржав (е, и) ла железо

яблоки червив (е, и) ли день ото
дня — бесхозяйственно гноили и червив(е, и)ли овощи при хранении

кожа шершав (е, и) ла от мороза —
«Черная работа шершав (е, и) т руки» (В.Даль).

Задание
8

Составьте и
запишите предложения с каждой из предложенных конструкций.

Конструкции

Примеры

употребления

Ввиду
(чего?)

В силу
(чего?)

Вследствие
(чего?)

За
неимением (чего?)

В
связи (с чем?)

Благодаря
(чему?)

Согласно
(чему?)

Вопреки
(чего?)

В
случае (чего?)

Задание 9.

Составьте словосочетания по образцу.

Образец. Содействие и координация — Содействие в
работе и её координация

Образовывать
и распоряжаться

внебюджетные
фонды

Руководство
и контроль

нижестоящие
организации

Распределять
и управлять

финансы

Выявление
и наблюдение

деятельность
партнеров

Координировать
и руководить

сотрудники

Задание 10

Найдите ошибки в управлении. Отредактируйте текст.

1. Согласно распоряжения декана вход на факультет
производится только при предъявлении студенческого билета.

2. Вопреки неблагоприятных погодных условий график работы
аэропорта был восстановлен.

3. Благодаря умелого руководства, в истекшем году прибыль
возросла почти вдвое.

4. Управляющий производства отклонил данное предложение.

5. Заместитель декана, в частности, должен курировать
научной работой студентов старших курсов.

6. Вследствие измененного расписания движение поездов
было приостановлено.

7. Согласно приказа ректора университета переходит на
шестидневную форму обучения.

8. В обязанности начальника над подразделением
гражданской обороны входит информирование подчиненных о возникновении
чрезвычайной ситуации.

Задание 12.

Объясните смысл образных выражений, вспомните их
источник. Представьте ситуацию, в которой можно употребить эти выражения.

1) Метать перуны;

2) курить
фимиам;

3) пиррова победа;

4) перейти Рубикон;

5) танталовы муки;

6) грубый
топтыгин;

7) Шемякин суд;

тришкин кафтан;

9) настоящий Дон Жуан;

10) синий
чулок;

11) легкая, как Терпсихора;

12) прокрустово ложе;

13) настоящий
Квазимодо;

14) гомерический смех;

15) дамоклов меч;

16) гордиев узел; этакий
Голиаф;

17) современный Дон Кихот;

18) мамаево побоище;

19) современные
монтекристо;

20) робинзонада;

21) фигура как у Аполлона;

22) троянский конь;

23) петь дифирамбы;

24) вылитый Плюшкин;

25) вернуться в свои пенаты.

Задание 13.

Постарайтесь по стилю речи отгадать, из произведений
каких писателей и поэтов взяты отрывки.

Комната была, точно, не без
приятности: стены были выкрашены какой- то голубенькой красной вроде серенькой,
четыре стула, одно кресло, стол, на котором лежала книжка с заложенной
закладкою, о которой мы уже имели случай упомянуть, несколько исписанных бумаг,
но больше всего было табаку. Он был в разных видах: в картузах и в табачнице,
и, наконец, насыпан был просто кучею на столе.

1. Настанет день — и миром осужденный,

Чужой в родном краю,

На месте казни — гордый, хоть
презренный Я кончу жизнь мою.

2. В марте семнадцатого года в один прекрасный день
пропали все калоши, в том числе две пары моих, три палки, пальто и самовар у
швейцара. И с тех пор калошная стойка прекратила свое существование. Пусть: раз
социальная революция — не нужно топить…. Почему убрали ковер с парадной
лестницы? Разве Карл Маркс запрещает держать на лестнице ковры?

3. Где, укажите нам, отечества отцы,

Которых мы должны принять за
образцы?

Не эти ли, грабительством богаты?

Защиту от суда в друзьях нашли, в
родстве,

Великолепные соорудив палаты,

Где разливаются в пирах и
мотовстве,

И где не воскресят клиенты –
иностранцы

Прошедшего житья подлейшие черты.

4. Травка зеленеет,

Солнышко блестит,

Ласточка с весною

В сени к нам летит.

5. Умом Россию не понять,

Аршином общим не измерить:

У ней особенная стать

В Россию можно только верить.

6. Приемная комната знаменитого петербургского адвоката
была полна, когда Алексей Александрович вошел в нее. Три дамы: старушка,
молодая купчиха, три господина: один — банкир — немец с перстнем на пальце,
другой — купец с бородой, третий — сердитый чиновник в вицмундире, с крестом не
шее, очевидно, давно уже ждали. Два помощника писали на столах, скрипя перьями.

Задание 14.

Отредактируйте тексты.

Вариант 1

1. Большинство учителей и методистов объединило свои
усилия в решении важной проблемы.

2. Уж, кажется, на что Татьяна любила Онегина, а нашла в
себе силы достойно отказать ему.

3. 3.0 том, как прошел матч, можно посмотреть по первой
программе телевидения.

4. Перстневой нож
одевают на вторую фалангу среднего пальца руки вязальщицы.

5. Рабочие места слабо освещались, в силу чего имелось
много случаев производственного травматизма.

6. У всего этого фамусовского сброда на уме одно: балы да
веселья.

7. Несколько ниже мы поговорим об этом.

8. Важным фактором успешной сдачи экзаменов является
систематическое выполнение домашних заданий.

9. Главное, на чем настаивали рабочие, — это на
требовании повышения зарплаты.

10. Читаешь Толстого — и укрепляется вера в то, что жизнь
— великая ценность и что она, жизнь, продолжаться будет, и мысль продолжаться
будет.

Вариант 2

1. После случая с «болотной копейкой» Павел носа не
повесил, а продолжал с удвоенной энергией тусоваться с рабочими.

2. Варвара говорит Катерине, что я не умела лгать, но
научилась.

3. Виновниками семейных
вспышек бывают взрослые, которые переносят грипп на ногах.

4. Судя по тем находкам, которые они нашли, уже есть
веские доказательства в пользу этой гипотезы.

5. Цензура не
хотела, чтобы поэма Гоголя увидела свет, но близкие Гоголя подсуетились и
пробили это дело.

6. На научном семинаре автор изложил главную суть этой
книги.

7. Путь, который избрал герой, ошибочен, неправильный.

8. Появление в рабочей слободке новых людей является
прогрессивным явлением.

9. Решение Пьера было воспринято как должное. Никто и
оспаривать-то его не стал, так как против фактов не попрешь.

10. Уши у него торчали кто куда.

Вариант 3

1. Простив Анну, Каренин не понизил своего достоинства.

2. Наша передовая
свинарка воспитала 257 поросят.

3. Машина будет
использована не только для решения задач, а также для контроля.

4. Водитель должен помнить, что, повернув с проспекта
направо, ему придется пересечь трамвайные пути.

5. Ленский вызвал Онегина на дуэль за то, что тот
цеплялся к Ольге.

6. Как вывести личность на новый уровень знания?

7. Содержание банки
следует вылить в кастрюлю.

8. В Никитском ботаническом саду открылась для посетителей
коллекция хризантем.

9. Андрей Болконский ищет смысл в подвиге, который он
хочет завоевать не в штабе, а на поле боя.

10. Он раскрылся нараспашку.

III. Контрольное тестирование.

Орфография и пунктуация.

Задание к текстам:

Раскрыть скобки, вставить пропущенные буквы и знаки
препинания.

Текст 1

Во дворе сразу два события пил…т
дрова и летают грачи. Денек окончательно разгулялся. Грачи с сум…тошным гамом
носятся над тополем. Лиш… на один миг пр…саживаются они на вершине дерева и
тогда видно как на сложе…ых крыльях вспыхивают солнечные блики.

Тополь то (же) оз..рен солнцем.
Толстые обн…женые ветви выделяются на веселой с…неве. Ветви заканчиваются
спл…шной путаницей мельчайших веточек.

Всю долгую жизнь тополь тян…тся
к солнцу. Он протягивает ветви на восток что(бы) встретить св…тило в самые
первые мгн…вения как только оно покаж…тся над краем земли. На юг простерто
ветвей больше чем в любую другую потому что в (полу)де…ные часы солнце очень
щедро. Даже тополь не упускает момента воспользоват… ся последним теплом
светом. Но и северная сторона то(же) во всеоружи… . Дерево и туда простерло
свои ветви на всякий случай если солнце вздумает заглянуть с этого края. Каждый
год он наращивает самые тонкие ветви новыми побегами что (бы) ей пр…близится
к св…тилу. Он не выглядывает из (за) заборов и сараев он поднялся над всем
этим что (бы) (н…) что (н…) мешало видеть источник света и первым встречать
его на восходе. Каждый раз он старат…льно располагает свои листья так что(бы)
н… один луч… упавший на него н… прол…тел мимо в какую(нибудь) пр…реху
в его кроне. Он раскинул над собой такой шатер что даже капля дождя не сможет
упасть на землю не(задев) листа. А что (бы) случайная буря не разрушила вековой
работы год от года наращивает мощ… ветвей и все глубже уходит корнями в
землю. Ему еще многие десятилетия держать на себе свое обширное поднебесное
царство.

(По Е. Носову)

Текст 2.

Старый скрипач (музыкант) любил
играть у п… дножия памятника Пушкину. Этот памятник стоял в Москве (в) начале
Тверского бульвара на нем написаны стихи и со всех четырех сторон к нему
поднимаются мрам . рные ступени. Поднявшись по этим ступеням к самому
п…ед…сталу старый музыкант обращался лиц…м на бульвар к дальним Никитским
воротам и трогал смычком струны на скрипке. У памятника сейчас (же)
соб…рались дети прохожие чтецы газет из местного к…оска и все они
ум…лкали в ожидани… музыки потому что музыка ут…ша…т людей она
обеща…т им счастье и славную жизнь. Футляр со своей скрипки музыкант клал на
землю против памятника он был закрыт и в нем лежал кусок черного хлеба и
ябл…ко что(бы) можно было поесть когда захоч.тся.

Обыкновенно старик выходил играть
под вечер. Для его музыки было полезней что

(б) в мире стало тише и темней. Старик скучал
от мысли, что он не приносит людям (н…) какого добра и поэтому добр…вольно
ходил играть на бульвар. Там звуки его скрипки разд…вались в воздух… в
сумрак… и хоть (из)редк… они доходили до глубины человеческого сердца
трогая его нежной и мужестве… ой силой увл…кавш…й жить высш…й пр..
красной жизнью. Некоторые слушатели музыки вын…мали деньги что(бы) под.,.рить
их старику но не знали куда их пол…жить футляр от скрипки был закрыт а сам
музыкант находился высоко на п…днож…, памятника почти рядом с Пушкиным.

Уходил домой он поздно иногда уже
в (пол) ночь, когда народ становился редким и лиш… какой (нибудь) случайный
одинокий челвек слушал его музыку.

(По А. Платонову)

Текст 3.

В эту (же) зиму узнал я и очень
горячо полюбил Писарева который находился еще в университете благородном п..
нсионате впрочем это было только за(пол)года перед выпуском воспита…иков. Он
участвовал в наших сп…ктаклях хотя сценических способностей у него не было
так (же) как у Загоскина. (Не)смотря на (восемнадцати)летнюю молодость
блестящий острый ум Писарева был уже серьезен и глубок. Вся пансионская
молодеж… признавала его превосходство и все кто знал смотрели на Писарева как
на будущего славного писателя его проза и стихи пр…возносились (не)только товарищами
и начальством пансиона но и всеми; театр литература были его пр.. званием
страстью жизни.

С первого свидания Писарев
почу…ствовал искре…ность моего участия и полюбил меня как брата; все время
свободное от клас.. .ного ученья он проводил у меня в доме. Писарев имел
раздражительный но сосредоточе… ный характер внешнее выр.. жение у него было
тихо спокойно и холодно даже и тогда когда он задыхался от внутреннего
волнения. Он не краснел (н…) от гнева (н…) от радости а бледнел.

Писарев вышел вторым
воспита…ником из университетского пансиона он был очень дружен только с одним
товарищ…м своим Юшневским; оба они получили при выпуск… десятый класс… но
имя Писарева написа…ое золотыми буквами на мраморной доске осталось
(на)всегда между именами Жуковского и других отличных воспита.Ников. В ту(же)
минуту после пансионского акта и за несколько часов до личного со мной свидания
Писарев написал ко мне м…лодое горячее письмо которое к сожалению потеряно.

(По С. Аксакову)

Текст 4.

Пр…хотли во извива…щаяся линия
м…ршрута ползет по карте (в)след за ними и врезается (в)глубь Владимирских
земель. Два паренька лет по двенадцати едут с нами за во ..чиков. Одного зовут
Коля другого Николай. Так они просили называть их (что)бы не было пута. Ницы. Они
рове…ники одного р…сточка оба (русо)головые бойкие смышле…ые. Кажется и
разница вся между ними только в произношении их то(же) одинаковых имен.
Чу…твуется что Коля и Николай (не)твердо знают дорогу и волнуются как(бы) не
завести чужих людей куда (не)надо.

Устраивайтесь, как следует
(по)хозяйски предлагает Коля. Мальч,..нки уходят в кусты и возвращаются с двумя
охапками (не) давно накош…ной травы перемеш…ной с цветами. Они разравнивают
ее по телеге. Едем (не)спеша. На подъеме в гору я спрыгнул с телеги и пошел
(по)тихоньку (с)зади Коля с Николаем то(же) соск…чили.

Так и едем (по) ровному месту на
телеге в гору пешком а с горы трусц…й и даже

(в)скач…
Плывут (на) встречу перелески осталась (по)зади стари…ая роща и (в) скоре мы
в…езжаем в самый настоящий лес перепута…ый (на)столько что без топора и не
пробраться. Это (в) сущности (не) просто лес а поле битвы (не) прекращающ…ся
(не, ни) днем (не, ни) ночью.

Чем дальше мы ехали (в) глубь леса
по у…кой дороге по которой до нас (в) ряд ли кто проехал в предыдущие два
месяца тем тревожнее переш…птывались Коля и Николай. (Не)ожнданно перед нами
возникла (не)широкая но глубокая канава из которой т.. рчали два облома…ых
скользких бревна.

Обязанности распр…делились так я
тянул лошадь (под) уз…ы Коля правил а Николай понукал упира…щуюся лошадь.

(В) сторону вдруг (не)своим
голосом закричал один из мальчиков. Инст…нктивно я отпрянул (в)сторону и в
то(же) мгновение на уровне моего лица мелькнуло в воздухе кова.. .ое лошади..
.ое копыто.

Нужно было обладать отнюдь
(не)мальчишеской опытностью что(бы) предугадать прыж…к лошади и (во)время
предупредить о нем.

А лошадь уже как н… (в)чем н…
бывало стояла на том берегу утопая в разноцветь… и разнотрав…

Через (пол)часа мы в…ехали в Дубки
то(есть) попали на мощ…ную дорогу соединяющую Владимир с Кольчугин…м.
(От)сюда хорошо было оглянуться (на) зад. До г…ризонта сплош.. .р..
.ст…лались леса черные (в)близи и (темно)синие (в)дали. —

Там (по)зади прот.. .нулась
(не)зримая ниточка пройд…ного нами пути.

(По Вл. Солоухину)

Текст 5.

Было далеко (за)полноч… когда я
вышел из дома. Все спало кругом и в безмолв… ночи гулк… и …диноко…
раздавались шаги мои по кр…мневой дороге. Обогнув небо луна стояла уже над
горою и ясным зеленоватым светом заливала Железноводск (по) которому я
пр…ходил и тень моя шла (в)переди меня длин…ая бледная и к…сая.

На плошад… на ступеньках
ц…рковной пап…рти меня ждал Халим сидя с поджатыми (под)себя ногами.
Залитый лун…ым светом как серебром базар был пуст и его до…атые шалаши где
целый день галдят татары и казаки стояли безлюдный м…лчаливы и всюду в…
круг было то(же) (без) молвно и пусты..но только топот наших коней ударявших по
кремнию п…дковами нарушал окрес…ную тишину и …ткликался эхом в горах. Под
лун…ым с…янием развертывалась перед нами вся ширь с лугами и холмами и было
видно далеко окрест.

Миновавши базар Халим п…вернул
(в)лев… по лесной тр…пинке. Сразу стало темно душно и у…ко и лошади пошли
шагом то шлепая ногами по лужам то звонко задевая подковой о камень или
сп…тыкаясь о корни. Кривой и чистый лес обступивший тр…пинку пр…никнутый
лу…ным светом ст…новился все глубже и темнее и только листья (на)верху
колеблясь от ветра то св…ркали то чернели то крутились мелкими блес…ками и
казалось что в лесу идет волшебный серебряный дождь. Наконец стало совсем
темно. Время шло.в лесу было совершено темно и душно и я не п…дозр;..вал что
в открытых д…линах поднялся уже предра…светный ветер.

(По Н. Телешову)

Текст 6.

Охота ок…залась хуже чем ожидал
Левин. Болото высохло и дупелей совсем (не)было. Он проходил целый день и
принес только три штуки но (за)то принес как и всегда с охоты отличный
ап…етит отличное расположение духа и то возбужденное умственное состояние
которым всегда сопровождалось у него сильное физическое движение. В то время он
казалось (н…) о чем не думал нет-нет и опять ему вспоминался старик со своей
семьёй и впечатления это как будто требовало к себе не только внимания но и
разрешения чего-то с ним связанного.

Вечером за чаем в присутствии…
двух помещиков приехавших по каким (то) делам опеки завязался тот самый
интересный разговор какого и ожидал Левин.

Левин сидел подле хозяйки у
чайного стола и должен был вести разговор с нею и свояченицей сидевшею против
него. Хозяйка была кругло(лицая) (бело)курая и (не) высокая женщина вся сияюшая
с ямочками.

Мучительно (не) ловко ему было
(от) того что против него сидела свояченица в особе…ном для него ему казалось
надетом платье с особе…ным (в) виде трапеции вырезом на белой груди этот
(четверо) угольный вырез (не) смотря на то что грудь была очень белая или
особе…но (по) тому что она была очень белая лишал Левина свободы мысли. Он
в…ображал себе вероятно ошибочно что вырез этот сделан на его счет и считал
себя не (в) праве смотреть на него и старался не смотреть на него но чувствовал
что он виноват уже за одно то что вырез сделан. Левину казалось что он кого
(то) обман…вает что ему следует объяснить что(то) но что объяснить этого (ни)как
нельзя и (по)этому он бе… преста…но краснел был беспокоен инеловок.
Неловкость его сообщалась и хорошенькой свояченице. Но хозяйка казалось не
замечала этого и нарочно втягивала ее в разговор.

Текст 7.

Утром Чагатаев заметил по
местности что здесь должна быть вода и нашел заглуше…ный колодец забитый
песком. Назар дорылся руками до влажной глубины начал ж…вать песок но
спл…вывать пр…ходилось больше чем получать (в)нутрь тогда он стал гл…тать
мокрый песок ц…ликом и мучень… жажда оставило его. В следующие четыре дня
Чагатаев старался идти (в)перед по пустыне но от слабости уходил (не)далеко и
вновь возвращался на мокрый песок что (бы) изн…могая от голода не ум…реть
от жажды. На пятый день он остался на месте решив набраться сил в дремоте и
(бе…)памятстве а затем догнать свой народ.Он с ..ел два оставшиеся у него
хин…ых порошка и разные карма…ные крошки (от)чего ему стало лучше. Он
понимал что народ его близко он то (же) не имен сил уйти от него далеко только
(не) известно было направление его пути.

Чагатаев закрылся от солнца во
влажный песок и пытался впасть в беспамятство для отдыха и для экономии жизни
но не умел и все время думал жил (по)немногу и смотрел в небо где слабым
туманом шел жаркий ветер с (юго)востока и было так пусто что не верилось в
существование твердого настоящего мира.

» Отл…жавшись Чагатаев пополз к
ближайшему бархану где он заметил задутый (на)половину песком куст
(перекати)поля. Он добрался до него отломил несколько высохших ветвей и
сж…вал их а оставш…йся куст вырыл из песка и отпустил бродить по ветру.
Куст покатился и (в)скорее исчез за барханами направляясь

куда(то) в дальн…ю землю.
Скатившись с бархана Назар заснул у его подножия и во сне на его слабое
сознание напали разные воспоминания бе…цельные забытые впеч…тления
воображения скучных лиц виденых когда(то) однажды вся прожитая жизнь вдруг
повернулась | назад и напала на Чагатаева.

(По А.Платонову)

Текст 8.

В то утро в Москве стояла (мало)
облачная погода со слабым (юго) западным ветерком.

Полю знобило от волнения (не)
смотря на ее (темно) синий свитерок связанный матерью перед от…ездом. По
затоптанной лестнице она поднялась на третий этаж и через казарменный коридор
(не)сколько (не)опрятный из(за) множества раскле…ных уведомлений на стенах
вместе со всеми вошла в ауд.. .торию.

Это было (полу)темное и (не)уютное
помещение но веселая зелень за окнами отражалась в вынош…ном п…ркете и
тяж…лом нависающ…м потолке чем (не)сколько ра…сеивалось первое
впечатление академического холода и неприязни.

Все было забито (до) отказа кроме
новичков со всех факультетов сюда в этот день так (же) приходили студенты —
(старше) курсники. Мест (не) хватало и Поле пришлось усес…ся (на) верху под
потолком и…ч…рканном карандашными надписями.

Ее соседка высокая загорелая
девушка (по)видимому ее рове… ница поделилась с ней восторжен…м
воспоминанием о такой(же)лекции прошлого года когда по ее словам стены здании
как(будто)раздвинулись и дремучий брусничный бор живо представился слушателям.

Попривыкнув Поля огляделась. В
переди далеко (в)низу стоял бедный залитый чернилами стол; дальше на стене
теснились портреты бородатых п…триархов отечественной лесной науки.

Гул голосов схлынул и за столом
появился подвижный (не) большого роста старик с проседью на висках. Он начал
говорить с (не) привычной для официального места образностью избегая ц…фр
способных затруднить (не)подготовленное внимание новичков. (Из)редк… он
умолка опираясь кулаком в стол и как(бы) выискивая возможных оппонентов. Какая
(то) сближаюшая искренность возр…стала (по)мере углубления в тему и (ни)кто
не заметил когда и как лекция превратилась в без…скуссвенный разговор старого
лесника с буду…ими товарищами по профессии.

(По Л. Леонову)

Персональное собеседование один на один с каждым сотрудником какая речевая ошибка допущена

Плеоназмы

Плеоназм — это рече­вое изли­ше­ство, упо­треб­ле­ние соче­та­ния слов, в кото­ром смысл одно­го сло­ва уже зало­жен в зна­че­нии дру­го­го. Плеоназм в рус­ском язы­ке — это лек­си­че­ская ошибка.

Лучший юмористический канал: анекдоты, приколы, юмор и куча ржачных видео

В пись­мен­ной и раз­го­вор­ной речи незна­ние точ­но­го зна­че­ния сло­ва, осо­бен­но заим­ство­ван­но­го рус­ским язы­ком, часто сопро­вож­да­ет­ся допу­ще­ни­ем лек­си­че­ских оши­бок, одной из кото­рых явля­ет­ся плео­назм. В линг­ви­сти­ке этот тер­мин вос­хо­дит к гре­че­ско­му сло­ву pleonasmos, что бук­валь­но зна­чит «пере­из­бы­ток».

Что такое плеоназм в русском языке

В раз­ных сфе­рах нашей жиз­не­де­я­тель­но­сти нам часто встре­ча­ет­ся сло­во­со­че­та­ние «сер­вис­ное обслу­жи­ва­ние». Его упо­треб­ля­ют для обо­зна­че­ния каче­ствен­но­го обслу­жи­ва­ния. Давайте заду­ма­ем­ся, насколь­ко пра­во­мер­но объ­еди­ня­ют­ся эти сло­ва с точ­ки зре­ния лек­си­че­ских норм рус­ско­го лите­ра­тур­но­го языка?

Прилагательное «сер­вис­ное» обра­зо­ва­но от сло­ва «сер­вис», заим­ство­ван­но­го из англий­ско­го язы­ка, в кото­ром service зна­чит «быто­вое обслу­жи­ва­ние».

Значит, зна­че­ние сло­ва «сер­вис­ный» уже вхо­дит в семан­ти­ку рус­ско­го сло­ва «обслу­жи­ва­ние» и явля­ет­ся лиш­ним в этой паре лексем.

Рассмотрим сло­во­со­че­та­ние «меню блюд». Французское сло­во «меню» обо­зна­ча­ет «под­бор блюд для зав­тра­ка, обе­да и т. д.», а так­же «лист с переч­нем пред­ла­га­е­мых блюд, напит­ков в ресто­ране, кафе, сто­ло­вой». И в этом соче­та­нии слов допу­ще­на рече­вая ошиб­ка — плеоназм.

Как видим, эта ошиб­ка воз­ни­ка­ет тогда, когда гово­ря­щий или пишу­щий не вни­ка­ет в зна­че­ние слов или не зна­ет точ­но­го их зна­че­ния и вкрап­ля­ет в свою речь лиш­ние с точ­ки зре­ния смыс­ла слова.

Определение

Укажем, какое опре­де­ле­ние этой рече­вой ошиб­ке дает Википедия.

Таким обра­зом, плео­назм — это более широ­кое поня­тие, кото­рое вклю­ча­ет не толь­ко упо­треб­ле­ние отдель­ных лиш­них слов, а целых обо­ро­тов речи и даже фраз, кото­рые мож­но упро­стить или изъ­ять из тек­ста вообще.

Примеры плеоназмов

Чаще все­го ука­зан­ной рече­вой ошиб­кой стра­да­ют соче­та­ния при­ла­га­тель­ных с суще­стви­тель­ны­ми, при­чем зна­че­ние при­ла­га­тель­но­го дуб­ли­ру­ет смысл опре­де­ля­е­мо­го им слова:

В каче­стве глав­но­го сло­ва в соче­та­нии высту­па­ет гла­гол, в зна­че­нии кото­ро­го уже зало­жен смысл лиш­не­го слова:

  • упасть вниз;
  • под­прыг­нуть вверх;
  • впер­вые познакомиться;
  • вер­нуть­ся обратно;
  • сжать кулак;
  • импор­ти­ро­вать из-за рубежа;
  • гос­пи­та­ли­зи­ро­вать в стационар;
  • пре­ду­пре­дить заранее.

Плеоназм и тавтология. Отличия

В лек­си­ко­ло­гии раз­но­вид­но­стью плео­назма счи­та­ет­ся тав­то­ло­гия (греч. tauto «то же самое» + logos «сло­во»).

Тавтология — это непред­на­ме­рен­ное упо­треб­ле­ние одно­ко­рен­ных слов в сло­во­со­че­та­нии или в одной фра­зе, а так­же необос­но­ван­ный повтор одно­го и того же слова.

Все спортс­ме­ны долж­ны сгруп­пи­ро­вать­ся в неболь­шие груп­пы по трое.

Ему надо прыг­нуть пры­жок пря­мо сейчас.

Петр оза­да­чил всех сотруд­ни­ков этой труд­ной зада­чей .

Авторская речь — это речь авто­ра .

Исходя из того, что в плео­назме дуб­ли­ру­ет­ся смысл язы­ко­вых еди­ниц, состав­ля­ю­щих сло­во­со­че­та­ние, но лек­се­мы не явля­ют­ся одно­ко­рен­ны­ми, мож­но утвер­ждать, что плео­назм — это скры­тая смыс­ло­вая тавтология.

Наше сов­мест­ное сотруд­ни­че­ство было плодотворным.

Сотрудничество — это дей­ствие, рабо­та вме­сте, уча­стие в общем деле.

Плеоназм созда­ет­ся в речи, когда дру­гим сло­вом обо­зна­ча­ет­ся одно и то же поня­тие, уже назван­ное лек­се­мой, состав­ля­ю­щей с ним соче­та­ние или фразу.

Тавтология же явля­ет­ся рече­вой ошиб­кой, где явно упо­треб­ля­ют­ся одно­ко­рен­ные сло­ва, созда­ю­щие излиш­ний назой­ли­вый повтор лек­сем с оди­на­ко­вым или похо­жим смыс­лом. С этой точ­ки зре­ния тав­то­ло­гия — это откры­тое язы­ко­вое изли­ше­ство в речи.

Примеры тавтологии

  • спро­сить вопрос;
  • зара­бо­тан­ная зарплата;
  • про­лив­ной ливень;
  • зво­нок звонит;
  • дымит­ся дымом;
  • вновь воз­об­но­вить;
  • город­ской градоначальник.

Тавтология быва­ет оправ­да­на толь­ко в текстах, напи­сан­ных в официально-деловом или науч­ном сти­ле, где повтор одно­го и того же сло­ва необ­хо­дим по смыс­лу высказывания.

Размещение недоб­ро­ка­че­ствен­ной рекла­мы с дан­ным содер­жа­ни­ем в дан­ном месте дан­ным спо­со­бом запре­ще­но зако­но­да­тель­ством страны.

В отли­чие от плео­назма в поэ­ти­че­ском язы­ке тав­то­ло­гия исполь­зу­ет­ся как один из видов повто­ров, уси­ли­ва­ю­щих эмо­ци­о­наль­ность и выра­зи­тель­ность речи. Повторяются либо одно­род­ные по сво­е­му зву­ча­нию и по смыс­лу сло­ва (гре­ет — погре­ва­ет, веет — пове­ва­ет), либо повто­ря­ют­ся сло­ва, раз­ные по зву­ча­нию, но близ­кие по смыс­лу (зна­ет — веда­ет, пла­чет — тужит, море-океан, тоска-печаль).

Если в риф­ме повто­ря­ет­ся одно и то же сло­во в изме­нён­ном его зна­че­нии, такую риф­му назы­ва­ют тавтологической:

Вот на берег вышли гости,
Царь Салтан зовёт их в гости.

А. С. Пушкин. Сказка о царе Салтане

Словарик плеоназмов

Персональное собеседование один на один с каждым сотрудником какая речевая ошибка допущена

Количество участников, получивших 1 балл
(всего 93234 участника)

Процент участников, получивших 1 балл

Задание 1. Чтение вслух

Интонация соответствует/ не соответствует пунктуационному оформлению текста

Темп чтения соответствует/ не соответствует коммуникативной задаче

Задание 2. Пересказ текста с включением высказывания

Все основные микротемы исходного текста сохранены/упущена или добавлена микротема

Фактических ошибок нет/ допущены фактические ошибки

Высказывание включено в текст уместно, логично/ не включено или приведено неуместно и нелогично

Ошибок при цитировании нет/ есть ошибки при цитировании

Грамотность речи (задания 1 и 2)

Грамматических ошибок нет/ допущены грамматические ошибки

Орфоэпических ошибок нет, или допущено не более 1 орфоэпической ошибки (исключая слово в тексте с поставленным ударением)/ допущены 2 и более орфоэпические ошибки

Речевых ошибок нет, или допущено не более 3 речевых ошибок/ допущены 4 и более речевых ошибок

Искажений слов нет/ допущены искажения слов

Задание 3. Монолог

Приведено 10 или более фраз по теме высказывания без фактических ошибок / приведено менее 10 фраз, и/или допущены фактические ошибки

Речевая ситуация учтена/ речевая ситуация не учтена

Высказывание характеризуется смысловой цельностью, речевой связностью и последовательностью, логикой изложения/высказывание не логично, изложение непоследовательно, допущены логические ошибки

Задание 4. Диалог

Даны ответы на все вопросы/ ответы не даны или даны односложные ответы

Речевая ситуация учтена/ речевая ситуация не учтена

Грамотность речи (задания 3 и 4)

Грамматических ошибок нет/ допущены

Орфоэпических ошибок нет, или допущено не более 2-х ошибок/ допущено 3 или более орфоэпические ошибки

Речевых ошибок нет, или допущено не более 3 речевых ошибок/ допущены 4 или более речевые ошибки

Речь отличается богатством и точностью словаря, используются разнообразные синтаксические конструкции/ отличается бедностью и/ или неточностью словаря, используются однотипные синтаксические конструкции

Как максимально эффективно и правильно провести собеседование

Не только от соискателя, но и от работодателя проведение собеседования требует огромной предварительной подготовки. Первым делом необходимо определиться с тем, какие требования выдвигать кандидату на вакантную должность. Кроме этого, следует учесть множество факторов, к примеру, какие ошибки могут возникнуть при приеме на работу и как правильно провести собеседование.

Зачастую при наборе кандидатов на должность совершается ряд ошибок, которые ведут к тому, что вместо креативных сотрудников можно принять хороших исполнителей, которые будут отлично выполнять свою работу, но не смогут сориентироваться при модернизации фирмы. Так, если станет необходимым поиск новых путей развития компании, то такие сотрудники не смогут даже внести какую-либо идею. Таким образом, следует учитывать неординарность мышления кандидата и его готовность посвятить себя работе.

Классические ошибки подбора персонала

Талант производить качественный набор сотрудников имеет каждый менеджер. Но чтобы действительно не прогадать на «отборочном туре», необходимо проявить смекалку. Первым делом нужно знать самые распространенные ошибки менеджера при отборе сотрудников, чтобы их избежать:

Требования к образованию

Чаще всего фирмой выдвигаются требования, чтобы сотрудник не просто имел высшее образование по специальности, но и мог представить подтверждение начального рабочего стажа (не менее трех лет). Таким образом, самая распространенная ошибка – брать специалистов с опытом работы и пренебрегать свежими идеями, которые может дать молодой специалист, не обремененный чужим опытом и правилами. Такие люди способны рисковать, что зачастую может привести компанию в выигрыш.

К примеру, выгодная сделка может состояться не только с помощью традиционного подхода: современные управители (главы корпораций) зачастую ищут неординарных людей, которые могут представить проекты, качественно отличающиеся от традиционных программ.

Личностные качества

К сожалению, во время приема на работу учитываются только те качества характера, которые могут пригодиться на работе, при этом под ними чаще всего подразумеваются трудолюбие, прилежность, пунктуальность, умение не пререкаться с начальством и без лишних вопросов выполнять свою работу. При этом в расчет зачастую не берутся такие стороны личности, как коммуникабельность, умение найти альтернативный подход, поиски необычных путей решения проблем фирмы, креативность.

Специальные требования

Чаще всего проводится отбор кандидатов. Специальными требованиями могут быть возрастные ограничения или набор по половому признаку. Также могут учитываться рекомендации с предыдущего места работы, периодичность смены места работы и причины увольнения (если человек часто меняет работу по собственному желанию, это может указывать на его непостоянство и ненадежность как сотрудника).

Зачем необходимо собеседование?

Основное значение собеседования – определить подходящих кандидатов, которые смогут полностью соответствовать вакантной должности.

Только в таком случае можно быть уверенным, что новый сотрудник полностью справится с возложенными на него задачами. Пример собеседования при поборе кадров можно представить в виде такой инструкции:

  1. Приветствие. Первым делом необходимо проверить, тот ли человек перед вами, который присылал свое резюме и подавал заявку на вакансию (бывают случаи жульничества, когда приходил другой человек и замечательно справлялся со всеми задачами, которые перед ним ставит комиссия). Постарайтесь сделать все, чтобы кандидат как можно быстрее адаптировался. Для этого достаточно рукопожатия и пары предложений о погоде.
  2. Введите претендента на должность в курс дела, поведайте цель собеседования. Если на собеседовании заседает комиссия, важно объяснить ее роль.
  3. Начинайте опрос кандидата. Следите, чтобы каждый следующий вопрос был сложнее предыдущего. В конце можно задавать открытые вопросы.
  4. Будьте объективны в суждениях. Старайтесь дать возможность претенденту высказаться. Проявляйте гибкость: при необходимости можно отказаться от некоторых вопросов в пользу развития интересной темы. Также важным является поддержание зрительного контакта.
  5. Кратко расскажите о работе и должности: зарплата, отпуск, график.
  6. Прежде чем завершить собеседование, необходимо поинтересоваться о том, есть ли вопросы у соискателя. Скажите спасибо человеку за встречу и сообщите о сроках решения по вакансии. Не следует прямо на собеседовании решать, принимать человека или нет. Информацию необходимо проверить и проанализировать. Если решение будет принято впопыхах, то при отказе в дальнейшем могут возникнуть проблемы, поскольку соискатель спокойно может передать дело на рассмотрение в суд.

Правила

Чтобы первая встреча с кандидатом прошла успешно и результативно для обеих сторон, нужно выполнять основные правила проведения собеседования:

  • Собеседование проводится не со всеми кандидатами, а только с теми, кто был отобран в результате отсеивания.
  • Никогда не пытайтесь подсказывать кандидату или наталкивать его на мысль, поскольку таким образом невозможно четко оценить знания и реальные навыки кандидата.
  • Если вы поняли, что кандидат не подходит, просто заканчивайте собеседование и не тратьте времени впустую.
  • Если вы поняли, что данному человеку лучше работать на другой должности в вашей фирме – предложите ему вакансию.
  • После окончания собеседования можете огласить оценку качеств каждого участника. Их можно письменно проинформировать (представить ксерокопию оценки комиссии). Это поможет им в дальнейшем проработать ошибки и добиться успеха в другой фирме.

Варианты проведения собеседования

Собеседование можно проводить несколькими способами:

Собеседование «один на один»

Это классический вид общения, когда с кандидатом наедине общается начальник, администратор, представитель отдела кадров или другое доверенное лицо компании. При таком виде собеседования можно без лишних стрессов поговорить с кандидатом. При этом обстановка будет располагать к откровенному общению, сотрудник не будет излишне переживать и сможет проявить себя с лучшей стороны. Такой способ позволит сложить правильное впечатление о человеке и проверить его профессиональные качества.

Встреча нескольких кандидатов с менеджером (доверенным лицом компании)

Подобный вид собеседования имеет свои недостатки и преимущества. К примеру, если вам нужно узнать, кто более стрессоустойчив и умеет вести борьбу с конкурентами, то такой способ – лучший вариант для сравнения кандидатов и их умения проявить себя, выделиться среди толпы. При таком собеседовании можно дать возможность посоревноваться в профессионализме: пусть кандидаты задают друг другу каверзные вопросы по специальности. Такая игра позволит увидеть, кто из них наиболее готов к должности.

Собеседование, при котором один кандидат общается с несколькими представителями фирмы

Для таких случаев собирается комиссия, члены которой будут задавать вопросы кандидату. Данный способ позволит не только узнать профессиональные качества и личностные особенности опрашиваемого, но и проверить его стрессоустойчивость.

Встреча нескольких кандидатов с отборочной комиссией

Подобные методы проведения собеседования позволяют провести сравнительную характеристику всех качеств кандидатов. При этом оценка будет выставляться непредвзятая, так как все качества будут взвешиваться всеми членами комиссии и по каждому кандидату будет высказано общее мнение.

Этот способ проведения собеседования требует определенных расходов фирмы. Так, руководитель должен одобрить заседание комиссии, выделить помещение и некоторые ресурсы. Также должен быть выписан приказ на разрешение заниматься членам комиссии отбором кандидатов вместо выполнения своей работы. Необходимо также решить много бюрократических моментов в виде распоряжений относительно сбора комиссии.

Принцип отбора кандидатов

После проведения различных методов отбора персонала: анкетирования, собеседования, ознакомления с резюме и других подводится качественная оценка каждого кандидата. При этом учитываются его ответы на собеседовании, а также вся информация, отображенная в резюме и анкетных данных. За время общения с кандидатом проводится сравнительная характеристика его личностных качеств. Это позволяет при дальнейшей работе знать, чего ожидать от такого работника.

Чаще всего некоторые личностные качества могут быть в приоритете в зависимости от вакантной должности. Помимо личностных качеств, существует также необходимость проверки профессиональных навыков и теоретических знаний. Только по прохождении всех видов отбора можно будет окончательно решить, подходит ли кандидат на вакантную должность или нет.

Вариант 1

1. Расставьте ударение в приведенных словах и выберите одно лишнее (с акцентологической точки зрения) слово из четырех. 1. Дояр, эксперт, яслей, столяр. 2. Начать, торта (род. п., ед. ч.), пасквиль, щавеля. 3. Диалог, средствами, недуг, некролог. 4. Уведомить, углубить, понять, начать. 5. Цепочка, отрочество, памятуя, шарфы. 6. Донельзя, генезис, добыча, завидно.

2. Мягко или твердо произносятся в современном литературном языке согласные перед звуком [э] в следующих заимствованных словах? В каких случаях допускается вариантное произношение? Термин, термос, текст, фланель, энергия, эффект, бизнес, бизнесмен, демагог, музей, пресса, тенденция, термин, террор, форель, шедевр, юриспруденция, агрессия, академик, альтернатива, компьютер, рельс.

3. В какой из этих пяти фраз нет плеоназма (употребления лишних слов, ничего не добавляющих к смыслу высказывания)? Объясните, в чем состоит лексическая избыточность остальных предложений. 1. На заводе хромает трудовая дисциплина. 2. Наука — наш главный приоритет. 3. Цены постоянно растут вверх. 4. Халатность инкриминируется ему в вину. 5. Да простят меня за избитую банальность!

4. Определите средства выражения экспрессивно-эмоциональной окраски в следующих словах. Городишко, дождинка, лисонька, уста, мальчонка, мамочка, хвостист, голубушка, пылища, бедненький, книжка, говорливый, пальчик, деляга, тихоня, око, беззаветный, беспримерный, прах, скорбеть, премудрый, чело, ноженька, работенка, трусишка, зайка, соловушка, водичка, доиграться, забегаться, призадуматься.

5. Найдите и исправьте ошибки, нарушающие лексические нормы. 1. Самое лучшее для таких больных – это укол смертельной инъекции. 2. Первое боевое крещение дедушка принял на Западном фронте в составе первого гвардейского кавалерийского, который непосредственно участвовал в обороне Москвы и освобождении важных коммуникаций западнее столицы. 3. Новый дебютный альбом Юлии Савичевой «Высоко…». 4. Зарплата у служащих должна быть небедной. 5. Самая большая глобальная печаль – это то, что ни в одной коллекции не выдержана цветовая гамма. 6. Он изо всех сил старается произвести на Софью внимание. 7. Персональное собеседование с каждым сотрудником один на один.

6. Объясните и исправьте ошибки, связанные с нарушением морфологических норм. 1. Я сейчас картошки почистю. 2. Ну, вот, дом теперь убратый. 3. Сдавайте польта в гардероб. 4. Лук, обдатый кипятком… 5. Я, конечно, извиняюсь, но вы здесь не стояли, когда я пришла, так что за мной будете. 6. Ну, возьми тюль и постирай в машинке. Она такая грязная. 7. В последнее время я стал читать книги более внимательнее. 8. В вашем учебнике нет семиста восьмидесяти пятой страницы. 9. Боюсь, что я ощущу неловкость. 10. А почему нам документы не ворочают?

7. Укажите, какая из двух параллельных форм вам представляется правильной. Если оба варианта, по вашему мнению, употребительны в современном литературном языке, подчеркните и тот, и другой и укажите, есть ли различие (какое?) в их использовании. Базироваться на достоверных фактах – базироваться на достоверные факты; вульгарное арго – вульгарный арго; директоры – директора; единственный – единственен; Иван – большая умница или Иван – большой умница; мать с дочерью не могла уснуть – мать с дочерью не могли уснуть; не любить абстрактное искусство – не любить абстрактного искусства; новая туфля – новый туфель; повары – повара; по десяти франков – по десять франков; работы Базилюка Ивана – работы Базилюк Ивана; рецензия на книгу – рецензия о книге; самый ближайший – самый близкий; сироп приятен на вкус – сироп приятный на вкус; старая плащ-палатка – старый плащ-палатка; сто гектаров – сто гектар; торты – торта; у озера Балатон – у озера Балатона; четверо работниц – четыре работницы; робеть перед экзаменом – робеть экзамена; самый юный – наиболее юный; согласно приказа – согласно приказу; ряд сотрудников направлен – ряд сотрудников направлены; удалиться друзей – удалиться от друзей; у нее – у ней.

8. Найдите неверные способы образования грамматических форм и отредактируйте высказывания. 1. Она более моложе, чем все остальные. 2. Его местов я не занимаю и никаких делов у меня с ним нету. 3. Все были готовые к походу. 4. Морозец щипает щеки, метет поземка. 5. Путь, который избрал герой, ошибочен, неправильный. 6. На занятиях не было обоих сестер. 7. Сарай был крыт толью, а на окнах дома висела красивая тюль. 8. И оно никогда не уйдет, не стерется из нашей памяти. 9. Он уехал не позавтракавши.

9. Исправьте ошибки, связанные с использованием деепричастного оборота. 1. Раскопав ямку до дна, на солнце заблестели голубые камни. 2. Открыв окно, запахло соснами. 3. Подъезжая к городу, начался сильный ветер. 4. Закончив экскурсию, в ресторане нас ждал обед. 5. Проснувшись, ему сказали, что завтрак уже подан.

10. Укажите ошибки в управлении. Предложите возможные варианты правки. 1. Не обязательно снимать людей – домики, животные и цветы тоже можно. 2. Данный текст относится к официально-деловому стилю: он регламентирует о правах читателя. 3. Сегодня работали за компьютерами, а я с ними никогда не пользовалась. 4. Товарищи, оплатите за проезд. 5. Ты зачем ему дал этих книг?

11. Исправьте ошибки в построении предложений. 1. Поезд потерпел крушение благодаря небрежности стрелочника. 2. Роман «Отцы и дети» написаны Тургеневым в 1862 году. 3. За счет неполадок в работе мотора произошла автокатастрофа. 4. И. Эйгес полагал, что музыка вообще занимала большое место в творчестве Чехова и романсы в частности. 5. Согласно распоряжения мэра города, пострадавшим будет выплачена компенсация. 6. В основном их интересовало возможность торговать на иркутском рынке и лечение в больницах областного подчинения. 7. Все его родственники живут в Украине, а это по нашим временам уже другая страна. 8. Такого простора для ног мало кто предлагает из проектировщиков машин бизнес-класса. 9. Я сегодня не смогу: мы будем в концерте. 10. Вероятно, вы ее найдете возле памятника Ленина.

12. Ответьте на поставленные вопросы, используя фразеологические обороты (например, «Как говорят о том, кого нетрудно уговорить сделать что-либо, пойти куда-либо?» — «Лёгок на подъём»). Как говорят 1) о том, кто часто меняет свои решения; 2) о том, кто вдруг стал замечать, понимать что-либо; 3) о человеке, который пришел не вовремя, некстати; 4) о кротком, безобидном человеке; 5) о человеке, который держится неестественно прямо; 6) о болтливом человеке; 7) о бесследном исчезновении чего-либо; о положении, в котором опасность грозит со всех сторон; 9) об усердно трудящемся человеке.

13. Докажите с помощью примеров из приведенного ниже текста (автор — В. А. Барабанщиков) его принадлежность к научному стилю; определите, в рамках какого подстиля он составлен, и аргументируйте свою точку зрения. Выделите языковые средства, с помощью которых достигается логичность изложения, укажите на лексические единицы, грамматические формы и синтаксические конструкции, свойственные научной речи.

На первый взгляд, между С. Л. Рубинштейном и Б. Ф. Ломовым мало общего: они принадлежат к разным поколениям, разным научным школам, занимались разработкой разных проблем, играли в развитии отечественной психологии различную роль. Вместе с тем, более внимательный анализ обнаруживает между ними глубокую внутреннюю связь. Во-первых, Рубинштейн и Ломов были яркими представителями смежных микроэпох развития отечественной науки: если поколение Рубинштейна разрабатывало основание, или фундамент новой психологии, то поколение Ломова возводило нижние этажи самого здания. Во-вторых, существует известная близость школ С. Л. Рубинштейна и Б. Г. Ананьева, к которой принадлежал Б. Ф. Ломов, а значит сходство развиваемых идей и подходов. В-третьих, и Рубинштейн, и Ломов в разные исторические периоды стояли перед необходимостью методологически осмыслить современное им состояние психологии, отразить тенденции и наметить пути ее развития. В-четвертых. Рубинштейна и Ломова роднит решение одной и той же научно-организационной задачи — создание в Академии Наук сильного психологического исследовательского центра. Открыв в 1971 году Институт психологии, Ломов, по существу, завершил работу, начатую Рубинштейном в 40-е годы. В-пятых, и Рубинштейн, и Ломов придерживались коллегиального способа исследовательской деятельности, собирая воедино и опираясь на все полезное и ценное, что сделано в мировой и отечественной науке. Наконец, в-шестых, в чисто человеческом плане их объединяет редкая способность брать на себя полноту ответственности за судьбы вверенных им людей. Добавлю, что ни Рубинштейн, ни Ломов не были кабинетными учеными, оторванными от событий реальной жизни; они всегда занимали ясную гражданскую позицию, принимая самое активное участие в построении современной им психологической науки, образования и практики. Вполне закономерным представляется также тот факт, что ученики и соратники Сергея Леонидовича вошли в состав созданного Б. Ф. Ломовым Института психологии и, опираясь на глубоко эвристичную концепцию Рубинштейна, успешно разрабатывают вопросы теории и методологии психологической науки, проблемы психологии познания, личности, социальной психологии и многие другие.

Б. Ф. Ломов неплохо знал труды Сергея Леонидовича, особенно «Основы общей психологии» (1946) и «Принципы и пути развития психологии» (1959). Наряду с Б. Г. Ананьевым и П. К. Анохиным Рубинштейн входил в группу авторов, на которых Ломов (1984) ссылался наиболее часто и идеи которых пытался конструктивно использовать и развивать.

В данной работе я кратко рассмотрю содержание психологических концепций Рубинштейна и Ломова, сделав акцент на главных линиях их соприкосновения, тех, которые характеризуют преемственность научных традиций.

Прежде всего, это глубокий интерес к проблеме человека, который устойчиво воспроизводится в российской науке и общественно-политической мысли с середины XIX века. Очевидно, что в психологии то или иное понимание человека — «альфа» и «омега» любых конкретных исследований.

Данная проблема разрабатывалась Рубинштейном на философско-методологическом уровне в терминах соотношения человека как реального практического существа и его бытия, и имела не только мировоззренческий, но и нравственный смысл. Отношения человека к бытию полагались как объективные, а сам человек выступал в качестве субъекта разнообразных форм и проявлений жизни. Поскольку специфика человеческого бытия виделась в общественном способе существования, в качестве центрального выделялось отношение человека к другим людям. Тем самым открывалась возможность наполнения понятия «человек» гуманистическим содержанием.

Предложенная концепция задавала принципиально новые пути конкретно-психологического исследования (во многом пока еще не пройденные позитивной наукой), а также определяла способ интеграции наук об обществе, природе и мышлении. Отмечу, что с проблемой интеграции, или синтеза гуманитарных и естественных наук Рубинштейн столкнулся еще в студенческие годы, обучаясь философии в Марбурге (эта проблема была центральной и у его знаменитых учителей — Г. Когена и П. Наторпа), и перманентно возвращался к ней на протяжении всего творчества, В частности, методологический принцип единства сознания и деятельности, разработанный в 30-е годы, предназначался в том числе и для организации разнородного психологического знания.

Б. Ф. Ломов рассматривал проблему человека в ином ключе: через призму позитивных наук, нередко в связи с решением практических задач. Согласно Ломову, феномен человека выражает единство законов природы и общества и в этом качестве является уникальным объектом исследования. Выявление основных свойств и отношений человека, закономерностей его организации и развития рассматривались Борисом Федоровичем в качестве важнейшей задачи научного познания. И антропоцентрический подход к анализу систем «человек — техника», и понятие «активного оператора», и шаги по активизации так называемого «человеческого фактора» — все это разные формы выражения гуманистической направленности его исследований. Однако, двигаясь по этому пути, исследователь рано или поздно оказывался перед проблемой места психологии в системе наук о человеке и связанной с ней проблемой синтеза знаний.

14. Исправьте максимально возможное количество ошибок в приведенном ниже стихотворении. Поставьте ударение в словах, выделенных полужирным курсивом.

Хоть я и не русский очень, но хочу любви.

Положил документ в портфель – стырили.

Сажаюсь на тубаретку, отложив дела.

Почто ты мне звонишь редко, где была ты?

Не понял, объясни, я ведь подросток в любви,

Как мне начать с тобой. Ты углубила мне боль:

Ножницами наскрозь режешь сердце мне до слёз,

Вот уже много дня не схожу я с билютня.

Я мышленье напрягаю, звоню в телефон,

Ложу тебе письма в ящик: полон он.

Тебе подарю я арбуз, грейфруктов куплю,

Кружовника насажаю – я тебя люблю.

Прецендент я создал, за транваем побежал,

Посклизнулся в снегу, поломал себе ногу.

И теперь я в пальте на инвалидном месте,

С костылем и в гипсу еду в лесополосу.

Там живешь ты в дому, адрес я храню в уму:

Квартал нужно пройтить, библиотеку найтить.

Мне достал родный брат зарубёжный мирмалат –

Прямо тает в роту, открывай, я накладу.

В колидор выходи, раболант моей любви,

Не застойся в дверях, жгёт огонь меня в нутрях.

Ты придёшь, я скажу, что словов не нахожу:

Так могуч, так велик этот русский твой язык.

ПРИ ВЫПОЛНЕНИИ КОНТРОЛЬНОЙ РАБОТЫ, А ТАКЖЕ ПРИ ПОДГОТОВКЕ К ЭКЗАМЕНУ (С ЦЕЛЬЮ ЕГО НЕЗАТРУДНИТЕЛЬНОЙ СДАЧИ) ПОЛЬЗУЙТЕСЬ СЛЕДУЮЩЕЙ ЛИТЕРАТУРОЙ:

1. Аванесов литературное произношение. М., 1984.

2. , Зарва ударений русского языка. М., 1993.

3. Введенская и искусство речи. Ростов-на-Дону, 1996.

4. , , Кашаева язык и культура речи. Ростов-на-Дону, 2003.

5. Головин культуры речи. М., 1980.

6. Голуб русского языка. М, 2001.

7. , , Ягубова язык и культура речи: Учебник для студентов-нефилологов / Под редакцией д. ф. н., проф. О. Б. Сиротининой. Саратов, 2001.

8. Горбачевич современного русского литературного языка. М., 1978.

9. К, , Катлинская правильность русской речи. Стилистический словарь вариантов. М., 1976.

10. Культура русской речи / Под ред. проф. и проф. . М., 1998.

11. , Шведова словарь русского языка. М., 1995.

12. Орфоэпический словарь русского языка. М., 2001.

13. Плещенко и культура речи: Учеб. Пособие / Т. П. Плещенко, , ; Под ред. . ¾ Мн.: «ТетраСистемс», 2001.

14. Розенталь стилистика русского языка. М., 1987.

15. Русский язык и культура речи: Учебно-методический комплекс для вузов / Под ред. доц. . Кемерово, 2002.

16. Русский язык и культура речи / Под ред. проф. . М., 2002.

17. Русский язык и культура речи / Под ред. проф. . М., 2000.

18. Русский язык и культура речи: Конспект лекций / , , Е. В Ваганова и др. – Челябинск: Изд-во ЮУрГУ, 2003.

1. Расставьте ударение в приведенных словах и выберите одно лишнее (с акцентологической точки зрения) слово из четырех. 1. Некролог, неприязнь, мытарство, духовник. 2. Банты, хвоя, торты, столяр. 3. Христианин, танцовщик, простолюдин, статут. 4. Жалюзи, догмат, средства, апостроф. 5. Свекла, километр, баловать, шасси. 6. Нефтепровод, туфля, каталог, эксперт.

2. Мягко или твердо произносятся в современном литературном языке согласные перед звуком [э] в следующих заимствованных словах? Бактерия, безе, деликатес, декан, депрессия, детектив, интервью, кофе, кодекс, кратер, кредо, крейсер, критерий, модель, нетто, одесский, орхидея, патент, сессия, свитер, синтез, темп, тент, терапевт.

3. В какой из этих пяти фраз нет плеоназма (употребления лишних слов, ничего не добавляющих к смыслу высказывания)? Объясните, в чем состоит лексическая избыточность остальных предложений. 1. Развод – это трагическая катастрофа для всей семьи. 2. Все бюджетники получат дополнительную надбавку к зарплате в связи с праздниками. 3. Власти ведут борьбу с незаконным оборотом наркотиков. 4. Семёнов неожиданно наткнулся на непредвиденные трудности. 5. Его диссертация посвящена натурфилософии природы.

4. Распределите книжную лексику по стилистическим группам (по сферам употребления): 1) лексика официально-деловая; 2) научная, 3) техническая, 4) публицистическая. 5) поэтическая; назовите ситуации, где может уместно употребляться каждая группа. Истец, славный, вышеуказанный, концепт, завет, константа, пакт, коварство, ореол, нижеподписавшиеся, вещий, препроводить, лазер, виртуальный, аккумулятор, осужденный, локальный, интервью, барокамера, приговор, макроэкономика, шествовать.

5. Исправьте ошибки, связанные с употреблением паронимов. 1. В зале ожидания находилось много командировочных. 2. Преподаватель представил студентам полную свободу в выборе темы для сочинения. 3. Наши воины совершили много геройских подвигов. 4. Поэт стоял у источников новой поэзии. 5. Писатель завоевал мировую признательность. 6. Члены делегации зашли по трапу в самолет.

6. Объясните и исправьте ошибки в образовании грамматических форм слов. 1. Их нужно соединить, а потом посмотреть, как они сочетутся. 2. Я в этом году в Сочах была, там так красиво! 3. По тому, как живет цветочек, можно определить, как к тебе относятся. Если начнет вять … сама понимаешь. 4. Я хожу без носок, босиком. 5. Сколько же платий тебе нужно? 6. Сколько доль наследства ты получил? 7. Только почетные гражданины города у нас ездят бесплатно. 8. Я обожаю романтическую музыку, поэтому мои любимые песни — это песни Элтон Джона и Джордж Майкла. 9. В той квартире, где она сейчас живет, раньше цыгане жили. И она после этих цыганей нашла у себя под половицей их карты.

7. Укажите, какая из двух параллельных форм вам представляется правильной. Если оба варианта, по вашему мнению, употребительны в современном литературном языке, подчеркните и тот, и другой и укажите, есть ли различие (какое?) в их использовании. Весь в пушистом снеге – весь в пушистом снегу; выговоры – выговора; живописный Капри – живописное Капри – живописные Капри; жираф – жирафа; из полутора метров – из полтора метров; купи творога – купи творогу – купи творог; лип – липнул; нет время – нет времени; обеих девочек – обоих девочек; пачка табака – пачка табаку; полощет – полоскает; пять килограмм – пять килограммов; Петя – такая умница или Петя – такой умница; работники ЖЭК – работники ЖЭКа; руководить молодого человека – руководить молодым человеком; с Иваном Гумберт – с Иваном Гумбертом; старый мозоль – старая мозоль; уделить внимание на этот вопрос – уделить внимание этому вопросу; чернило – чернила; кресло-кровать стояло – кресло-кровать стояла; множество людей любовались – множество людей любовалось; не бойся – не бойсь – не боись; не терять надежды – не терять надежду; острая боль – острый боль; под Бородином – под Бородиным; старая плащ-палатка – старый плащ-палатка; старинное бра – старинный бра; тарелка борща – тарелка борщу; удивлен поступком – удивлен поступку; я с сыновьями уеду – я с сыновьями уедем.

8. Допишите окончания и объясните свой выбор. 1. В зоопарке живет маленьк. кенгуру. 2. Детеныш выглядывал из сумки больш. кенгуру. 3. Какаду отложил. яйца и высиживал. птенцов. 4. В соседней клетке сидел. какаду. 5. Шуми. многолюдн. Осло. На главн. авеню – уютн. кафе «Розов. фламинго». В разнообразн. меню есть бутерброды с аппетитн. салями, салат из свеж. кольраби, черн. кофе, абрикосов. суфле, крепк. виски. 6. Крупнейш. цунами обрушил. сь на берег. 7. Эсперанто был… создан. доктором Л. Заменгофом.

9. Укажите ошибки в управлении. Предложите возможные варианты правки. 1. Начался обмен мнений. 2. Мы восторгались красоте летнего заката. 3. Я очень рада решением суда: все получилось так, как я хотела. 4. Дорогие друзья, Святославу Ещенко сегодня день рождения! 5. Он всегда удивлялся трудолюбием сына.

10. Исправьте ошибки, связанные с использованием деепричастного оборота. 1. Родившись в такой кинематографической семье, Наташино будущее было, в общем-то, предопределено. 2. Посмотрев этот фильм, мне стало очень грустно. 3. Придя домой, радио играло на всю громкость. 4. Выйдя из квартиры, у него появились слезы. 5. Приехав домой, мне стало значительно лучше.

11. Исправьте ошибки в построении предложений. 1. Полиция нередко безучастно созерцала на разгорающийся конфликт. 2. Вследствие предстоящей защиты я пятую ночь не сплю. 3. Крестьянство выступали против помещиков. 4. Прошу предоставить отпуск ввиду болезни. 5. Это же молодое поколение, которое растут. 6. Во многих странах мира действует правило, согласно которого все более-менее значительные объемы внешних заимствований отдельных рыночных субъектов данной страны гарантируются в той или иной степени государством. 7. Нашла у кого советоваться. 8. Благодаря его мы все успели вовремя. 9. Сегодня на меню картошка с котлетами. 10. Ты во сколько сегодня пришёл со школы?

12. Ответьте на поставленные вопросы, используя фразеологические обороты (например, «Как говорят о том, кого нетрудно уговорить сделать что-либо, пойти куда-либо?» — «Лёгок на подъём»). Как говорят 1) о чем-нибудь надоевшем, много раз слышанном; 2) об очень дальних родственниках; 3) о человеке, от которого ожидают успеха в какой-либо деятельности 4) о неразговорчивом человеке; 5) о том, кто никак не может прекратить смеяться; 6) о беспорядке, неразберихе, царящих где-либо; 7) о чем-нибудь необычайном, невероятном; о человеке, имеющем большой опыт в чем-либо; 9) о предмете всеобщих разговоров, пересудов, сплетен.

13. Докажите с помощью примеров из приведенного ниже текста (автор — Л. В. Ухова) его принадлежность к научному стилю; определите, в рамках какого подстиля он составлен, и аргументируйте свою точку зрения. Выделите языковые средства, с помощью которых достигается логичность изложения, укажите на лексические единицы, грамматические формы и синтаксические конструкции, свойственные научной речи.

Особенностью публичной речи как разновидности речевого общения является также и то, что это вид прямой коммуникации, контролируемый обоими участниками общения. Это позволяет обеим сторонам корректировать характер взаимодействия, влиять на его результаты [Шмелева 1985], что обусловливает такую специфическую черту публичной речи, как спонтанность, которая, однако, носит ограниченный характер и прежде всего потому, что текст публичного выступления, имея устную форму презентации, как правило, готовится заранее и чаще фиксируется в письменном виде.

Диалог, будучи одной из форм существования языка, является едва ли не важнейшей областью проявления языковых закономерностей. Отличие диалога от других сфер функционирования языка заключается прежде всего в сложной картине взаимодействия интенций коммуникантов. Нормальный ход диалога предполагает согласование иллокутивных намерений (коммуникативных целей) участников диалога, которое заключается в удовлетворении их взаимных претензий.

Участники диалога вынуждены выполнять разнообразные речевые и неречевые действия, заставляя партнера реагировать на них определенным образом. Каждое такое речевое и неречевое вынуждение получает ответную реакцию со стороны другого участника (ср. у Бахтина представление о смене речевых субъектов и о выполнении взаимных обязательств участников как об основной особенности диалога в сравнении с монологом [Бахтин 1979: 251, 255]).

Говоря о взаимодействии участников общения в рамках диалога, А. Н. Баранов, Г. Е. Крейдлин вводят понятие иллокутивного вынуждения как «одного из проявлений законов сцепления, действующих на пространстве диалога» [Баранов, Крейдлин 1992, № 2]. Речевые акты, связанные в речевом контексте отношением иллокутивного вынуждения, авторы предлагают называть соответственно иллокутивно независимым и иллокутивно зависимым. Иллокутивно независимый речевой акт они определяют как речевой акт, иллокутивное назначение которого на данном шаге определяется интенциями самого говорящего, а иллокутивно зависимый речевой акт как речевой акт, иллокутивное назначение котого всецело определяется иллокутивным назначением какой-либо предшествующей реплики (из данного речевого отрезка), соответственно распределяя иллокутивно независимые и иллокутивно зависимые реплики. Структура диалога, по мнению авторов, опирается на отношение иллокутивного вынуждения, подобно тому как структура предложения формируется на основе синтаксических связей. «Между тем иллокутивное вынуждение не тождественно синтаксической связи. Если такая связь, как синтаксическая зависимость, основывается исключительно на категориальных свойствах языковых единиц, то вынуждение, действуя на пространстве речевых актов, формируется не только под влиянием иллокутивной функции речевых высказываний, но и находится под воздействием общих законов функционирования диалога» [Баранов, Крейдлин 1992: 88]. (К последним, в частности, принадлежат социально обусловленные законы-максимы Грайса и принцип вежливости Линча.)

Исследование закономерностей речевого взаимодействия, выявление путей оптимизации такого взаимодействия – задачи, которые на сегодняшний день лежат в области интересов различных направлений антрополингвистики [Седов 2000], и в частности, — лингвистической конфликтологии. О природе конфликтов много и достаточно интересно писали психологи [Берн, 1997; Шейнов, 1999]. В работах зарубежных и отечественных исследователей показаны причины конфликтов, даны рекомендации по их предотвращению и выходу из конфликтной ситуации. Языковеды делают лишь первые шаги в освоении этого объекта изучения [Горелов, Седов 1998; Енина 1999; Жельвис 1997; Седов 1996; Стернин 1995; Ширяев 2000].

Определение механизмов диалогического взаимодействия – задача далеко не решенная в современном языкознании, хотя область эта интенсивно изучается. О. С. Иссерс отмечает [Иссерс, 1999], что некоторые объяснения этих процессов могут быть получены через описание стратегических направлений и тактических приемов, реализуемых по ходу диалога. По мнению автора, стратегия речевого поведения охватывает всю сферу построения процесса коммуникации, когда ставится целью достижение определенных долговременных результатов. В общем смысле «речевая стратегия включает в себя планирование процесса речевой коммуникации в зависимости от конкретных условий общения и личностей коммуникантов, а также реализацию этого плана [Иссерс 1999: 54].

14. Исправьте максимально возможное количество ошибок в приведенном ниже стихотворении. Поставьте ударение в словах, выделенных полужирным курсивом.

Хоть я и не русский очень, но хочу любви.

Положил документ в портфель – стырили.

Сажаюсь на тубаретку, отложив дела.

Почто ты мне звонишь редко, где была ты?

Не понял, объясни, я ведь подросток в любви,

Как мне начать с тобой. Ты углубила мне боль:

Ножницами наскрозь режешь сердце мне до слёз,

Вот уже много дня не схожу я с билютня.

Я мышленье напрягаю, звоню в телефон,

Ложу тебе письма в ящик: полон он.

Тебе подарю я арбуз, грейфруктов куплю,

Кружовника насажаю – я тебя люблю.

Прецендент я создал, за транваем побежал,

Посклизнулся в снегу, поломал себе ногу.

И теперь я в пальте на инвалидном месте,

С костылем и в гипсу еду в лесополосу.

Там живешь ты в дому, адрес я храню в уму:

Квартал нужно пройтить, библиотеку найтить.

Мне достал родный брат зарубёжный мирмалат –

Прямо тает в роту, открывай, я накладу.

В колидор выходи, раболант моей любви,

Не застойся в дверях, жгёт огонь меня в нутрях.

Ты придёшь, я скажу, что словов не нахожу:

Так могуч, так велик этот русский твой язык.

ПРИ ВЫПОЛНЕНИИ КОНТРОЛЬНОЙ РАБОТЫ, А ТАКЖЕ ПРИ ПОДГОТОВКЕ К ЭКЗАМЕНУ (С ЦЕЛЬЮ ЕГО НЕЗАТРУДНИТЕЛЬНОЙ СДАЧИ) ПОЛЬЗУЙТЕСЬ СЛЕДУЮЩЕЙ ЛИТЕРАТУРОЙ:

1. Аванесов литературное произношение. М., 1984.

2. , Зарва ударений русского языка. М., 1993.

3. Введенская и искусство речи. Ростов-на-Дону, 1996.

4. , , Кашаева язык и культура речи. Ростов-на-Дону, 2003.

5. Головин культуры речи. М., 1980.

6. Голуб русского языка. М, 2001.

7. , , Ягубова язык и культура речи: Учебник для студентов-нефилологов / Под редакцией д. ф. н., проф. О. Б. Сиротининой. Саратов, 2001.

8. Горбачевич современного русского литературного языка. М., 1978.

9. К, , Катлинская правильность русской речи. Стилистический словарь вариантов. М., 1976.

10.Культура русской речи / Под ред. проф. и проф. . М., 1998.

11., Шведова словарь русского языка. М., 1995.

12.Орфоэпический словарь русского языка. М., 2001.

13.Плещенко и культура речи: Учеб. Пособие / Т. П. Плещенко, , ; Под ред. . ¾ Мн.: «ТетраСистемс», 2001.

14.Розенталь стилистика русского языка. М., 1987.

15.Русский язык и культура речи: Учебно-методический комплекс для вузов / Под ред. доц. . Кемерово, 2002.

16.Русский язык и культура речи / Под ред. проф. . М., 2002.

17.Русский язык и культура речи / Под ред. проф. . М., 2000.

18.Русский язык и культура речи: Конспект лекций / , , Е. В Ваганова и др. – Челябинск: Изд-во ЮУрГУ, 2003.

источники:

http://hr-portal.ru/article/kak-maksimalno-effektivno-i-pravilno-provesti-sobesedovanie

http://pandia.org/text/78/040/630.php

ПРИГОВОР

именем Российской Федерации

Крапивинский районный суд

Кемеровской области

в составе:

председательствующего: Воробьевой В.И.

заседателей Степановой, Гавриленковой

при секретаре Набритовой В.В.

с участием прокурора Тиунова В.Г.

и адвокатов Котенкова, Козлова

рассмотрев в открытом судебном заседании в п. Крапивинский

16.12.99года

дело по обвинению КРАЙНОВА ВЛАДИМИРА ЮРЬЕВИЧА,

родившегося 6 июля 1979 г. в с. Ястребовка Марк-

совского района Саратовской обл., немца, с обра-

зованием неполным средним, холостого, судимого:

25.06.96 г. по ст. 144 ч.3 УК РФ к 2 г. л/св.,

1.04.97г. по ст. 145 ч.1, 40 ч.3 УК РФ к 2 г л/св., ос-

вобожден 17.01.98г., прож. П. Крапивинский ул.

Гагарина, 10.

МИНИНА ВАДИМА АНАТОЛЬЕВИЧА, родивше-

гося 16 мая 1973 г. в п. Крапивинский Кемеровской

обл., русского, с образованием 8 кл., холостого, не

работающего, проживающего п. Крапивинский ул.

Совхозная, д. 42 кв.2, в совершении преступления

Крайновым по ст. 162 ч.3 п. Г УК РФ, Мининым

По ст. 162 ч.2 п. А,Б,В,Г УК РФ, суд

У С Т А Н О В И Л:

В ночь на 11.03.99г. предварительно договорившись между собой подсудимые Минин и Крайнов с целью кражи компьютера подошли к зданию Крапивинского межлесхоза, предварительно взяв с собой из дома Минина выдергу, а из дома Крайнова мешок, скотч, складной нож, расположенного в п. Крапивинский по ул. Пятаковича, 19.

Зная, что в здании находятся сторож, имея умысел на разбойное нападение вошли в тамбур конторы. Дежуривший в эту ночь Первунин, услышав шум вышел из здания.

Минин выдергой нанес удар Первунину по голове и руке. А когда Первунин упал, Крайнов, демонстрируя принесенным с собой ножом, высказывая в адрес потерпевшего угрозы, связал последнему скотчем руки и вдвоем завели Первунина в здание конторы, гдя, имея умысел на разбойное нападение с целью завладения компьютером, привязали потерпевшего к скамейке, после чего, взломав запоры на двух дверях кабинета бухгалтерии, проникли во внутрь кабинета, откуда похотили компьютер стоимостью 10300 р., принтер за 2200 р. и модем за 975 р. Все похищенное сложили в принесенный с собой мешок и в штору с окна кабинета. Оставив привязанным к скамье сторожа Первунина, закрыли двери конторы на замок. Похищенное унесли в дом Федечкина по ул. Совхозной, д.49 и спрятали в подполье, в последствии увезли в г. Кемерово и продали неустановленным лицам.

Своими действиями Минин и Крайнов причинили потерпевшему согласно заключения СМЭ, закрытый перелом средней трети правой локтевой части, повлекший вред здоровью средней тяжести по признаку длительности расстройства здоровья, связанного с временной утратой общей трудоспособности продолжительностью свыше 21 дня и ушибленную рану правой височно-теменной области, которая повлекла легкий вред здоровью по признаку кратковременного расстройства здоровья, связанного с утратой общей трудоспособности, продолжительностью не свыше 21 дня. И материальный ущерб межлесхозу в сумме 13475 р., который не возмещен.

Допрошенные в качестве подсудимых Минин и Крайнов виновными себя не признали и пояснили, что в ночь на 11.03.99г. разбойное нападение на сторожа Крапивинского межлесхоза Первунина не совершали, кражу компьютера из конторы также не совершали.

Однако, вина подсудимых подтверждается показаниями потерпевшего, свидетелей.

Так, потерпевший Первунин пояснил суду, что в ночь на 11.03.99г. во время его дежурства в конторе Крапивинского межлесхоза было совершено разбойное нападение на него. Около часа ночи он на улице услышал шум, вышел в тамбур, за дверьюстояли двое в масках. Один из них ударил его монтажкой по голове 2 раза и по руке – 2 раза. Он упал. Ударил, который был повыше. А тот, что был пониже ростом достал нож и высказал угрозы, демонстрируя ножом перед ним, велел молчать. Он очень испугался, угрозу воспринял как реальную. Заведя его в контору, подсудимые скотчем заклеили ему рот и связали руки, привязали к скамейке капроновыми нитками. После чего спросили, где находится компьютер. Он указал на кабинет бухгалтерии. Он слышал как взламывали дверь. С собой у них был мешок. Он видел как они выносили мешок.

Гражданский истец Мустаев пояснил в суде, что ночью 11.03.99г. ему по телефону Первунин сообщил, что неизвестные лица совершили на него разбойное нападение, похитили компьютер. В конторе он обнаружил, что в кабинете бухгалтерии взломаны запоры на 2х дверях и похищены компьютер, принтер и модем общей стоимостью 13475р. С окна бухгалтерии сорвана штора. Сумму иска просит взыскать с подсудимых.

Свидетель Федечкин в судебном заседании от показаний, данных им на предварительном следствии отказался, мотивируя тем, что Минин и Крайнов у него не были и компьютер к нему не приносили.

Однако, доводы защиты к подсудимым о том, что в ночь на 11.03.99г. они не были в конторе Крапивинского межлесхоза и кражу компьютера не совершали, опровергаются показаниями подсудимых на предварительном следствии, показаниями потерпевшего, свидетелей.

В материалах дела имеются явки с повинной на л.д. 63 – Минина и на л.д. 69 – Крайнова, которые они писали собственноручно и в которых они рассказали, что в начале марта 1999г. Минину Ерофеенко предложил совершить кражу компьютера из конторы межлесхоза, Минин согласился и рассказал Крайнову «Греку». Кражу они решили совершить вдвоем без Ерофеенко. Придя к конторе, они увидели, что там сторож. Когда он вышел на улицу, Минин ударил его выдергой по голове и он упал. Вдвоем они его завели в контору, связали. Взломали запоры на дверях бухгалтерии. Компьютер положили в мешок и унесли к Федечкину.

На следующий день компьютер забрали у Федечкина и увезли в г.Кемерово, где продали за три тысячи рублей.

На л.д. 64 Минин пояснил, что раньше он давал ложные показания, т.к. не хотел признаваться, а сейчас он решил чистосердечно все рассказать и далее он даст показания аналогичные его показаниям в явке с повинной.

Аналогичны его показания на л.д. 79-80, 81-86.

Показания Минина и Крайнова последовательны, правдивы. В дальнйшем в суде, подсудимые изменили свои показания с целью уйти от ответственности.

На л.д. 81 Федечкин пояснил, что в середине марта 1999 г. он несколько дней пил Однажды ночью к нему пришли его племянник Мининс парнем по кличке «Крек». Они что-то принесли с собой и спрятали в подполье. 31.03.99г. Минин ему сказал, что приносил компьютер.

В судебном заседании Федечкин изменил свои показания с целью скрыть преступление, совершенное его родственником – племянником Мининым.

Суд считает, что его показания на предварительном следствии правдивы.

Доводы подсудимых и свидетеля Федечкина, что на предварительном следствии к ним применялось насилие необоснованны и опровергаются показаниями следователя Саловой, которая пояснила всуде, что она лично допрашивала подсудимых Минина и Крайнова и свидетеля Федечкина и никакого насилия к ним не применялось, показания они давали добровольно.

Свидетель Коробейников Д.М. пояснил в суде, что в ночь на 11.08.99г. они с братом ехали на лошади, запряженной в сани на берегу р.Томь и увидали как в снегу лежат 2 парней. В одном из них он узнал Ерофеенко, возможно, он ошибся, второго он не знал. С ними был мешок с чем-то.

Его брат Коробейников М.М. на л.д.16 пояснил, что в ночь на 11.03.99г. ехали на лошади, запряженной в сани. В стороне от дороги в снегу лежали 2 парней, один из них Ерофеенко, второй скорее всего Минин. С ними был мешок. К показаниям свидетеля Буслаевой Е. суд относится критически, поскольку она является сожительницей Минина и заинтересована в исходе дела. От Карис он ушел в 23 часа.

Кроме того, вина подсудимых подтверждается заявлением Мустаева л.д.4, осмотром места происшествия л.д.69, исковым заявлением л.д.19, заключением СМЭ л.д.97-98 и другими материалами дела.

Действия подсудимого Минина ст.162 ч.2 п.А,Б,В,Г УК РФ классифицированны правильно, поскольку он совершил разбой, т.е. нападение в целях хищения чужого имущества, совершенное с применением насилия, опасного для жизни и здоровья, группой лиц, по предварительному сговору, неоднократно, с незаконным проникновением в помещение с применением предметов, используемых в качестве оружия.

Органами предварительного следствия Крайнов обвиняется в совершении преступления, предусмотренного ст. 162 ч.3 п. Г УК РФ, т.е. в совершении разбоя лицом, ранее судимым два раза за хищение.

Однако, данное обвинение не нашло подтверждения в суде, поскольку Крайнов осужден был 25.06.96г. к 2 годам л/св., 1.04.97г. по ст.145 ч.1 УК РФ с применением ст.40 ч.3 УК РФ по совокупности преступлений по приговору от 25.06.96г. к окончательной мере наказания в виде двух лет лишения свободы.

Поэтому его действия необходимо квалифицировать ст.162 ч.2 п.А,Б,В,Г УК РФ, как совершение преступления группой лиц по предварительному сговору, неоднократно, с незаконным проникновением в помещение с применением предметов, используемых в качестве оружия.

Вместе с тем, учитывая их положительные характеристики, судимость у Минина погашена, в настоящее время они имеют постоянное место жительства, находит возможным их исправление без изоляции от общества в соответствии со ст.УК РФ, назначив уголовное наказание.

Поскольку имущество, подлежащего описи у подсудимых нет, дополнительное наказание – конфискацию имущества в отношении подсудимых не применять.

Гражданский иск, заявленный гражданским истцом подлежит удовлетворению.

Руководствуясь ст.301-303 УПК РФ, суд

П Р И Г О В О Р И Л:

КРАЙНОВА ВЛАДИМИРА ЮРЬЕВИЧА и МИНИНА ВАДИМА АНАТОЛЬЕВИЧА признать виновными по ст.162 ч.2 п. А,Б,В,Г УК РФ и назначить им наказание по данной статье в виде семи лет лишения свободы каждому.

В соответствии со ст.73 УК РФ наказание считать условным каждому, с испытательным сроком в два года, каждому.

Меру пресечения до вступления приговора в законную силу изменить на подписку о невыезде, освободив из-под стражи в зале суда.

Зачесть время содержания под стражей: Крайнов с 31.03.99г. по 16.12.99г., Минину с

Обязать Крайнова В.Ю. И Минина Вадима Анатольевича в пользу Крапивинского межлесхоза периодически являться в органы внутренних дел на регистрацию.

Приговор может обжалован в Кемеровский областной суд через Крапивинский районный суд в течение семи суток.

Судья: Воробьева В.И.

Заседатели:

Копия верна.

Пост. 22.12.99года

Исп. Мусияченко Т.Г.

22.12.1999 года

Контрольная работа

Задание 1. Изложить содержание теоретического вопроса.

Примечания. 1. Материал задания составляют вопросы, рекомендованные для изучения на семинарских занятиях (см. разделы Содержание практических занятий и Контрольные вопросы рабочих программ курса). 2. Реферативное исследование той или иной проблемы необходимо строить с опорой на библиографические указатели литературы к рабочим программам и учебному пособию.

1. Стилевые черты и стилистические нормы научного текста*.

2. Общая характеристика официально-делового стиля.

3. Публицистический стиль: стилевые черты и их экспликация в языковых средствах.

4. Средства экспрессивности публицистического текста.

5. Разговорный стиль и разговорная речь. Коммуникативные и системно-структурные особенности разговорной речи.

6. Язык художественной литературы в системе функциональных стилей русского языка.

7. Основные черты русского литературного произношения. Лингвогеографические явления в орфоэпии. Динамика норм и стилей произношения.

8. Своеобразие норм ударения. Причины колебания норм ударения. Источники нарушения акцентологических норм.

9. Фонетическая стилистика. Стилистические приемы усиления звуковой организации речи.

10. Своеобразие лексических норм. Понятие смысловой точности речи.

11. Стилистическая дифференциация лексики русского языка.

12. Стилистическое использование в речи лексики ограниченной сферы распространения.

13. Стилистическое использование в речи лексики пассивного состава языка.

14. Стилистическое использование в речи синонимов, антонимов, полисемантов, омонимов, паронимов.

15. Стилистические ресурсы фразеологии.

16. Лексические образные средства.

17. Речевые и стилистические ошибки в сфере лексики и фразеологии.

18. Словообразовательные нормы и стилистика. Вопрос о стилистически не оправданном использовании единиц словообразования.

19. Понятие морфологической нормы. Причины повышенной вариантности морфологических норм.

20. Вариантные и синонимичные формы различных частей речи: стилистическая окрашенность, условия выбора и использования.

21. Речевые ошибки и стилистические ошибки в сфере морфологии.

22. Стилистические ресурсы простого предложения. Типичные ошибки в строе простого предложения.

23. Стилистика сложного предложения.

24. Речевые и стилистические ошибки в сфере синтаксиса.

25. Синтаксические средства экспрессивной речи.

Задание 2. Представить стилистический анализ текста*, принадлежащего указанному речевому жанру. Доказать стилевую определенность текста.

Примечание. Примерный план стилистического анализа текста. 1. Коммуникативно-прагматические признаки текста (стилеобразующие признаки (факторы) – социально-значимая сфера коммуникации, цель, содержание, адресованность текста, форма, вид речи, характер и тональность общения – и стилевые черты текста). 2. Речевые признаки текста (речевой строй текста – особенности речевой нормативности на лексическом, морфологическом, синтаксическом уровнях). 4. Центральные и периферийные (несвойственные данному стилю) речевые признаки (причины их использования в анализируемом тексте). 5. Общий вывод о статусе текста в коммуникативно-прагматической и системно-структурной иерархии функциональных стилей (о функционально-стилевой, подстилевой и жанрово-ситуативнойпринадлежности текста.)

1. Стилизация разговорной речи в художественном тексте. 2. Научная статья по языкознанию. 3. Автобиография. 4. Рецензия на кинофильм. 5. Исковое заявление. 6. Рекламное объявление (любой рубрики). 7. Аннотация на книгу по литературоведению. 8. Школьный учебник. 9. Запись разговорной речи (любого жанра). 10. Рекламная статья . 11. Судебная речь (любого жанра). 12. Интервью. 13. Лекционная речь. 14. Деловое (служебное) письмо. 15. Научно-популярная статья по лингвистике. 16. Естественная письменная речь (письмо, записка, поздравление). 17. Социально-политическая речь (любого жанра). 18. Телефонный разговор. 19. Докладная записка. 20. Очерк. 21. Школьное сочинение. 22. Рецензия на статью, монографию по литературоведению. 23. Духовная проповедь. 24. Фельетон. 25. Дискутивно-полемическая речь.

Задание 3. На материале стилистическогоанализа художественного текста* (любой формы и жанра) докажите тезис об «открытости» и особых речевых функциях языка художественной литературы. Охарактеризуйте тропы и фигуры речи, используемые в пределах анализируемого текста.

Задание 4. Дать нормативно-стилистическую оценку фонетических, лексических, морфологических, синтаксических вариантов: определить нормативность / ненормативность, равноправность / неравноправность колебания языковых норм.

Примечание. При выполнении задания используйте схему 1.

Уровни
языка
Равноправные
варианты
Неравноправные
варианты
Ненормативные варианты
Полные Частичные Стилисти
ческие
Семантические Хронологические
             

Схема 1

1. Истекший – истёкший, оптовый – оптовый, опасный – опасливый, контральто (м.р.) – контральто (ср.р.), гулять вечерами – гулять по вечерам.

2. Танц[ые]вать – танц[а]вать, видение –видение, архаический – архаичный, ужасная забияка – ужасный – забияка, документ с подписью – документ за подписью.

3. Безнадежный – безнадёжный, алкоголь – алкоголь, длинный – длительный, тарелка супа – тарелка супу, слыть весельчаком – слыть за весельчака.

4. [Ж`ур`и] – [жур`и], свёкла – свекла, беспокойство – волнение – тревога – смятение, купить масло – купить масла, много учеников посетило – много учеников посетили.

5. Ж[ые]леть – ж[а]леть, шарфов – шарфов, высота – вышина, семь мальчиков – семеро мальчиков, большинсто учеников отсутствовало – большинство учеников отсутствовали.

6. Маневры – манёвры, по средам – по средам, возрождать – восстанавливать, лес величествен – лес величественен, два новых дома – два новые дома.

7. Оглашенный – оглашённый, волнам – волнам, теперь – сейчас – ныне – нынче – иначе, мок – мокнул, промокший – промокнувший, четыре новых автомобиля – четыре новые автомобиля.

8. Оседлый – осёдлый, развитый – развитый, ошибка – просчет – накладка – оплошка – ляп, машет – махает, три маминых книги – три мамины книги.

9. Ака[д`]емия – ака[д]емия, ржаветь – ржаветь, тяжелый – трудный, коленей – коленьев – колен, ждать трамвая – ждать трамвай.

10. Жаждущий – жаждующий, шасси – шасси, недотрога – мимоза – недотыка, из дома – из дому, на доме – на дому, знать историю – знать об истории.

11. То[чн]о – то[шн]о, коротка – коротка, жесткий – жестокий, вымпелы – вымпела, по несколько человек – по нескольку человек.

12. Ску[чн]о – ску[шн]о, углубить – углубить, вылечить – излечить – исцелить – поднять (на ноги), крепкого чая – крепкого чаю, по пять книг – по пяти книг.

13.
Моло[чн]ая – моло[шн]ая, отзыв – отзыв, следовательно – итак – таким образом – стало быть – стало – значит – выходит – следственно, встретить спутника – встретить спутник, по предъявлении документа – по предъявлению документа.

14. Горчи[чн]ик – горчи[шн]ик, феномен – феномен, особа – персона – лицо – фигура – птица, купить хлеба – купить хлеб, скучать о друге – скучать по другу.

15. Ла[з`]ер – ла[з]ер, шёлковый – шелковый, очень – весьма – сильно – здорово – больно – шибко, апельсинов – апельсин, оплачивать за проезд – оплачивать проезд.

16. Эксп[р]ессивный – эксп[р`]ессивный, пломбировать – пломбировать, принять – предпринять, девочка красивая – девочка красива, по приезду – по приезде.

17. А[сф]альт – а[св]альт, на дом – на дом, долгий – продолжительный – длинный – длительный – долговременный, низовий – низовьев, несколько человек явились – несколько человек явилось.

18. [Т`]емп – [т]емп, творог – творог, натуральный – естественный, баржей – барж, по нему – по нем.

19. [Д`]екан – [д]екан, за стенами – за стенами, удачный – успешный – счастливый – благополучный, дверями – дверьми, лекция на тему – лекция по теме.

20. Серде[чн]ый – серде[шн]ый, торгов – торгов, единый – один, уведоми – уведомь, характеристика на кого – характеристика о ком – характеристика кого.

21. Желчный – жёлчный, нормировать – нормировать, индифферентный – равнодушный, вверг – ввергнул, на расстоянии десяти метров – на расстоянии в десять метров – на расстоянии десять метров.

22. Щ[а]вель – щ[ие]вель, петля – петля, очаровательный – прелестный – пленительный – чарующий, соболи – соболя, не оказывает влияние – не оказывает влияния.

23. Афера – афёра, квартал – квартал, драматический – драматичный, два доцента – двое доцентов, приемущество над кем-либо – преимущество перед кем-либо.

24. До[жд`] – до[ж`ж`] – до[шт`] – до[ш`ш`], генезис – генезис, красный – кумачовый – карминный – кинаровый – рубиновый – рубинный – червонный – шарлаховый, новый толь – новая толь, отмечается о главном – отмечается главное.

25. Инцидент – инциндент, форзац – форзац, истинный – настоящий – подлинный – заправский, кондукторы – кондуктора, большинство проголосовали – большинство проголосовало.

Задание 5. Отредактируйте предложения. Установите причины, характер и тип речевых ошибок.

Примечания. 1. При выполнении задания используйте схему 2. 2. О классификации ошибок см.: Кожина М.Н. Стилистика русского языка. М., 1993. С. 96-104; РЯШ. 1980. № 3. С. 122-123; Капинос В.И. Об оценке речевых навыков учащихся // РЯШ. 1973. № 6; Чижова Т.И. Основы методики обучения стилистике в средней школе. М., 1987.

Неисправленный
вариант
Причина
ошибки
Характер и тип ошибки
(по классификации М.Н. Кожиной)
Характер и тип ошибки (в школьном курсе русского языка)
 
Исправленный
вариант

Схема 2

1. 1. Евгений Онегин получил уведомление о том, что его дядя заболел. 2. Мы должны бороться всеми нашими ресурсами. 3. Плохо, когда человек не воспринимает критику. 4. Большая часть населения находится за чертой уровня бедности. 5. Став более взрослым, Онегин принимает участие на балах. 6. Мы не всегда отдаем отчет тому, как важны праздники в нашей жизни. 7. Глядя на то, что происходит сегодня у меня руки развязаны. 8. Об этом мы познакомим вас позже. 9. Софья любила Молчалина, но не высказала мнения по этому вопросу. 10. Автор не согласился с оценкой критики своего романа.

2. 1. У нее были очень огромные глаза. 2. Встретившись с Хлебниковым в душе Ромашова наступил перелом. 3. У руля руководства стояла мать Татьяны Лариной. Отец был смещен ею с поста главы дома. 4. Согласно приказа декана заведующая кафедры уехала в командировку. 5. Сосед по парте спросил, кто пойдет со мной в кино. 6. Большинство стихотворений Цветаевой пронизаны лирикой и красотой написания. 7. Систематическое образование Татьяна Ларина не получила: родители не уделяли этому должного внимания. 8. И как жесткий укор на католической иконе в доме «странного человека» остается висеть браслет, который героиня вернула ему со своим мужем. 9. Любой человек бывает малость грубоват, хотя должен следить за собой. 10. Татьяне чужды интересы высшего света, поэтому она не принимает участия в его мероприятиях.

3. 1. Прекрасный славянский обычай – встречать гостей, поднося им на скатерти буханку душистого хлеба и соль. 2. Большинство этих художников било мастерами цвета. 3. У нас ведется большая работа по улучшению качества производимой продукции. 4. Федоров был мужественен и решителен. 5. Перед нами сборник коротких, но правдивых новелл. 6. Всю ночь верующие держали почетную вахту при гробе господнем. 7. Большое значение играет укрепление производственной дисциплины, необходимо широко пропагандировать безопасные методы труда. 8. Я обратно повторил свой вопрос. 9. Мы выполняем не только вашу заявку, но и предлагаем новую песню композитора. 10. Хочется сказать огромное спасибо персоналу больницы, на чью голову свалилась такая тяжелая больная.

4. 1. Чтобы создать новый сорт пшеницы, нужно ого-го как много и долго трудится. К тому же в этой работе замешано большое количество людей и профессий. 2. Дряхлая избушка стоит на берегу реки. 3. Здесь справил новоселье только что созданный музей. 4. Для языка Дикого, как и для других представителей «темного царства», характерны грубые выражения. 5. Из-за засухи урожай был плохим. 6. Проверка будет произведена назначенной администрацией комиссией. 7. Новый прораб ходит за бригадиром и выясняет, что и как называется и куда присобачивается. 8. В магазине не оказалось ни чулков, ни носок. 9. Галина Уланова – одна из крупнейших представителей русской балетной школы. 10. Память и знание прошлого наполняют мир, делают его интересным, значительным, одухотворенным.

5. 1. Большинство учителей и методистов объединило свои усилия в решении важной проблемы. 2. Уж, кажется, на что Татьяна любила Онегина, а нашла в себе силы достойно отказать ему. 3. О том, как прошел матч, можно посмотреть по первой программе телевидения. 4. Перстневой нож одевают на вторую фалангу среднего пальца руки вязальщицы. 5. Рабочие места слабо освещались, в силу чего имелось много случаев производственного травматизма. 6. У всего этого фамусовского сброда на уме одно: балы да веселья. 7. Несколько ниже мы поговорим об этом. 8. Важным фактором успешной сдачи экзаменов являеться систематическое выполнение домашних заданий. 9. Главное, на чем настаивали рабочие,- это на требовании повышения зарплаты. 10. Читаешь Толстого – и укрепляется вера в то, что жизнь – великая ценность и что она, жизнь, продолжаться будет, и мысль продолжаться будет.

6. 1. После случая с «болотной копейкой» Павел носа не повесил, а продолжал с удвоенной энергией тусоваться с рабочими. 2. Варвара говорит Катерине, что я не умела лгать, но научилась. 3. Виновниками семейных вспышек бывают взрослые, которые переносят грипп на ногах. 4. Судя по тем находкам, которые они нашли, уже есть веские доказательства в пользу этой гипотезы. 5. Цензура не хотела, чтобы поэма Гоголя увидела свет, но близкие Гоголя подсуетились и пробили это дело. 6. На научном семинаре автор изложил главную суть этой книги. 7. Путь, который избрал герой ошибочен, неправильный. 8. Появление в рабочей слободке новых людей является прогрессивным явлением. 9. Решение Пьера было воспринято как должное. Никто и оспаривать-то его не стал, так как против фактов не попрешь. 10. Уши у него торчали кто куда.

7. 1. Простив Анну, Каренин не понизил своего достоинства. 2. Наша передовая свинарка воспитала 257 поросят. 3. Машина будет использована не только для решения задач, а также для контроля. 4. Водитель должен помнить, что, повернув с проспекта направо, ему придется пересечь трамвайные пути. 5. Ленский вызвал Онегина на дуэль за то, что тот цеплялся к Ольге. 6. Как вывести личность на новый уровень знания? 7. Содержание банки следует вылить в кастрюлю. 8. В Никитском ботаническом саду открылась для посетителей коллекция хризантем. 9. Андрей Болконский ищет смысл в подвиге, который он хочет завоевать не в штабе, а на поле боя. 10. Он раскрылся нараспашку.

8. 1. Н.В. Гоголь изваял сатирическую галерею образов помещиков. Вот, например, праздничный мечтатель Манилов. 2. Подъезжая к городу, начался сильный ветер. 3. В конце спортивной залы стояли легкие креслы. 4. Геннадий позвал нас к нему. 5. Иудушка Головлев был профессиональным подхалимом. 6. Надо поторопиться, ибо мы опаздываем на поезд. 7. Широкомасштабный комплекс инициатив, выдвинутых в последнее время, остается в силе. 8. В своей автобиографии Есенин своим учителем называет Пушкина. 9. Благодаря ей состоялась экскурсия. 10. В «Восемнадцатом году» А. Толстого описываются события гражданской войны.

9. 1. На предложение Кабанихи выйти повыть на крыльце по поводу отъезда ее мужа Катерина скромно возразила: «Чего народ-то смешить?» 2. Программы местного телевидения оставляют желать много лучшего. 3. Магазин ориентирован специально для школьников. 4. Владимир уговорил Марью Гавриловну тайно расписаться. 5. Согласно приказа директора магазин перешел на круглосуточную работу. 6. Придавая большое внимание методологии исследования, был выполнен ряд экспериментов. 7. ЛДПР выступил с критикой правительства. 8. Вес некоторых грибов достигает полтора грамм. 9. Диссертация содержит анализ теории и практики решения проблемы укрепления эконимической базы предприятия нашего времени. 10. В повести Л.Н.Толстого «Детство» изображен облик простой русской женщины.

10. 1. Наша страна поддерживает дружеское отношение со многими странами мира. 2. Большое внимание благоустройству города будет оказано в ближайшее время. 3. Общественные контролеры передали прокурору акт, который сейчас принимает свои меры. 4. Если бы Ольга не спасла Обломова, он так бы и умер в халате. 5. Билет с плацкартом на субботний день достать очень трудно. 6. Археологи заметили, что покойники из южного захоронения перекликаются с покойниками из северного захоронения. 7. Впереди его бежала серая лошадка. 8. На поверхности рельс поблескивали огоньки уходящего поезда. 9. На стене висели портреты Репина. 10. Лишь блеснул и сейчас же закатился персонаж Ленского в романе А.Пушкина «Евгений Онегин».

11. 1. Пьер целеустремленно стремится к намеченной цели. 2. Том и Гек решили устроить побег Джиму – ихнему негру. 3. Мальчик выглядел как бы нездоровым. 4. По багажу нам поступает много претензий. 5. Необходимо признание креугольных прав человека. 6. Лермонтов отвергает сложившийся веками устой крепостной России. 7. Как сказал бы наш брат журналистка… 8. В книге описывается происхождение религии и какими обрядами она сопровождается. 9. Узнать о мировоззрениях Павла, хозяйка насторожилась. 10. Имея достаточный навык в работе машиниста, мне хочется дать несколько дельных советов.

12. 1. Участвовать на выборах – ответственное мероприятие. 2. «Тайм» опубликовала интересную статью. 3. Не убоявшись прессинговой политики, депутатами был дан отлуп центру, приняв специальное постановление, подтвердившее суверенизацию субъекта Федерации. 4. Жена его повешалась, сын умер от чахотки, дома осталась одна старушка мать. 5. Это наказание носит чисто воспитательский характер. 7. Учитель обязан содействовать исправлению допущенных родителями ошибок. 8. Силясь побороть слабость, она шла вперед. 9. Руки малыш заклал за спиной, а ноги расставил. 10. Превознемогая боль, раненый полз к избе.

13. 1. Хозяйка – женщина, заботившая о том, чтобы ее дом был первым в городе. 2. Скромность взяла верх, и он вошел в комнату, а сошел вниз по ступенькам. 3. Прежняя практика создания предприятий навечно ушла в прошлое. 4. Сейчас важно осознать ту роль, которую заняла церковь в жизни общества. 5. Депутат должен носить ответственность в своих действиях перед избирателями. 6. Брав с боем склад, был ранен командир. 7. Ломка феодальных пут дала свои результаты. 8. Если это гипотетически предположить, то ситуация видится в ином свете. 9. Середина 60-х годов примечательна расцветом и апогеем сурового стиля. 10. Раскольников – убийца, Соня – проститутка, но вместе им хорошо.

14. 1. Студент спросил, что можно ли ему присоединиться к нашей группе. 2. В романе такие абсурды, что голова болит, пока найдешь разгадку. 3. Пораженное место кислотой или щелочью необходимо быстро и осторожно протереть чистой тряпкой, бинтом или ватой. 4. Еще очень огромные богатства морей остаются втуне. 5. Хлестаков воспользовался заблужденностью чиновников. 6. Наращиваются новые связи… 7. На первой странице нарисован рисунок, изображающий женщину с цветком. 8. Образ моря, стихии, движения прекрасны в стихах А.Пушкина. 9. Раненый хотел пить, как резаный. 10. Это были недобитки буржуазизма.

15. 1. Андрея Болконского отдали местным жителям для выздоровления. 2. Красной строкой в его творчестве проходить тема любви. 3. Кондитерский цех запущен. 4. Через пару минут состоится знакомство с певицей. 5. Такие вдохновенные строки посвятил Н.А.Некрасов женам декабристов, последовавших за своими осужденными мужьями в Сибирь. 6. Юноша оказался в более лучшем положении. 7. Передавайте ей привет. 8. Написав «Смерть поэта», которое мгновенно облетело весь Петербург, стало ясно, что в России появился новый великий поэт. 9. Пришло порядка двадцати человек. 10. Мальчик одел очки и сразу повзрослел.

16. 1. В книге описываются события, имеющие место в XVI веке. 2. Впереди с широкой грудью шел рабочий. 3. Огурцы моются под струей холодной воды. 4. Заяц очень испугался, и шерсть у него встала колом. 5. Одной из главных проблем, выдвигаемых в романе, — это становление личности Пьера. 6. Бабушка от неожиданности заорала. 7. В Москве она побывала у Зинаиды Волконской, которая для проводов Марии пригласила всех итальянских певцов и певиц, которые были в то время в Москве. 8. Он выполнил данную им торжественную аннибалову клятву. 9. Художник видел и верил в будущее страны. 10. Солдаты князя Игоря отступили в неравной схватке.

17. 1. Боясь о том, что цензура не пропустит, поэт не печатал это стихотворение. 2. Поклоняться и боготворить Молчалина – вот смысл жизни Софьи, однако Чацкий не может в это поверить. 3. Фурманов изобразил Чапаева добрым, простым человеком и в то же время мужественным героем. 4. Не только наша молодежь, но и молодежь всего мира боролись и будут бороться против тех, кто нарушает права человека. 5. Актер, сыгравший бы эту роль, заболел. 6. Сестра старше него. 7. Крымские впечатления и наблюдения нашли отражение в произведениях М. Горького. 8. Столкновение с жизнью накапливает не по возрасту большой опыт. 9. Дельфины относятся к млекопитающимся. 10. Зная, чем ему грозит за участие и тем более руководство демонстрацией, Павел не отказался от великой чести нести знамя.

18. 1. Побывав на Кавказе, у Лермонтова стало много друзей. 2. Игрушки, выпускающиеся на нашей фабрике, известны помимо нашей страны. 3. У большей части машин были худые кузова. 4. Не обращая ни на что вокруг, он стал говорить. 5. Поэт пишет удивительное стихотворение «Родина», в которой он ставится выше всех предрассудков ложного патриотизма. 7. Здесь человека берут в такой оборот, что он поет прощальную лебединую песню. 8. Членов правительства нельзя будет призывать к уголовной ответственности. 9. В образе Манилова Гоголь показал помещика, который живет мечтами и неспособного к практической деятельности. 10. Как вам отдыхается?

19. 1. Вспомним еще одну аналогию из мифологии: Геракл стал пионером гидросмыва, пустив воды реки в авгиевы конюшни. 2. Стрелу Днепра на лук тугой плотины Ты, Запорожье, ложишь для добра. 3. Степь поросла глухой травой-кобелем. 4. Еще совсем недавно ему пели фимиамы. 5. В передаче «Доброе утро» я дебютирую впервые. 6. Скотинин прямо предлагал себя в женихи опекуну Софьи. 7. В нашем магазине вы можете приобрести памятные сувениры. 8. Моя приятельница – веселый, задорный и, можно сказать, примитивный человек. 9. Прятаться за чужие широкие спины – дело недостойное. 10. Он показал смерть девушки одной очень хорошей. Эта девушка любила сильно парня, но любовь у них стала несчастьем. Мать этого парня сказала, что она не позволит жениться. Потому что девушка была очень бедная, а парень был очень сильно богат. Но они никак не могут ужиться без друга. Парень говорит своей любимой, мол, я всегда буду рядом с тобой. Когда же девушка бежит топиться, он стоит в стороне, как вроде он совсем не виноват в ее гибели и смерти.

20. 1. Выступающий управляющий щедро приводил примеры, доказывающие, что общение молодежи в общежитии оставляет желать лучшего. 2. Забитая и малограмотная, а порой и вовсе неграмотная, Пелагея с детства всегда чего-то боялась. Но вот ее муж, казалось бы, умер. В ее глазах появился крохотный луч. От него у Ниловны и зарябило в глазах. 3. Вдруг Герман услыхал скрип рессор. Это была графиня. 4. Он крепко держит в руках штурвал руля. 5. Все более и более накипало во мне настоящее негодование, ведь она знает же, что я взаправду люблю ее. 6. Ухватила Поволжье кощавая рука голода. 7. Получить фиаско может каждый спортсмен. 8. Шерсть у зайца на грудке желтоватая, на брюшке – светло-серая, дымчатая, но заяц веселый и сообразительный. 9. Ведется работа в деле быстрого обновления дойного стада. 10. Утки проплывают мимо селезней с таким видом, будто век их видеть не хотят. У этих вообще полный беспредел. По жизни селезень обречен гибнуть за утку. В момент опасности он не имеет никакого морального права взлететь раньше ее. Подлые подруги довольно часто подставляют пернатых мужиков, не отрываясь от земли до последнего. Именно поэтому при определенной сноровке селезня можно поймать голыми руками.

21. 1. Эти места отличаются наибольшим интенсивным движением. 2. Он приехал на стройку два года тому назад. 3. В романе Л.Н.Толстого «Война и мир» эпопейный размах. 4. Свернувшегося калачиком солнце грело дремлющегося кота. 5. Русь отбивалась от печенегов, половцев и прочей дряни. 6. Нам махают платками. 7. Критичный анализ этого исследования был не очень критический. 8. Самое отвратительное в Молчалине то, что он не имеет своего мнения, и низкопоклонство. 9. Участники творческой экспедиции рассказали о перспективах на будущее. 10. Сейчас важно задействовать в качестве ключевого человеческий фактор.

2

Зайцева

Учитель русского языка и литературы

1.Спиши текст, расставляя знаки препинания и вставляя пропущенные буквы. 

Под большими городами человек скоро опустоша..т природу. Ра..гоняется птица уход..т зверь леса очищаются и скудеют одни грибы долго спор..т уп..раются каждое мочливое лето родятся в самых казалось бы истопта…ых исхож…ных местах. Но слышно и они начали …давать. Отступают грибы от городов. Раньше пешком за ними ходил полегоньку теперь езжай на поезде трясись на мотоц..кле и как знать не будет ли время когда только с вертолётом ра…живёш..ся настоящими сырыми ельничными груздями. (Л. Никонов)

-Запиши основную мысль текста.

— Приведи 2 литературных примера, подтверждающих мысль Л. Никонова.

— К какому стилю относится этот текс ?

-Какой тип речи представлен в тексте?

-Укажи средства связи предложений 2и 3, 4 и 5.

— из последнего предложения выпишите синонимы. В чём их стилистическая разница?

— В каком значении употреблено слово РАЗЖИВЁШЬСЯ в последнем предложении? Подбери к нему синоним.

2. Укажите случаи неверного, разнотипного оформления однородных частей или членов предложения. Исправьте ошибки.

1. Путь, который выбрал герой, ошибочен, неправильный.

________________________________________________________________

2. Книга научила смелости, честности и не болеть.

_________________________________________________________________

3. Стихотворение «Анчар» было отдано в печать, но не пропущенное цензурой.

__________________________________________________________________

4. Желаю вам здоровья и не болеть.

__________________________________________________________________

5. Всем понравилась повесть, опубликованная в последнем номере журнала
и который рассказывает о современной молодёжи.

3. Подберите к каждому прилагательному существительное.

Деловой ________________________

Деловитый ______________________

Искусственный __________________

Искусный _______________________

Обидный ________________________

Обидчивый ______________________

Эффектный ______________________

Эффективный ____________________

Рыбий __________________________

Рыбный _________________________

Зачетная работа по русскому языку 11 класс

Блок 1. Орфографические и синтаксические нормы языка

1.     
Распределите слова по группам с учётом
орфографических особенностей. Назовите орфограммы.

Безнравственный,
бескорыстный, колются, ветреный, вовсе не интересный, скворчонок, строят,
раскорчёванный (лес), навзничь, не вооружён, карманный, холщовый, по-медвежьи,
презрение, прекрасный, приморский, выкачанная (нефть), тушёнка, серебряный,
соломенный, чрезмерный, нехоженый.

2.     
Перепишите текст, расставьте пропущенные буквы и
знаки препинания, раскройте скобки.

Каждое
новое пок…ление детей всегда созд…ёт снов… и снов… великое множество подобных
глаголов (н…)зам…чая своего языкового новаторства. Активность оглагол…вания
имён существительных сн…жае(т,ть)ся у них лиш(?) (по)мере того как они выход…т
из дошкольного возр…ста. Как близко пр…мыкают созд…ваемые ими глагольные формы
к тем формам которые созд…(н,нн)ы и созд…ю(т,ть)ся народом видно (на)пример из
слова «ра…кулачить». (В)первые я услыхал это слово (пол)века назад даже раньше.
Гай внуч…нок И.Е. Репина крепко …жал свой кулачишк… и сказал Ну(ка) ра…кулач(?)
мои пальцы!

В
те времена такого слова ещё (не)существовало в народе так как ра…кулач…вание
ещё (н…)стало историческим фактом. Для того что(бы) ребёнок мог зара(н,нн)ее так
сказать (на)перёд ск…нструировать то самое слово которое лет двадцать спустя
было созд…(н,нн)о народными ма(с,сс)ами нужно что(бы) он в сов…ршенстве владел
теми(же) пр…ёмами построения слов которые выр…ботал (в)течени… тыс…ч…летий
народ.

(К.И.
Чуковский. «От двух до пяти»)

3.     
Поставьте существительное, стоящее в скобках, в
форму родительного падежа множественного числа.

Купить
килограмм (абрикос, ананас, апельсин); группа (грузин); десять (ватт, вольт);  двести
(грамм); много (килограмм, километр); группа (таджик, татарин, узбек, якут); вернуться 
из далёких (ущелье); пара (носок); килограмм (мандарин); несколько чистых
(полотенце); салат из (помидор); урожай (баклажан); много свободных (место); несколько
(юноша)

4.     
Укажите синтаксические средства выразительности,
использованные В. Астафьевым в данном тексте (средство — номера
предложений)

 (1)Растаял
мокрый снег. (2)Осталось на стекле приклеившееся птичье пёрышко. (3)Смятое.
(4)Тусклое и до боли сиротливое. (5)Может, птаха малая стучалась ночью клювом
по стеклу, просилась в тепло, а я, тугой на ухо человек, не услышал её, не
пустил. (6)И пёрышко это, как укор, белеет на стекле.

(7)Потом
обсушило солнцем стекло. (8)Унесло куда-то пёрышко. (9)А тоска осталась.
(10)Должно быть, не перезимовала птичка, не дотянула до тепла и весны, вот
сердцу-то и неловко, печально. (11)Залетело, видать, в  меня пёрышко.
(12)Прилипло к моему сердцу.

5.     
Укажите случаи неверного, разнотипного
оформления однородных частей или членов предложения. Исправьте ошибки.

1.
Путь, который выбрал герой, ошибочен, неправильный.

2.
Книга научила смелости, честности и не болеть.

3.
Стихотворение «Анчар» было отдано в печать, но не пропущенное цензурой.

4.
Желаю вам здоровья и не болеть.

5.
Всем понравилась повесть, опубликованная в последнем номере журнала
и который рассказывает о современной молодёжи.

6.     
Постройте схемы предложений с прямой речью,
исправив допущенные ошибки. Расставьте знаки препинания.

1.
Сергей сказал, что я вернусь на следующей неделе.

2.
Я знаю проговорила она вас не убедишь.

3.
Гроза будет сказал Федя посматривая на облака.

4.
Врач спросил какая у вас была температура.

5.
Ученик сказал, что я ещё не подготовился к ответу.

6.
Тарас сказал, что завтра же мы едем в Запорожье, чего откладывать.

7.     
Устраните ошибки, связанные с употреблением вводных
слов и предложений.

1.
Кажется, видно, работа ещё не закончена.

2.
Но увы, к счастью, коньки были с ним.

3.
Вошёл какой-то незнакомый мужчина, пожалуй, скромно, даже бедно, одетый.

4.
Покупатель медлит, обдумывая, какой лучше выбрать подарок, вероятно.

5.
Это был огромный дуб, с обломанными, видно, сучьями давно.

6.
Девочки напряжённо вглядывались в даль моря: наверное, на горизонте появится
лодка.

7.
По словам рыбаков, ночью был шторм, а теперь штиль.

8.
Книга – источник знаний, как видят многие.

8.     
Укажите недочёты в построении предложений
(нарушение связи между словами, смешение конструкции,  пропуск членов
предложения и т.д.). Отредактируйте фразы.

1.
Учащиеся нашей школы приняли участие и стали призёрами областного смотра
фольклорных коллективов художественной самодеятельности.

2.
Солома, впитывая в себя ценные питательные соки, выделяемые в силосе при
молочнокислом брожении, охотно доедается скотом.

3.
Где проходила наша экспедиция, то были глухие места.

4.
Необходимо знания и навыки, полученные учащимися в школе, впоследствии могли бы
быть применены ими в жизни.

5.
На трикотажной фабрике установлено 200 новых станков, а также для выработки
объёмной полиэфирной нити.

6.
Ставшая популярной ипотечная программа банков, она успешно работает уже на
протяжении двух лет.

7.
После обретения независимости Киргизию покинуло свыше тысяч граждан.

9.     
Установите соответствие между предложениями и их
характеристикой с точки зрения соблюдения морфологических норм.

А. Нехорошо
присваивать чужие идеи.

Б. Нечего меня
задобривать.

В. Позови меня,
когда вода вытекет из ванны.

Г. Не трожь этот
кустик: он здесь к месту.

Д. Спортсмен
достигнул хороших результатов.

1. Нарушены нормы
образования личной формы глагола.

2. Нарушены нормы
образования формы прошедшего времени глагола.

3. Нарушены нормы
образования видовой пары глагола.

4. Нарушены нормы
образования формы повелительного наклонения глагола.

5.
Морфологические нормы не нарушены.

10.  Исправьте ошибки в употреблении
падежных форм и предлогов.

1.
За это мы поговорим с вами на следующей неделе.

2.
Тренер уверен за своего воспитанника.

3.
Преподаватель подчеркнул о необходимости выполнять домашнее задание.

4.
Его новый образ жизни сводился лишь в отказе от курения.

5.
Мы всей семьёй будем контролировать за ходом лечения бабушки.

6.
Наш профессор только вчера вернулся с Киева, где он выступал с докладом на
научной конференции.

7.
В журнале «Новый мир» опубликована рецензия о новой книге Александра
Солженицына.

8.
По окончанию института вы станете высококвалифицированными специалистами.

Блок 2. Работа с текстом

Прочитайте предложения и выполните задания 1, 2, 3.

А. Им было легче, чем европейским коллегам, провести такие
исследования: в США и Канаде водится монарх – классический образец
перелётной бабочки.

Б. Американские энтомологи внесли немалый вклад в изучение перелётов
бабочек.

В. Регулярно в строго определённое время, весной и осенью, эти красивые
бабочки совершают дальние путешествия.

Г. Среди других бабочек нет пожалуй равных им по умению преодолевать на
крыльях далёкие расстояния

1.                 
В каком предложении допущена пунктуационная
ошибка: не выделено вводное слово?

2.                 
В какой последовательности нужно расположить
предложения, чтобы получился текст?

3.                 
Укажите ошибку в синтаксической характеристике
предложения.

1) Предложение А сложное, бессоюзное, с
обособленным приложением.

2) Предложение Б простое, распространённое.

3) Предложение В простое, распространённое,
осложнённое уточняющими обстоятельствами.

4) Предложение Г простое, распространённое,
грамматическая основа бабочек нет.

Блок 3. Задание с развернутым ответом

Прочитайте текст и выполните задания к нему

(1)Кажется,
я был в пятом классе, когда у нас появились сразу несколько новых молодых
учителей, только что вышедших из университета. (2)Одним из первых появился
Владимир Васильевич Игнатóвич – учитель химии. (3)Это был молодой
человек, только что с университетской скамьи, с чуть заметными усиками,
маленького роста, с пухлыми розовыми щеками, в золотых очках. (4)Говорил он
голосом, в котором звучали тонкие, как будто детские, нотки. (5)В классе
несколько робел, и лицо его часто заливал застенчивый румянец. (6)Новый учитель
обращался с нами вежливо, преподавал старательно, заданное спрашивал редко, к
отметкам выказывал пренебрежение, уроки объяснял, как профессор, читающий
лекцию.

(7)Первым
результатом его системы было то, что класс почти перестал учиться.
(8)Вторым – то, что ему порой начали слегка грубить. (9)Бедный юноша,
приступавший к нам с идеальными ожиданиями, вынужден был расплачиваться за
общую систему, которая вносила грубость и цинизм. (10)Впрочем, это было
недолго. (11)Однажды, когда класс шумел и Игнатóвич напрасно надрывал свой
мягкий голосок, одному из нас показалось, будто он назвал нас стадом баранов.
(12)Другие учителя очень часто называли нас стадом баранов, а порой и хуже.
(13)Но то были другие. (14)Они были привычно грубы, а мы привычно покорны.
(15)Игнатóвич же сам приохотил нас к другому обращению.

(16)Один
из учеников, Заруцкий, очень хороший, в сущности, малый, но легко поддававшийся
настроениям, встал среди шумевшего класса.

– (17)Господин
учитель, – сказал он громко, весь красный и
дерзкий. – (18)Вы, кажется, сказали, что мы стадо баранов.
(19)Позвольте вам ответить, что… в таком случае…

(20)Класс
вдруг затих так, что можно было слышать пролетевшую муху.

– (21)Что
в таком случае… вы сами баран…

(22)Стеклянная
колбочка, которую держал в руках Игнатович, звякнула о реторту. (23)Он весь
покраснел, лицо его как-то беспомощно дрогнуло от обиды и гнева. (24)В первую
минуту он растерялся, но затем ответил окрепшим голосом:

– Я
этого не говорил… (25)Вы ошиблись…

(26)Простой
ответ озадачил. (27)В классе поднялся ропот, значение которого сразу разобрать
было трудно, и в ту же минуту прозвенел звонок. (28)Учитель вышел; Заруцкого
окружили. (29)Он стоял среди товарищей, упрямо потупившись и чувствуя, что
настроение класса не за него. (30)Сказать дерзость учителю, вообще говоря,
считалось подвигом, и если бы он так же прямо назвал бараном одного из
«старых», то совет бы его исключил, а ученики проводили бы его горячим
сочувствием. (31)Теперь настроение было недоуменно-тяжёлое, неприятное…

– (32)Свинство,
брат! – сказал кто-то.

– (33)Пусть
жалуется в совет, – угрюмо ответил Заруцкий.

(34)Для
него в этой жалобе был своего рода нравственный выход: это сразу поставило бы
нового учителя в один ряд с учителями старыми и оправдало бы грубую выходку.

– (35)И
пожалуется! – сказал кто-то.

– (36)Конечно!
(37)Думаешь, спустит?

(38)Этот
вопрос стал центром в разыгравшемся столкновении. (39)Прошло два дня, о жалобе
ничего не было слышно. (40)Прошёл день совета… (41)Признаков жалобы не было.

(42)На
следующий урок химии Игнатóвич явился несколько взволнованный; лицо его было
серьёзно, глаза чаще потуплялись, и голос срывался. (43)Видно было, что он
старается овладеть положением и не вполне уверен, что это ему удастся.
(44)Сквозь серьёзность учителя проглядывала обида юноши, урок шёл среди
тягостного напряжения. (45)Минут через десять Заруцкий, с потемневшим лицом,
поднялся с места. (46)Казалось, что при этом на своих плечах он поднимает
тяжесть, давление которой чувствовалось всем классом.

– (47)Господин
учитель… – с усилием выговорил он среди общей тишины. (48)Веки у
молодого учителя дрогнули под очками, лицо всё покраснело. (49)Напряжение в
классе достигло высшего предела.

– (50)Я…
прошлый раз… – начал Заруцкий глухо. (51)Затем, с внезапной
резкостью, он закончил:

– Я
извиняюсь.

(52)И
сел с таким видом, точно сказал новую дерзость. (53)Лицо у Игнатóвича
посветлело, хотя краска залила его до самых ушей. (54)Он сказал просто и
свободно:

– Я
говорил уже, господа, что баранами никого не называл.

(55)Инцидент
был исчерпан. (56)В первый ещё раз такое столкновение
разрешилось таким образом. (57)«Новый» учитель выдержал испытание.
(58)Мы были довольны и им и – почти бессознательно – собою,
потому что также в первый раз не воспользовались слабостью этого юноши, как
воспользовались бы слабостью кого-нибудь из «старых». (59)Самый эпизод скоро
изгладился из памяти, но какая-то ниточка своеобразной симпатии, завязавшейся
между новым учителем и классом, осталась.

 (По
В.Г. Короленко*)

 *Владимир
Галактионович Короленко (1853–1921) –  русский писатель, журналист, публицист,
общественный деятель.

1.     
Какие из высказываний соответствуют содержанию
текста? Укажите номера ответов.

1) Стремясь
перекричать класс и понимая, что ему не удаётся это сделать, учитель химии
оскорбил учеников.

2) Учитель химии
Владимир Васильевич Игнатóвич получил университетское образование.

3) Одноклассники
Заруцкого не одобрили его поступок по отношению
к учителю химии.

4) Заруцкий принёс
учителю свои извинения, потому что побоялся, что тот пожалуется в совет.

5) Разрешение
конфликта привело к возникновению определённой симпатии во взаимоотношениях
класса с учителем.

2.     
Ознакомьтесь с информацией о тексте. Заполните
пропуски.

 Примерный круг проблем

Авторская позиция

1. <…>

1. Уважительное
отношение со стороны учителя создаёт условия для формирования в характере
учеников лучших качеств: способности к объективной оценке своего поведения по
отношению к другим людям, способности к совершению честного поступка не в
связи со внешним давлением, а по велению собственной совести, потребности в
самоуважении. Учитель может влиять на формирование характеров учеников самой
своей личностью, свойственной ему манерой поведения, своей речью.

2. Проблема
решения конфликта между учителем и учениками. (При каком условии последствия
конфликта, недоразумения, возникшего между учителем и учениками, могут быть
устранены?)

2. <…>

3. <…>

3. Человеку
бывает непросто признать свою вину, но, для того чтобы восстановить
справедливость, вернуть доверительные взаимоотношения и сохранить
самоуважение, важно принести свои извинения тому человеку, которому была
нанесена обида.

4. Проблема
высокой требовательности по отношению к интеллигентному человеку. (Какие
требования предъявляются к интеллигентному человеку?)

4. <…>

5. Проблема
противостояния человека и сложившейся системы. (Может ли человек
противостоять системе?)

5. Человек,
имеющий твёрдые принципы, способен не только противостоять общепринятой и
даже одобряемой системе отношений, но и добиваться успеха в этом
противостоянии.

3.     
Напишите комментарий к одной из проблем текста
(укажите проблему)

1.
Теоретические
основы
культуры
речи

1.1
Кто
традиционно
считается
родоначальником
русского
литературного
языка?

1)
Л.Н.Толстой;

2)
К.Г.Паустовский;

3)
А.С.Пушкин;

4)
А.П.Чехов.

1.2.
Разновидность
русского

языка,

существующая

как

система
местных
говоров,
распространенных
на
определенной
территории

это…

1)
жаргон;

2)
диалект;

3)
литературный
язык;

4)
просторечие.

1.3.
К
книжным
стилям
литературного
русского
языка
не
относится

1)
разговорный
стиль;

2)
художественный
стиль;

3)
публицистический
стиль;

4)
научный
стиль.

1.4.
К
функционально-смысловым
типам
речи
не
относится:

1)
обобщение;

2)
рассуждение;

3)
описание;

4)
повествование.

1.5.
Речь

социальных
и

профессиональных

групп

людей,

объединенных
общностью
занятий,
интересов,
социального
положения
и
т.д.

это…

1)
литературный
язык;

2)
просторечие;

3)
диалект;

4)
жаргон.

1.6.
К
аспектам
культуры
речи
не
относится:

1)
нормативный
аспект;

2)
коммуникативный
аспект;

3)
психологический
аспект;

4)
этический
аспект.

1.7.
Высшей
формой
национального
русского
языка
является:

1)
просторечие;

2)
жаргон
и
арго;

3)
литературный
язык;

4)
диалект.

2.
Лингвистические
словари
русского
языка

2.1
Каким
словарем
вы
не
воспользуетесь
при
выборе
варианта:

Экономный

экономичный

Августовский
или
агустОвский

Смелый

храбрый

1)
орфоэпическим
словарем;

2)
этимологическим
словарем;

3)
словарем
паронимов;

4)
словарем
синонимов.

2.2.
Что
содержит
этимологический
словарь?

1)
ударение;

2)
лексическую
сочетаемость;

3)
происхождение;

4)
написание.

2.3.
Какой

словарь
содержит

слова
в

их

правильном
произношении

и
ударении?

1)
орфографический
словарь;

2)
орфоэпический
словарь;

3)
этимологический
словарь;

4)
толковый
словарь.

2.4.
Каким
словарем
Вы
воспользуетесь,
чтобы
узнать
разницу
в
значении
слов
командированный

командировочный,
элитный

элитарный,
бережный

бережливый?

1)
словарем
синонимов;

2)
словарем
антонимов;

3)
словарем
паронимов;

4)
словарем
фразеологизмов.

2.5.
В
каком
словаре
можно
узнать
значение
сочетания
«ахиллесова
пята»?

1)
в
орфоэпическом
словаре;

2)
в
орфографическом
словаре;

3)
в
толковом
словаре;

4)
в
словаре
фразеологизмов.

2.6.
Из
какого
словаря
можно
узнать
значение

слов
«аккумулировать»,

«маклер»,
«аутсорсинг»?

1)
словаря
паронимов;

2)
словаря
синонимов;

3)
словаря
иностранных

слов;

4)
словаря
лексической
сочетаемости.

2.7.
Каким
словарем
вы
не
воспользуетесь
при
выборе
варианта:

Афера
или
афёра

Ростовщик
или
растовщик

Заплатить

оплатить

1)
орфографическим
словарем;

2)
орфоэпическим
словарем;

3)
словарем
антонимов;

4)
словарем
паронимов.

3.
Нормы
современного
русского
литературного
языка:
нормы
ударения

3.1.
Правильно
выделен
ударный
гласный
в
слове:

1)
клалА;

2)
тУфля;

3)
арахИс;

4)
щАвель.

3.2.
В
каком
слове
ударение
на
четвёртом
слоге?

1)
осведомить;

2)
демократия;

3)
аристократия;

4)
заиндеветь.

3.3.
В
каком
слове
ударение
падает
на
первый
слог?

1)
баловать;

2)
мастерски;

3)
кремень;

4)
кухонный.

3.4.
Укажите
ряд,
в
котором
во
всех
словах
падает
ударение
на
второй
слог.

1)
ревень,
коклюш,
мытарство;

2)
триптих,

дремота,
надолго;

3)
форзац,
фетиш,
оптовый;

4)
заем,
кедровый,
облегчить.

3.5.
В
каком
слове
ударение
поставлено
неправильно?

1)
кладовАя;

2)
квартАл;

3)
диспансЕр;

4)
искрА.

3.6.
В
каком
из
слов
постановка
ударения
определяется
контекстом?

1)
свекла;

2)
щавель;

3)
камбала;

4)
рожки.

3.7.
В
каком
словосочетании
ударение
падает
на
предлог?

1)
из
дома;

2)
(раздать)
по
пять
(штук);

3)
(бросить)
на
пол;

4)
(посмотреть)
на
дом.

4.
Нормы
современного
русского
литературного
языка:
орфоэпические
нормы

4.1.
В
каком
ряду
верно
указано
произношение
слова?

1)
преЁмник;

2)
скабрЁзный;

3)
одновремЁнный;

4)
гладкошЁрстный.

4.2.
В
каком
слове
выделенная
буква
обозначает
твёрдый
согласный?

1)
муЗей;

2)
Тесты;

3)
акаДемик;

4)
Тема.

4.3.
Сочетание
ШН
произносится
в
слове:

1)
молочный;

2)
спичечный;

3)
горничная;

4)
бутылочный.

4.4.
В
каком
слове
допущена

орфографическая

ошибка,

появившаяся
в
связи
с
неправильным
произношением
слов?

1)
инциндент;

2)
поскользнуться;

3)
оплатить;

4)
юриспруденция.

4.5.
В
каком
ряду
оба
слова
произносятся

со
звуком
[О]
под
ударением?

1)
оседлость,
белесый.

2)
маневр,
острие;

3)
осетр,
опека;

4)
афера,
поблекший.

4.6.
В
каком
слове
выделенная
буква
обозначает
мягкий
согласный?

1)
эДем;

2)
синТез;

3)
Декан;

4)
криТерий.

4.7.
Сочетание
ЧН
произносится
в
слове:

1)
скучно;

2)
Ильинична;

3)
копеечный;

4)
булочная.

5.

Нормы
современного

русского
литературного

языка:
нормы
лексической
сочетаемости

5.1.
Лексическая
сочетаемость
слов
не
нарушена
в
предложении:

1)
Днем
температура
воздуха
прогревалась
до
14
градусов.

2)
Наблюдается
чудовищное
улучшение
условий
жизни.

3)
Ученики
пристально
слушали
рассказ
учителя.

4)
Все
остальные
выразили
глубокое
сомнение,
что
он
справится.

5.2.
Слово
запасный
лексически
сочетается
со
словом:

1)
игрок;

2)
выход;

3)
мяч;

4)
вариант.

5.3.
Лексическая
сочетаемость
слов
не
нарушена
в
словосочетании:

1)
крепкая
любовь;

2)
крепкая
ненависть;

3)
крепкий
ливень;

4)
крепкая
жара.

5.4.
Лексическая
сочетаемость
слов
не
нарушена
в
предложении:

1)
Перед
нами
предстает
самый
многочисленный
класс

крестьяне.

2)
Побывав
в
кино,
я
думаю,
способна
ли
я
на
такие
поступки,
которые
делают
герои
фильма.

3)
Шесть
стран соорудили спасательные экспедиции
на поиски экипажа

Италии.

4)
Этот
человек
произвёл
на
меня
большое
воздействие.

5.5.
Слово
закоренелый
лексически
сочетается
со
словом:

1)
патриот;

2)
домочадец;

3)
эгоист;

4)
президент.

5.6.
Лексическая
сочетаемость
слов
не
нарушена
в
словосочетании:

1)
большинство
времени;

2)
верить
в
неминуемую
победу;

3)
первоочередное
внимание;

4)
занять
у
кого-либо
денег.

5.7.
Лексическая
сочетаемость
слов
нарушена
в
словосочетании:

1)
повысить
урожайность;

2)
полчища
тружеников;

3)
по
замыслу
автора;

4)
завзятые
математики.

6.
Нормы
современного
русского
литературного
языка:
лексические
нормы

6.1.
В каком
предложении
нет
ошибки, связанной с употреблением
паронимов?

1)
Черные

цвета
вперемешку

с
бурыми
полосками

делают
оперение
селезня

очень
эффективным.

2)
Он
не
любил
работать
и
вёл
праздничный

образ
жизни.

3)
Этот
человек
был
полный
невежда
в
искусстве.

4)
Директор
сегодня
подписал
заявление
о
представлении
мне
отпуска.

6.2.
В
каком
предложении
нет
ни
тавтологии,
ни
плеоназма?

1)
Газета
сообщила,
что
в
нашем
городе
вновь
открывается
новый
магазин.

2)
Проект
гостиницы в центре города
был спроектирован
молодым
архитектором.

3)
При
ответе
абитуриент
должен
излагать
суть
вопроса.

4)
Мы
оснастились
рыболовными
снастями
и
отправились
рыбачить.

6.3.
Лексические
нормы
не
нарушены
в
предложении:

1)
Известно,

что
именно
русские

вложили
значительный

вклад

в
освоение

Аляски.

2)
Дорога
каждая
минута
времени.

3)
Алексей
мысленно
подумал,
что
это
немец.

4)
Ранние
произведения
поэта
понятны
и
доступны
каждому.

6.4.
В каком
предложении
вместо слова ЦАРСТВЕННЫЙ
нужно
употребить
ЦАРСКИЙ?

1)
У
графини
был
поистине
ЦАРСТВЕННЫЙ
вид.

2)
Императрица
ЦАРСТВЕННЫМ
жестом
отпустила
иностранных
послов.

3)
Был
у
матушки
Москвы,
и
у
Каспия
широкого,
и
у
ЦАРСТВЕННОЙ
Невы.

4)
В
восемнадцатом
веке
произошло
укрепление
ЦАРСТВЕННОЙ
власти.

6.5.
Ошибка, связанная с речевой
недостаточностью,
не допущена
в
предложении:

1)
На
плечи
фермера
ложится
ответственность
за
содержание
и
сохранность.

2)
Нужно

не

допускать

на

страницы

печати

и

телевидения

высказывания,

способные
разжечь
межнациональную
вражду.

3)
У
нас
ведется
большая
работа
по
улучшению
качества.

4)
Героизм
Корчагина

проявляется
не
только
во
время
войны,
в
бою,
но
и
в
мирные
дни.

6.6.
Укажите
предложение

без
двусмысленности.

1)
Есть
большое
желание
заполучить
очередное
издание
детектива.

2)
Коснемся
русских
наречий.

3)
Это
книга
о
технике.

4)
В
газетах
был
опубликован
закон
об
упразднении
транспортных
судов.

6.7.
В
каком
предложении
вместо
слова
БУДНИЧНЫЙ
нужно
употребить

БУДНИЙ?

1)
Однажды,
в
БУДНИЧНЫЙ
день,
поутру
я
с
дедом
разгребал
на
дворе
снег.

2)
Из
одной
БУДНИЧНОЙ
обстановки
пришлось
бы
увлечь

её
в
другую
такую
же
или
ещё
более
однообразную

и
скучную.

3)
Этот
художник
значительное

место
отводит
изображению

БУДНИЧНОЙ

стороны
войны.

4)
«Вот
мы
и
приехали
домой»,

уже
другим,
БУДНИЧНЫМ
голосом
сказал
капитан.

7.

Нормы
современного

русского
литературного

языка:
лексические

нормы
фразеологизмов

7.1.
В
каком
предложении
нет
ошибки
в
употреблении
фразеологизма?

1)
В
этих
событиях
важную
скрипку
играют
военные.

2)
Я
согласился
на
это
скрипя
сердцем.

3)
Я
не
ударился
лицом
в
грязь
и
блестяще
сдал
экзамены.

4)
Власти
не
собираются
искупить
свою
вину.

7.2.
В
книжно-письменной
речи
используется
фразеологизм:

1)
лиха
беда
начало;

2)
рог
изобилия;

3)
белыми
нитками
шито;

4)
втирать
очки.

7.3.
Во
фразеологизме
держать

востро
(со
значением
«будь
осторожен,

гляди
в
оба»)
пропущено
слово

1)
глаза;

2)
нос;

3)
волосы;

4)
ухо.

7.4.
Каково
значение
фразеологизма
«гомерический
смех
(хохот)»?

1)
радостный,
веселый;

2)
громкий,
неудержимый;

3)
детский;

4)
беззаботный.

7.5.
Ошибка
при
употреблении
фразеологизма
допущена
в
предложении:

1)
Он
постоянно
сидит
сложа
руки.

2)
Этот
человек
произвёл
на
меня
большое
впечатление

3)
Успехи
этого
ученика
желают
много
лучшего.

4)
Рассматривая

редакционную

почту,

я

обратил

внимание

на

необычное
письмо.

7.6.
Какой
из
фразеологизмов
является
разговорным?

1)
стереть
с
лица
земли;

2)
на
поле
брани;

3)
заря
новой
жизни;

4)
раскинуть
умом.

7.7.
Каково
значение
фразеологизма
«драть
глотку»?

1)
болеть;

2)
кричать;

3)
пить;

4)
наказывать
ремнем.

8.

Нормы
современного

русского
литературного

языка:

морфологические
нормы

8.1.
В
каком
ряду
все
существительные

относятся
к
мужскому
роду?

1)
шампунь,
стол,
бандероль;

2)
бюллетень,
тюль,
рояль;

3)
домишко,
диван-кровать,
ЦРУ;

4)
аэрозоль,
студень,
вуаль.

8.2.
В
каком
ряду
все
существительные

относятся
к
среднему
роду?

1)
авокадо,
кофе,
рагу;

2)
боржоми,
кафе,
сари;

3)
конфетти,
какао,
иваси;

4)
кольраби,
шасси,
пенсне.

8.3.
Найдите
прилагательное, от которого нельзя

образовать простую
форму
сравнительной
степени.

1)
громоздкий;

2)
хороший;

3)
дорогой;

4)
плохой.

8.4.
Укажите,

в

каком

ряду

оба
глагола

не

образуют
формы
1

лица
настоящего
или
будущего
простого
времени.

1)
служить,
хлестать;

2)
чудесить,
угораздить;

3)
переубедить,
полоскать;

4)
сиживать,
рыскать.

8.5.
Найдите
ряд,
в
котором
все
формы
образованы
правильно:

1)
в
обеих
сумках,
без
комментарий,

более
молодой;

2)
пара
туфлей,
около
пятисот
километров,
инженеры;

3)
восемьюстами
двумя,
кремы,
пара
носков;

4)
секторы,
яблок,
девяносто
двух.

8.6.
В
каком
ряду
все
три
формы
образованы
правильно?

1)
бухгалтеры,
инженеры,
брелоки;

2)
донья,
месяцы,
сектора;

3)
годы,
крема,
вечера;

4)
вееры,
выборы,
свитеры.

8.7.
В
каком
ряду
обе
формы
образованы

правильно?

1)
6
килограммов,
7
грузинов;

2)
пришел
из
яслей,
много
вафель;

3)
8
грабель,
выбор
туфель;

4)
10
кочерёг,
5
мандарин.

9.

Нормы
современного

русского
литературного

языка:

морфологические
нормы
имени
числительного

9.1.
В каком
предложении
допущена ошибка в
употреблении
числительного?

1)
Я
прочитала
свыше
семи
тысяч
восьмисот
пятидесяти
книг.

2)
К
ста
пятидесяти
шести
прибавить
семьсот
восемьдесят
три.

3)
Предложение
принято
пятистами
семьюдесятью
пятью
голосами
против
пятидесяти
восьми.

4)
Она
прочитала
книгу
из
пятисот
шестидесяти
восьми
страниц.

9.2.
В
каком
ряду
все
три
формы
образованы
правильно?

1)
три
сироты,
два
больных,
четыре
дома;

2)
двое
прохожих,

трое
граблей,
пятеро
раненых;

3)
две
руки,
два
письма,
трое
девушек;

4)
четверо
медвежат,
двое
растяп,
два
ботинка.

9.3.
Укажите

правильные

формы
творительного

падежа

числительного

8567.

1)
восьми
тысячами
пятьюстами
шестьюдесятью
семью;

2)
восьмью
тысячами
пятьюстами
шестьюдесятью
семью;

3)
восьмью
тысячами
пятистами
шестьюдесятью
семью;

4)
восьмью
тысячами
пятьюстами
шестидесятью
семью.

9.4.
В
каком
ряду
все
формы
числительного
образованы
правильно?

1)
до
ста,
до
семиста,
к
семистам;

2)
к
ста,
в
семиста,
семьюстами;

3)
в
семистах,
до
семисот,
в
ста;

4)
в
семистах,
до
семисот,
в
стах.

9.5.
Правильные

формы
числительных

использованы
в
словосочетаниях

1)
двое
котят,
трое
щипцов,
четверо
детей;

2)
две
девушки,

двое
ножниц,

трое
больных;

3)
два
окна,
четверо
женщин,

двое
друзей;

4)
четверо
сыновей,
пятеро
нерях,
трое
сердец.

9.6.
В каком
предложении
допущена ошибка в
употреблении
числительного?

1)
В

годы
второй
мировой
войны
в
Югославии

погибло
свыше

миллиона

семисот
тысяч
человек.

2)
Помимо
компании
ДОМО
в
рейтинг
быстрорастущих
компаний
попали
еще
двадцать

пять
торговых
сетей,

из
них
двенадцать

работают

в
продуктовой

рознице
и
только
две
в
розничной
торговле
бытовой
электроникой.

3)
К

двумстам

тридцати

участникам

конференции

присоединились

еще

сто
шестьдесят
три
делегата.

4)
В
энциклопедическом

словаре
насчитывается

около
шести

тысяч
пятиста
восьмидесяти
иллюстраций.

9.7.
Форма
числительного
дана
неверно
в
сочетании:

1)
достигать
ста
семидесяти
тысяч
пятисот
восьмидесяти
девяти;

2)
с
девятьюстами

двадцатью

пятью
тысячами

шестьюстами

семьюдесятью
четырьмя;

3)
о
трех
тысячах
семисот
восьмидесяти
девяти;

4)
к
двумстам
семи
тысячам
пятистам
девяноста
пяти.

10.
Синтаксические
нормы
русского
литературного
языка

10.1.
Найдите

предложение

без
нарушения
синтаксических
норм.

1)
Узкая
дорога
была
покрыта
проваливающимся
снегом
под
ногами.

2)
Я
разочаровался
отзывом,
данным
рецензентом
на
мою
дипломную
работу.

3)
Режиссер
сказал:
«Мы
работаем
сейчас
над
двумя
постановками:
«Вишневый
сад»
Чехова
и
пьеса
Володина
«С
любимыми
не
расставайтесь».

4)
Побывав

в
кино,

я
думаю,
способна

ли
я
на

такие

поступки,
которые
совершают
герои
фильма.

10.2.
Укажите
предложение
с
грамматической
ошибкой.

1)
Варвара
говорит
Катерине,
что
я
не
умела
лгать,
но
научилась.

2)
Водитель
должен
помнить,
что,
повернув
с
проспекта

направо,
он
должен
будет
пересечь
трамвайные
пути.

3)
Зрители
следили
за
ходом
драматических
событий,

произошедших

в
начале
лета
1928
года.

4)
Путь,
который
избрал
герой,
ошибочен,
неправилен.

10.3.
Найдите

предложение

без
нарушения
синтаксических
норм.

1)
Адмирал
был
назначен
командующим

военно-морских

сил
США
на
Тихом

океане.

2)
Большинство

стихотворений

Цветаевой

пронизаны

лирикой
и

красотой
написания.

3)
Будучи реалистом,

писатель

всесторонне

показал

жизнь
крестьян

того

времени.

4)
Не
только
героизм
Корчагина

проявляется
во
время
войны,
в
бою,
но
и
в
мирные
дни.

10.4.
Выберите
грамматически
правильное
продолжение
предложения:

Отправляя
телеграмму,

1)
всегда
указывается
обратный
адрес.

2)
обязательно
укажите
обратный
адрес.

3)
без
обратного
адреса
её
не
примут.

4)
мне
не
хватило
денег.

10.5.
Найдите

предложение

без
нарушения
синтаксических
норм.

1)
Боясь
о
том,
что
цензура
не
пропустит,

поэт
не
печатал
это
стихотворение.

2)
Анализ
этого
исследования
был
не
очень
критический.

3)
Немало
и
тех,
кто
стали
инвалидами.

4)
Поклоняться
и
боготворить
Молчалина

вот
смысл
жизни
Софьи.

10.6.
Выберите
грамматически
правильное
продолжение
предложения:

Допуская
ошибки,

1)
шанс
поступить
в
университет
очень
мал.

2)
вам
может
помочь
словарь.

3)
не
стоит
рассчитывать
на
высокую
отметку.

4)
часто
не
применяются
правила
орфографии.

10.7.
Найдите

предложение

без
нарушения
синтаксических
норм.

1)
Пораженное
место
кислотой
или
щелочью
необходимо
быстро
и
осторожно

протереть
чистой
тряпкой,
бинтом
или
ватой.

2)
Преподаватель
сказал,
что
я
не
буду
сегодня
спрашивать.

3)
Из-за
пожара
был
уничтожен
большой
участок
леса.

4)
Буржуазные

журналисты

и

те,

кто

их

поддерживают,

пытаются

скрыть
правду.

11.
Орфографические
нормы
русского
литературного
языка

11.1.
В
каком
ряду
во
всех
словах
пропущена
одна
и
та
же
буква?

1)
пр..родители,
пр..медление,
раз..слать

2)
и..чезнуть,
во..горание,
не..гораемый

3)
об..ём,
ад..ютант,
кон..ячный

4)
пр..морский,
пр..украсить,
без
пр..крас

11.2.
В
каком
предложении
на
месте
пропуска
пишется
И
(НИ)?

1)
Путнику
для
дружеского

общения
с
местными
жителями
н..
нужны
никакие

особенные
премудрости.

2)
Перспектива
ждать
несколько
часов
в
душном

зале
аэровокзала
н..мало
нас
не
прельщала.

3)
Город
наш
вовсе
н..
отличался
архитектурными
достопримечательностями.

4)
Н..
эффектная
внешность
и
н..
поставленный
голос
делает
актёра
актёром.

11.3.
В
каком
предложении
НЕ
пишется
слитно?

1)
Многие
школьники
понятия
(не)имели
о
том,
что
звёзды,
так
же
как
и
солнце

и
луна,
передвигаются
по
небосводу.

2)
Известно,
что
взрослые
и
дети
часто
(не)допонимают
друг
друга.

3)
Если
вы
привыкли
бросать
дело,
(не)доводя
его
до
конца,

преодолеть
эту
привычку
очень
трудно.

4)
Основой
производства

на

Севере

было
(не)пашенное

земледелие,

как

в
центральных
районах
России,
а
вольное
предпринимательство.

11.4.
НН
пишется
на
месте
обоих
пропусков
в
предложении:

1)
Неожида…о
в
тишине
послышалась
соловьи…ая
трель.

2)
В
очерче…ом
радугой
полукруге
виднелся
силуэт
ветря…ой
мельницы.

3)
В
период
долгих
осе…их
дождей
на
душе
всегда
тревожно,
неопределё…о.

4)
После
редакцио…ого

совещания
так

и
не

было
определе…о

время

пресс-

конференции.

11.5.
Отметьте
глагол
с
суффиксом
-ЫВА-.

1)
совет..вать;

2)
доклад..вать;

3)
участв..вать;

4)
планир..вать.

11.6.
В
каком
случае
в
окончании
прилагательного
пропущена
буква
И?

1)
в
син..м
небе;

2)
осенн..е
утро;

3)
на
свеж..м
возухе;

4)
зимн..м
днём.

11.7.
В
каком
ряду
во
всех
трёх
словах
пропущена
буква
О?

1)
соприк..сновение,
комп..нент,
настор..житься;

2)
р..стовщический,
осн..cтить,
..реол;

3)
благотв..рительный,
в..кансия,
разб..лтать;

4)
р..вномерный,
пан..рама,
задр..жать.

12.
Пунктуационные
нормы
русского
литературного
языка

12.1.
Укажите
предложение
с
пунктуационной
ошибкой.

1)
Наша
телега
то
въезжала
в
сугроб,
то
проваливалась
в
яму.

2)
Всё:
топор,
ветки,
стволы
деревьев
было
покрыто
тонким
слоем
льда.

3)
Заря
бывает
не
только
утренняя,
но
и
вечерняя.

4)
Талант

писателя

позволяет

сочетать

правду

и

фантастику

и

заставляет
поверить
в
реальность
событий.

12.2.
В

каком

предложении

не

ставится

тире?

(Знаки

препинания

не
расставлены.)

1)
Снег
глубок
хлеб
хорош.

2)
Павел
сказал
Нам
это
не
нужно.

3)
Основная
цель
жизни
этого
человека
собственное
благополучие.

4)
Старое
сено
солома
почки
всё
порастает
зелёной
травой.

12.3.
Укажите

правильный
вариант
расстановки

запятых
в
предложении

«Правила (1) как известно
(2) создаются или
выбираются для
определённой
ситуации.
Со
временем
(3)
складывается

новая
ситуация,

при
которой прежняя теория
уже
не способна вести
вперёд и
(4)

следовательно
(5)
должна
быть
заменена».

1)
1,
2,
3,
5

2)
1,
2,
4,
5

3)
1,
3,
4

4)
2,
5

12.4.
Укажите

предложение,

в

котором

нужно

поставить
одну
запятую.
(Знаки
препинания
не
расставлены.)

1)
Иногда

один
и
тот
же

смысл
в
языке

может

быть
передан

как

формой

единственного
так
и
формой
множественного
числа.

2)
Под
прикрытием
берёзы
или
осины
ель
хорошо
растёт.

3)
Свежие
и
высушенные
плоды
инжира
равно
вкусны
и
полезны.

4)
Оленин
то
брался
за
книгу
то
выходил
на
крыльцо
то
ложился
на
постель.

12.5.
В
каком
варианте

ответа
правильно
указаны

все
цифры,
на
месте
которых

в

предложении

«Изредка

маленькая
снежинка

прилипала
снаружи
к
стеклу
(1)
и
(2)
если
пристально
вглядеться
(3)
то
можно
было
увидеть

её
тончайшее

кристаллическое

строение
(4)
и
поразиться
этому
чуду
природы»
должны
стоять
запятые?

1)
1,
3

2)
2,
3

3)
1,
2,
3,
4

4)
1,
4

12.6.
Укажите
предложение
с
пунктуационной
ошибкой.

1)
Эксперимент

требовал

от

участников,

как

специальных знаний,

так

и
постоянного
внимания.

2)
Я
прислушивался
к
звучанию
музыки
и
природы
и
брал
подсказанное
ими
в
основу
своего
творчества.

3)
Рокочущий
шум
моторов
то
отдалялся,
то
снова
приближался.

4)
Посаженные
выпускниками
деревья: клён,
акации,
берёзы – быстро
разрослись.

12.7.
В

каком

предложении

не

ставится

тире?

(Знаки

препинания

не
расставлены.)

1)
Природа
вечный
образец
искусства.

2)
Москва
есть
столица
нашей
Родины.

3)
Скрипнешь
дверью
зайцы
так
и
запрыгают
под
окнами
домика.

4)
В
траве
в
кустах
повсюду
заливались
цикады.

13.
Орфографическая
грамотность

13.1.
Орфографические ошибки допущены в
подчеркнутых словах в
предложении:

1)
Когда

выстраивается
шеренга

солдат,

то

впереди

становятся

рослые

и
бравые,

а
в
конце
всегда
бывает
самый
маленький.

2)
Мы
собирались
отправиться
в
путь
пораньше,
так
чтобы
пересечь
Каменную
пустошь
до
жары.

3)
Понастоящему

трудно
нам

стало,

когда

солнце

поднялось

повыше

и
в
несколько
минут
накалило
воздух,
разбудив
мириады
злобных
мошек.

4)
Мы
успели
вовремя
спрятаться
в
тень
леса,
как
раз
тогда,
когда
кровь
как

будто
закипала
в
жилах.

13.2.
Какой
ряд состоит из слов, в
которых пропущены только
проверяемые
ударением
гласные
корня?

1)
сокр…щать,
благосл…вить,
р…скошный;

2)
предв…рительно,
ск…кать,
изб…ратель;

3)
безотл…гательный,
отр…сль,
ди…гональ;

4)
предпол…жительно,
прор…стать,
п…норама.

13.3.
В
каком
ряду
в
обоих
словах
пропущена
буква
Е?

1)
зерно
мел..тся,
колебл..мый
ветром;

2)
возьмите
и
вынес..те
вещи,
рассматрива..мый;

3)
кто
замет..т
ошибку,

незамеч..нная
ошибка;

4)
малыш
плач..т,
невид..мый
объект.

13.4.
В
каком
ряду
во
всех
словах
пропущена
буква
Ю?

1)
се..щий,
(посевы)
выгор..т;

2)
самокле..щийся,
(собаки)
почу..т
(волка);

3)
волну..щийся,
(они)
вывес..т
(объявление);

4)
бор..щийся,
(они)
наде..тся.

13.5.
В
каком
предложении
оба
выделенных
слова
пишутся
слитно?

1)
ЧТО(БЫ)
воспитать

щенка,

необходимо

много
усилий,
ЗА(ТО)

сколько

радости
он
вам
доставит!

2)
Младший
сын
ТАК(ЖЕ)
любил
книги,
как
и
все
в
нашей
семье,
(ПО)ЭТОМУ

у
нас
не
возникло
сомнения
в
том,
что
подарить
ему
в
день
рождения.

3)
ЧТО(БЫ)
рыба

клевала

ТАК(ЖЕ)

хорошо,
как

вчера,

я
накопал

свежих
червей.

4)
Я
благодарю

Вас,

а

ТАК(ЖЕ)

Вашего

секретаря

ЗА(ТО),

что
мне

дали

возможность
изучить
этот
проект.

13.6.
В
каком
предложении
допущена
ошибка
в
правописании
частиц
НЕ
и

НИ?

1)
Что
ни
спроси,
обо
всём
он
знает.

2)
Чем
он
только
ни
занимался!

3)
Из
строя
он
не
уходил,
пока
не
иссякли
патроны.

4)
Не
одна
во
поле
дороженька
пролегала.

13.7.
Укажите
предложение,
в
котором
на
месте
пропуска
нужно
писать
И?

1)
В
лесу
я
был
н..
один.

2)
Куй
железо,
пока
н..
остыло.

3)
Мы
н..
раз
были
на
охоте.

4)
Он
всем
оказывал
помощь,
кто
бы
к
нему
н..
обратился.

14.
Пунктуационная
грамотность

14.1.
Подчеркнутые

слова
следует

обособить
в

предложениях…

(знаки
препинания
в
предложениях
не
расставлены):

1)
Огромные
ветви
старой
ели
качались
и
закрывали
усеянное
звёздами
небо.

2)
Высокопоэтичным
может
быть
слово
повседневное,
разговорное.

3)
Для
арабских

мыслителей

греческие

философские

тексты

утратив
свой
живой
и
непосредственный
характер
превратились
в
своего
рода
священные
книги.

4)
Среди
поэтов
Серебряного
века
А.
Блок
занимает
особое
положение.

14.2.
Постановка

двоеточия

в
предложении
«С
утра
прихворнул
Лукич:
покалывало

в
поясницу,
от
боли
глухой

ноги

сделались

чугунными,

к
земле
липли»
объясняется
тем,
что

1)
вторая
часть
бессоюзного

сложного
предложения

раскрывает

содержание

того,
о
чём
говорится
в
первой
части.

2)
вторая
часть
бессоюзного

сложного
предложения

указывает

на

следствие
того,
о
чём
говорится
в
первой
части.

3)
первая

часть

бессоюзного

сложного
предложения

указывает

на
время

совершения
того,
о
чём
говорится
во
второй
части.

4)
двоеточие
ставится
при
обобщающем
слове.

14.3.
Как

объяснить
постановку

тире

в
предложении
«Слой
облаков

был
очень
тонок

сквозь
него
просвечивало
солнце»?

1)
Отделяется
обособленное
приложение.

2)
Вторая

часть

бессоюзного

сложного
предложения

указывает

на

причину
того,
о
чём
говорилось
в
первой
части.

3)
Вторая
часть
бессоюзного

сложного
предложения

указывает

на
следствие
того,
о
чём
говорилось
в
первой
части.

4)
Тире
стоит
на
месте
пропуска
главного
члена
предложения.

14.4.
Укажите

правильное

объяснение

пунктуации

в

предложении

«Тишина
была
первозданная
(
) и
вспомнилось
майору
его
детство».

1)
Простое

предложение
с
однородными

членами,

перед
союзом

И
не
нужна

запятая.

2)
Простое

предложение

с
однородными

членами,

перед

союзом
И

нужна
запятая.

3)
Сложное
предложение,
перед
союзом
И
нужна
запятая.

4)
Сложное
предложение,
перед
союзом
И
не
нужна
запятая.

14.5.
Укажите

верное

объяснение

постановки

тире

в

предложении

«В
местной

команде

особенно

выделялись

двое

футболистов

один
из
нападающих
и
защитник».

1)
разделяется
подлежащее
и
сказуемое;

2)
после
обобщающего
слова
следуют
однородные
члены;

3)
выделяются
обособленные
приложения;

4)
разделяются
части
бессоюзного
сложного
предложения.

14.6.
В

каком

варианте
ответа

правильно

указаны

и

объяснены

все
запятые
в
предложении
«Благородные
пропорции
белого
храма
(1)
отражающиеся
больше
восьми
веков
в
водах
реки
(2)
естественно
вписываются
(3)
в
окружающий
его
(4)
пейзаж»?

1)
1,
3,
4

выделяются
причастные
обороты;

2)
1,
4

выделяются
причастные
обороты;

3)
1,
2

выделяется
причастный

оборот;

4)
1,
3

выделяется
деепричастный
оборот.

14.7.
В

каком

предложении

придаточную

часть

сложноподчинённого
предложения
нельзя
заменить
обособленным
определением,
выраженным
причастным
оборотом?

1)
Впечатления,

которые

мы
получаем

в
раннем

детстве,

оказывают

на

нас

огромное
влияние.

2)
В

небольшом

зале

два

тенора

пели

дуэт
из
«Пуританки»,

и
всё

вокруг
затоплялось

волнами

бархатных

звуков,
которые

лились
в

души,
опьяняя
южной
страстью.

3)
При
воспоминании

об
Аделине

Патти

я
переживаю

вновь
то
состояние,

которое
испытал,
слушая
её
колоратуру.

4)
У
почти
безголосого
тенора
Нодена,
который
тем
не
менее
считался
лучшим
вокалистом,
была
совершенно
поразительная
манера
пения.

15.
Грамматические
ошибки

15.1.
Укажите предложение
с грамматической ошибкой (неправильно
построенное).

1)
Те,
кто
ездил
в
трудовой
лагерь,
умеют
работать
по-настоящему.

2)
Некоторые
школьники

удостоены
медалью
«За
отвагу».

3)
В
списке
отличников,

помещённом
на
стенде,
моей
фамилии
не
было.

4)
В
«Науке
и
жизни»
помещают
много
полезных
советов.

15.2.
Определите
ряд,
в
котором
нет
грамматических
ошибок:

1)
уверенность
в
себе,
по
приезду
в
город;

2)
по
завершении
работы,
рецензия
о
книге;

3)
скучаю
по
вас,
заведующий
столовой;

4)
согласна
приказу,
приехать
с
города.

15.3.
Укажите предложение
с грамматической ошибкой (неправильно
построенное).

1)
Натянуть
и
выстрелить
из
лука
непросто.

2)
Тем,

кто
расположился
на
ночлег

под
навесом,
пришлось

искать

другого
укрытия.

3)
По
берегам
реки
Ангары
раскинулась
тайга.

4)
Руки
девушки,
загрубевшие
от
работы
на
холоде,
не
были
изящны.

15.4.
Укажите предложение
без грамматических ошибок (правильно
построенное).

1)
Гринёв
является
главным
героем
повести
«Капитанской
дочки».

2)
Гарибальди
встал
во
главе
людей,
сражающейся
за
независимость
Италии.

3)
Всем,
кто
был
на
субботнике,

объявили
благодарность.

4)
Учёные
сравнивают
и
наблюдают
за
жизнью
животных.

15.5.
Укажите предложение
с грамматической ошибкой (неправильно
построенное).

1)
Доктор
долго
ходил
по
лабиринту
улиц,
проложенных

без
всякого
плана.

2)
Те,
кто
пришли
последними,
остались
без
мест
и
стояли
в
проходах
зала.

3)
С
романом
«Обломов»
школьники
знакомятся
в
старших
классах.

4)
Бабушка
любила
единственного
внука
и
гордилась
им.

15.6.
Укажите предложение
без грамматических ошибок (правильно
построенное).

1)
Родители
спросили
меня,
что
не
хочу
ли
я
поступить
в
вуз.

2)
Все,
кто
знают
творчество
Чехова,
восхищаются
им.

3)
В
журнале
«Современник»
печатаются
публицистические
произведения.

4)
По
окончанию
школы
я
буду
поступать
в
институт.

15.7
Укажите
предложение
с
грамматической
ошибкой.

1)
Люди,
работающие
в
одном
коллективе,
должны
быть
внимательными
друг
к

другу.

2)
К.

Паустовский

утверждал,

что
голос
сердца
чаще

всего

мы
слышим
в
юности,
когда
ничто
не
приглушает
свежий
мир
наших
чувств.

3)
Те,

кто

приехал

в

Воронеж

в

феврале,

попал
на

вернисаж

художника

Голубовского,

пейзажиста
и
анималиста.

4)
Благодаря комментариям
Ю.
Лотмана к
«Евгению
Онегину»
каждый
читатель
романа
лучше
понимает
эпоху
Пушкина.

16.
Речевая
грамотность

16.1.
Отметьте

номер

слова,
лексическое

значение

которого

определено
неверно.

1)
арбитр

«тот,
кто
судит
спортивную
игру,
состязание»;

2)
вепрь

«дикая
свинья»;

3)
изморозь

«очень
мелкий
дождь»;

4)
корысть

«выгода,
польза
для
себя».

16.2.
Синонимом
слова
исподволь
является
слово/словосочетание

1)
не
по
своей
воле;

2)
постепенно;

3)
ненавязчиво;

4)
неустойчиво.

16.3.
Речевая
ошибка
допущена
в
предложении:

1)
Илья
Муромец
изображен
в
старинных
доспехах.

2)
Древние
греки
достигли
больших
культурных
успехов.

3)
Напиши
автобиографию.

4)
Большой
вклад
в
дело
победы
над
фашистами
дали
советские
матросы.

16.4.
Определите

предложение

без
речевых
ошибок:

1)
Закопчённое
копотью
окно
избушки

не
пропускало
даже
яркого
солнечного

света.

2)
Конструкторы
установили
сверхмощный
двигатель.

3)
Мы
столкнулись
с
фактором
незаконной
продажи
спиртных
напитков.

4)
Слово
представили
самому
молодому
участнику
конкурса.

16.5.
Отметьте

номер

слова,
лексическое

значение

которого

определено
неверно.

1)
планёр

«безмоторный
летательный
аппарат»;

2)
позёмка

«низовой

ветер
зимой,
а
также
снег,
переносимый
этим
ветром»;

3)
просека

«небольшой
лиственный
лес»;

4)
символ

«условный
знак».

16.6.
Речевая
ошибка
допущена
в
предложении:

1)
Больной
был
немедленно
госпитализирован.

2)
Все
ученики
хорошо
освоили
теорему
Пифагора.

3)
За
торт
можно
заплатить
в
первой
кассе.

4)
Победители
олимпиады
были
награждены
сувенирами.

16.7.
Определите

предложение

без
речевых
ошибок:

1)
Эти
стихи
я
очень
хорошо
выучил
назубок.

2)
Я
впервые
познакомился
с
ним
на
Урале.

3)
Премьера
этого
балета
состоялась
в
Риге.

4)
Пологий
берег
реки
зарос
густыми
зарослями.

Персональное собеседование один на один с каждым сотрудником какая речевая ошибка допущена

Плеоназмы

Плеоназм — это рече­вое изли­ше­ство, упо­треб­ле­ние соче­та­ния слов, в кото­ром смысл одно­го сло­ва уже зало­жен в зна­че­нии дру­го­го. Плеоназм в рус­ском язы­ке — это лек­си­че­ская ошибка.

В пись­мен­ной и раз­го­вор­ной речи незна­ние точ­но­го зна­че­ния сло­ва, осо­бен­но заим­ство­ван­но­го рус­ским язы­ком, часто сопро­вож­да­ет­ся допу­ще­ни­ем лек­си­че­ских оши­бок, одной из кото­рых явля­ет­ся плео­назм. В линг­ви­сти­ке этот тер­мин вос­хо­дит к гре­че­ско­му сло­ву pleonasmos, что бук­валь­но зна­чит «пере­из­бы­ток».

Что такое плеоназм в русском языке

В раз­ных сфе­рах нашей жиз­не­де­я­тель­но­сти нам часто встре­ча­ет­ся сло­во­со­че­та­ние «сер­вис­ное обслу­жи­ва­ние». Его упо­треб­ля­ют для обо­зна­че­ния каче­ствен­но­го обслу­жи­ва­ния. Давайте заду­ма­ем­ся, насколь­ко пра­во­мер­но объ­еди­ня­ют­ся эти сло­ва с точ­ки зре­ния лек­си­че­ских норм рус­ско­го лите­ра­тур­но­го языка?

Прилагательное «сер­вис­ное» обра­зо­ва­но от сло­ва «сер­вис», заим­ство­ван­но­го из англий­ско­го язы­ка, в кото­ром service зна­чит «быто­вое обслу­жи­ва­ние».

Значит, зна­че­ние сло­ва «сер­вис­ный» уже вхо­дит в семан­ти­ку рус­ско­го сло­ва «обслу­жи­ва­ние» и явля­ет­ся лиш­ним в этой паре лексем.

Рассмотрим сло­во­со­че­та­ние «меню блюд». Французское сло­во «меню» обо­зна­ча­ет «под­бор блюд для зав­тра­ка, обе­да и т. д.», а так­же «лист с переч­нем пред­ла­га­е­мых блюд, напит­ков в ресто­ране, кафе, сто­ло­вой». И в этом соче­та­нии слов допу­ще­на рече­вая ошиб­ка — плеоназм.

Как видим, эта ошиб­ка воз­ни­ка­ет тогда, когда гово­ря­щий или пишу­щий не вни­ка­ет в зна­че­ние слов или не зна­ет точ­но­го их зна­че­ния и вкрап­ля­ет в свою речь лиш­ние с точ­ки зре­ния смыс­ла слова.

Определение

Укажем, какое опре­де­ле­ние этой рече­вой ошиб­ке дает Википедия.

Таким обра­зом, плео­назм — это более широ­кое поня­тие, кото­рое вклю­ча­ет не толь­ко упо­треб­ле­ние отдель­ных лиш­них слов, а целых обо­ро­тов речи и даже фраз, кото­рые мож­но упро­стить или изъ­ять из тек­ста вообще.

Примеры плеоназмов

Чаще все­го ука­зан­ной рече­вой ошиб­кой стра­да­ют соче­та­ния при­ла­га­тель­ных с суще­стви­тель­ны­ми, при­чем зна­че­ние при­ла­га­тель­но­го дуб­ли­ру­ет смысл опре­де­ля­е­мо­го им слова:

В каче­стве глав­но­го сло­ва в соче­та­нии высту­па­ет гла­гол, в зна­че­нии кото­ро­го уже зало­жен смысл лиш­не­го слова:

  • упасть вниз;
  • под­прыг­нуть вверх;
  • впер­вые познакомиться;
  • вер­нуть­ся обратно;
  • сжать кулак;
  • импор­ти­ро­вать из-за рубежа;
  • гос­пи­та­ли­зи­ро­вать в стационар;
  • пре­ду­пре­дить заранее.

Плеоназм и тавтология. Отличия

В лек­си­ко­ло­гии раз­но­вид­но­стью плео­назма счи­та­ет­ся тав­то­ло­гия (греч. tauto «то же самое» + logos «сло­во»).

Тавтология — это непред­на­ме­рен­ное упо­треб­ле­ние одно­ко­рен­ных слов в сло­во­со­че­та­нии или в одной фра­зе, а так­же необос­но­ван­ный повтор одно­го и того же слова.

Все спортс­ме­ны долж­ны сгруп­пи­ро­вать­ся в неболь­шие груп­пы по трое.

Ему надо прыг­нуть пры­жок пря­мо сейчас.

Петр оза­да­чил всех сотруд­ни­ков этой труд­ной зада­чей .

Авторская речь — это речь авто­ра .

Исходя из того, что в плео­назме дуб­ли­ру­ет­ся смысл язы­ко­вых еди­ниц, состав­ля­ю­щих сло­во­со­че­та­ние, но лек­се­мы не явля­ют­ся одно­ко­рен­ны­ми, мож­но утвер­ждать, что плео­назм — это скры­тая смыс­ло­вая тавтология.

Наше сов­мест­ное сотруд­ни­че­ство было плодотворным.

Сотрудничество — это дей­ствие, рабо­та вме­сте, уча­стие в общем деле.

Плеоназм созда­ет­ся в речи, когда дру­гим сло­вом обо­зна­ча­ет­ся одно и то же поня­тие, уже назван­ное лек­се­мой, состав­ля­ю­щей с ним соче­та­ние или фразу.

Тавтология же явля­ет­ся рече­вой ошиб­кой, где явно упо­треб­ля­ют­ся одно­ко­рен­ные сло­ва, созда­ю­щие излиш­ний назой­ли­вый повтор лек­сем с оди­на­ко­вым или похо­жим смыс­лом. С этой точ­ки зре­ния тав­то­ло­гия — это откры­тое язы­ко­вое изли­ше­ство в речи.

Примеры тавтологии

  • спро­сить вопрос;
  • зара­бо­тан­ная зарплата;
  • про­лив­ной ливень;
  • зво­нок звонит;
  • дымит­ся дымом;
  • вновь воз­об­но­вить;
  • город­ской градоначальник.

Тавтология быва­ет оправ­да­на толь­ко в текстах, напи­сан­ных в официально-деловом или науч­ном сти­ле, где повтор одно­го и того же сло­ва необ­хо­дим по смыс­лу высказывания.

Размещение недоб­ро­ка­че­ствен­ной рекла­мы с дан­ным содер­жа­ни­ем в дан­ном месте дан­ным спо­со­бом запре­ще­но зако­но­да­тель­ством страны.

В отли­чие от плео­назма в поэ­ти­че­ском язы­ке тав­то­ло­гия исполь­зу­ет­ся как один из видов повто­ров, уси­ли­ва­ю­щих эмо­ци­о­наль­ность и выра­зи­тель­ность речи. Повторяются либо одно­род­ные по сво­е­му зву­ча­нию и по смыс­лу сло­ва (гре­ет — погре­ва­ет, веет — пове­ва­ет), либо повто­ря­ют­ся сло­ва, раз­ные по зву­ча­нию, но близ­кие по смыс­лу (зна­ет — веда­ет, пла­чет — тужит, море-океан, тоска-печаль).

Если в риф­ме повто­ря­ет­ся одно и то же сло­во в изме­нён­ном его зна­че­нии, такую риф­му назы­ва­ют тавтологической:

Вот на берег вышли гости,
Царь Салтан зовёт их в гости.

А. С. Пушкин. Сказка о царе Салтане

Словарик плеоназмов

Персональное собеседование один на один с каждым сотрудником какая речевая ошибка допущена

Количество участников, получивших 1 балл
(всего 93234 участника)

Процент участников, получивших 1 балл

Задание 1. Чтение вслух

Интонация соответствует/ не соответствует пунктуационному оформлению текста

Темп чтения соответствует/ не соответствует коммуникативной задаче

Задание 2. Пересказ текста с включением высказывания

Все основные микротемы исходного текста сохранены/упущена или добавлена микротема

Фактических ошибок нет/ допущены фактические ошибки

Высказывание включено в текст уместно, логично/ не включено или приведено неуместно и нелогично

Ошибок при цитировании нет/ есть ошибки при цитировании

Грамотность речи (задания 1 и 2)

Грамматических ошибок нет/ допущены грамматические ошибки

Орфоэпических ошибок нет, или допущено не более 1 орфоэпической ошибки (исключая слово в тексте с поставленным ударением)/ допущены 2 и более орфоэпические ошибки

Речевых ошибок нет, или допущено не более 3 речевых ошибок/ допущены 4 и более речевых ошибок

Искажений слов нет/ допущены искажения слов

Задание 3. Монолог

Приведено 10 или более фраз по теме высказывания без фактических ошибок / приведено менее 10 фраз, и/или допущены фактические ошибки

Речевая ситуация учтена/ речевая ситуация не учтена

Высказывание характеризуется смысловой цельностью, речевой связностью и последовательностью, логикой изложения/высказывание не логично, изложение непоследовательно, допущены логические ошибки

Задание 4. Диалог

Даны ответы на все вопросы/ ответы не даны или даны односложные ответы

Речевая ситуация учтена/ речевая ситуация не учтена

Грамотность речи (задания 3 и 4)

Грамматических ошибок нет/ допущены

Орфоэпических ошибок нет, или допущено не более 2-х ошибок/ допущено 3 или более орфоэпические ошибки

Речевых ошибок нет, или допущено не более 3 речевых ошибок/ допущены 4 или более речевые ошибки

Речь отличается богатством и точностью словаря, используются разнообразные синтаксические конструкции/ отличается бедностью и/ или неточностью словаря, используются однотипные синтаксические конструкции

Как максимально эффективно и правильно провести собеседование

Не только от соискателя, но и от работодателя проведение собеседования требует огромной предварительной подготовки. Первым делом необходимо определиться с тем, какие требования выдвигать кандидату на вакантную должность. Кроме этого, следует учесть множество факторов, к примеру, какие ошибки могут возникнуть при приеме на работу и как правильно провести собеседование.

Зачастую при наборе кандидатов на должность совершается ряд ошибок, которые ведут к тому, что вместо креативных сотрудников можно принять хороших исполнителей, которые будут отлично выполнять свою работу, но не смогут сориентироваться при модернизации фирмы. Так, если станет необходимым поиск новых путей развития компании, то такие сотрудники не смогут даже внести какую-либо идею. Таким образом, следует учитывать неординарность мышления кандидата и его готовность посвятить себя работе.

Классические ошибки подбора персонала

Талант производить качественный набор сотрудников имеет каждый менеджер. Но чтобы действительно не прогадать на «отборочном туре», необходимо проявить смекалку. Первым делом нужно знать самые распространенные ошибки менеджера при отборе сотрудников, чтобы их избежать:

Требования к образованию

Чаще всего фирмой выдвигаются требования, чтобы сотрудник не просто имел высшее образование по специальности, но и мог представить подтверждение начального рабочего стажа (не менее трех лет). Таким образом, самая распространенная ошибка – брать специалистов с опытом работы и пренебрегать свежими идеями, которые может дать молодой специалист, не обремененный чужим опытом и правилами. Такие люди способны рисковать, что зачастую может привести компанию в выигрыш.

К примеру, выгодная сделка может состояться не только с помощью традиционного подхода: современные управители (главы корпораций) зачастую ищут неординарных людей, которые могут представить проекты, качественно отличающиеся от традиционных программ.

Личностные качества

К сожалению, во время приема на работу учитываются только те качества характера, которые могут пригодиться на работе, при этом под ними чаще всего подразумеваются трудолюбие, прилежность, пунктуальность, умение не пререкаться с начальством и без лишних вопросов выполнять свою работу. При этом в расчет зачастую не берутся такие стороны личности, как коммуникабельность, умение найти альтернативный подход, поиски необычных путей решения проблем фирмы, креативность.

Специальные требования

Чаще всего проводится отбор кандидатов. Специальными требованиями могут быть возрастные ограничения или набор по половому признаку. Также могут учитываться рекомендации с предыдущего места работы, периодичность смены места работы и причины увольнения (если человек часто меняет работу по собственному желанию, это может указывать на его непостоянство и ненадежность как сотрудника).

Зачем необходимо собеседование?

Основное значение собеседования – определить подходящих кандидатов, которые смогут полностью соответствовать вакантной должности.

Только в таком случае можно быть уверенным, что новый сотрудник полностью справится с возложенными на него задачами. Пример собеседования при поборе кадров можно представить в виде такой инструкции:

  1. Приветствие. Первым делом необходимо проверить, тот ли человек перед вами, который присылал свое резюме и подавал заявку на вакансию (бывают случаи жульничества, когда приходил другой человек и замечательно справлялся со всеми задачами, которые перед ним ставит комиссия). Постарайтесь сделать все, чтобы кандидат как можно быстрее адаптировался. Для этого достаточно рукопожатия и пары предложений о погоде.
  2. Введите претендента на должность в курс дела, поведайте цель собеседования. Если на собеседовании заседает комиссия, важно объяснить ее роль.
  3. Начинайте опрос кандидата. Следите, чтобы каждый следующий вопрос был сложнее предыдущего. В конце можно задавать открытые вопросы.
  4. Будьте объективны в суждениях. Старайтесь дать возможность претенденту высказаться. Проявляйте гибкость: при необходимости можно отказаться от некоторых вопросов в пользу развития интересной темы. Также важным является поддержание зрительного контакта.
  5. Кратко расскажите о работе и должности: зарплата, отпуск, график.
  6. Прежде чем завершить собеседование, необходимо поинтересоваться о том, есть ли вопросы у соискателя. Скажите спасибо человеку за встречу и сообщите о сроках решения по вакансии. Не следует прямо на собеседовании решать, принимать человека или нет. Информацию необходимо проверить и проанализировать. Если решение будет принято впопыхах, то при отказе в дальнейшем могут возникнуть проблемы, поскольку соискатель спокойно может передать дело на рассмотрение в суд.

Правила

Чтобы первая встреча с кандидатом прошла успешно и результативно для обеих сторон, нужно выполнять основные правила проведения собеседования:

  • Собеседование проводится не со всеми кандидатами, а только с теми, кто был отобран в результате отсеивания.
  • Никогда не пытайтесь подсказывать кандидату или наталкивать его на мысль, поскольку таким образом невозможно четко оценить знания и реальные навыки кандидата.
  • Если вы поняли, что кандидат не подходит, просто заканчивайте собеседование и не тратьте времени впустую.
  • Если вы поняли, что данному человеку лучше работать на другой должности в вашей фирме – предложите ему вакансию.
  • После окончания собеседования можете огласить оценку качеств каждого участника. Их можно письменно проинформировать (представить ксерокопию оценки комиссии). Это поможет им в дальнейшем проработать ошибки и добиться успеха в другой фирме.

Варианты проведения собеседования

Собеседование можно проводить несколькими способами:

Собеседование «один на один»

Это классический вид общения, когда с кандидатом наедине общается начальник, администратор, представитель отдела кадров или другое доверенное лицо компании. При таком виде собеседования можно без лишних стрессов поговорить с кандидатом. При этом обстановка будет располагать к откровенному общению, сотрудник не будет излишне переживать и сможет проявить себя с лучшей стороны. Такой способ позволит сложить правильное впечатление о человеке и проверить его профессиональные качества.

Встреча нескольких кандидатов с менеджером (доверенным лицом компании)

Подобный вид собеседования имеет свои недостатки и преимущества. К примеру, если вам нужно узнать, кто более стрессоустойчив и умеет вести борьбу с конкурентами, то такой способ – лучший вариант для сравнения кандидатов и их умения проявить себя, выделиться среди толпы. При таком собеседовании можно дать возможность посоревноваться в профессионализме: пусть кандидаты задают друг другу каверзные вопросы по специальности. Такая игра позволит увидеть, кто из них наиболее готов к должности.

Собеседование, при котором один кандидат общается с несколькими представителями фирмы

Для таких случаев собирается комиссия, члены которой будут задавать вопросы кандидату. Данный способ позволит не только узнать профессиональные качества и личностные особенности опрашиваемого, но и проверить его стрессоустойчивость.

Встреча нескольких кандидатов с отборочной комиссией

Подобные методы проведения собеседования позволяют провести сравнительную характеристику всех качеств кандидатов. При этом оценка будет выставляться непредвзятая, так как все качества будут взвешиваться всеми членами комиссии и по каждому кандидату будет высказано общее мнение.

Этот способ проведения собеседования требует определенных расходов фирмы. Так, руководитель должен одобрить заседание комиссии, выделить помещение и некоторые ресурсы. Также должен быть выписан приказ на разрешение заниматься членам комиссии отбором кандидатов вместо выполнения своей работы. Необходимо также решить много бюрократических моментов в виде распоряжений относительно сбора комиссии.

Принцип отбора кандидатов

После проведения различных методов отбора персонала: анкетирования, собеседования, ознакомления с резюме и других подводится качественная оценка каждого кандидата. При этом учитываются его ответы на собеседовании, а также вся информация, отображенная в резюме и анкетных данных. За время общения с кандидатом проводится сравнительная характеристика его личностных качеств. Это позволяет при дальнейшей работе знать, чего ожидать от такого работника.

Чаще всего некоторые личностные качества могут быть в приоритете в зависимости от вакантной должности. Помимо личностных качеств, существует также необходимость проверки профессиональных навыков и теоретических знаний. Только по прохождении всех видов отбора можно будет окончательно решить, подходит ли кандидат на вакантную должность или нет.

Вариант 1

1. Расставьте ударение в приведенных словах и выберите одно лишнее (с акцентологической точки зрения) слово из четырех. 1. Дояр, эксперт, яслей, столяр. 2. Начать, торта (род. п., ед. ч.), пасквиль, щавеля. 3. Диалог, средствами, недуг, некролог. 4. Уведомить, углубить, понять, начать. 5. Цепочка, отрочество, памятуя, шарфы. 6. Донельзя, генезис, добыча, завидно.

2. Мягко или твердо произносятся в современном литературном языке согласные перед звуком [э] в следующих заимствованных словах? В каких случаях допускается вариантное произношение? Термин, термос, текст, фланель, энергия, эффект, бизнес, бизнесмен, демагог, музей, пресса, тенденция, термин, террор, форель, шедевр, юриспруденция, агрессия, академик, альтернатива, компьютер, рельс.

3. В какой из этих пяти фраз нет плеоназма (употребления лишних слов, ничего не добавляющих к смыслу высказывания)? Объясните, в чем состоит лексическая избыточность остальных предложений. 1. На заводе хромает трудовая дисциплина. 2. Наука — наш главный приоритет. 3. Цены постоянно растут вверх. 4. Халатность инкриминируется ему в вину. 5. Да простят меня за избитую банальность!

4. Определите средства выражения экспрессивно-эмоциональной окраски в следующих словах. Городишко, дождинка, лисонька, уста, мальчонка, мамочка, хвостист, голубушка, пылища, бедненький, книжка, говорливый, пальчик, деляга, тихоня, око, беззаветный, беспримерный, прах, скорбеть, премудрый, чело, ноженька, работенка, трусишка, зайка, соловушка, водичка, доиграться, забегаться, призадуматься.

5. Найдите и исправьте ошибки, нарушающие лексические нормы. 1. Самое лучшее для таких больных – это укол смертельной инъекции. 2. Первое боевое крещение дедушка принял на Западном фронте в составе первого гвардейского кавалерийского, который непосредственно участвовал в обороне Москвы и освобождении важных коммуникаций западнее столицы. 3. Новый дебютный альбом Юлии Савичевой «Высоко…». 4. Зарплата у служащих должна быть небедной. 5. Самая большая глобальная печаль – это то, что ни в одной коллекции не выдержана цветовая гамма. 6. Он изо всех сил старается произвести на Софью внимание. 7. Персональное собеседование с каждым сотрудником один на один.

6. Объясните и исправьте ошибки, связанные с нарушением морфологических норм. 1. Я сейчас картошки почистю. 2. Ну, вот, дом теперь убратый. 3. Сдавайте польта в гардероб. 4. Лук, обдатый кипятком… 5. Я, конечно, извиняюсь, но вы здесь не стояли, когда я пришла, так что за мной будете. 6. Ну, возьми тюль и постирай в машинке. Она такая грязная. 7. В последнее время я стал читать книги более внимательнее. 8. В вашем учебнике нет семиста восьмидесяти пятой страницы. 9. Боюсь, что я ощущу неловкость. 10. А почему нам документы не ворочают?

7. Укажите, какая из двух параллельных форм вам представляется правильной. Если оба варианта, по вашему мнению, употребительны в современном литературном языке, подчеркните и тот, и другой и укажите, есть ли различие (какое?) в их использовании. Базироваться на достоверных фактах – базироваться на достоверные факты; вульгарное арго – вульгарный арго; директоры – директора; единственный – единственен; Иван – большая умница или Иван – большой умница; мать с дочерью не могла уснуть – мать с дочерью не могли уснуть; не любить абстрактное искусство – не любить абстрактного искусства; новая туфля – новый туфель; повары – повара; по десяти франков – по десять франков; работы Базилюка Ивана – работы Базилюк Ивана; рецензия на книгу – рецензия о книге; самый ближайший – самый близкий; сироп приятен на вкус – сироп приятный на вкус; старая плащ-палатка – старый плащ-палатка; сто гектаров – сто гектар; торты – торта; у озера Балатон – у озера Балатона; четверо работниц – четыре работницы; робеть перед экзаменом – робеть экзамена; самый юный – наиболее юный; согласно приказа – согласно приказу; ряд сотрудников направлен – ряд сотрудников направлены; удалиться друзей – удалиться от друзей; у нее – у ней.

8. Найдите неверные способы образования грамматических форм и отредактируйте высказывания. 1. Она более моложе, чем все остальные. 2. Его местов я не занимаю и никаких делов у меня с ним нету. 3. Все были готовые к походу. 4. Морозец щипает щеки, метет поземка. 5. Путь, который избрал герой, ошибочен, неправильный. 6. На занятиях не было обоих сестер. 7. Сарай был крыт толью, а на окнах дома висела красивая тюль. 8. И оно никогда не уйдет, не стерется из нашей памяти. 9. Он уехал не позавтракавши.

9. Исправьте ошибки, связанные с использованием деепричастного оборота. 1. Раскопав ямку до дна, на солнце заблестели голубые камни. 2. Открыв окно, запахло соснами. 3. Подъезжая к городу, начался сильный ветер. 4. Закончив экскурсию, в ресторане нас ждал обед. 5. Проснувшись, ему сказали, что завтрак уже подан.

10. Укажите ошибки в управлении. Предложите возможные варианты правки. 1. Не обязательно снимать людей – домики, животные и цветы тоже можно. 2. Данный текст относится к официально-деловому стилю: он регламентирует о правах читателя. 3. Сегодня работали за компьютерами, а я с ними никогда не пользовалась. 4. Товарищи, оплатите за проезд. 5. Ты зачем ему дал этих книг?

11. Исправьте ошибки в построении предложений. 1. Поезд потерпел крушение благодаря небрежности стрелочника. 2. Роман «Отцы и дети» написаны Тургеневым в 1862 году. 3. За счет неполадок в работе мотора произошла автокатастрофа. 4. И. Эйгес полагал, что музыка вообще занимала большое место в творчестве Чехова и романсы в частности. 5. Согласно распоряжения мэра города, пострадавшим будет выплачена компенсация. 6. В основном их интересовало возможность торговать на иркутском рынке и лечение в больницах областного подчинения. 7. Все его родственники живут в Украине, а это по нашим временам уже другая страна. 8. Такого простора для ног мало кто предлагает из проектировщиков машин бизнес-класса. 9. Я сегодня не смогу: мы будем в концерте. 10. Вероятно, вы ее найдете возле памятника Ленина.

12. Ответьте на поставленные вопросы, используя фразеологические обороты (например, «Как говорят о том, кого нетрудно уговорить сделать что-либо, пойти куда-либо?» — «Лёгок на подъём»). Как говорят 1) о том, кто часто меняет свои решения; 2) о том, кто вдруг стал замечать, понимать что-либо; 3) о человеке, который пришел не вовремя, некстати; 4) о кротком, безобидном человеке; 5) о человеке, который держится неестественно прямо; 6) о болтливом человеке; 7) о бесследном исчезновении чего-либо; 8) о положении, в котором опасность грозит со всех сторон; 9) об усердно трудящемся человеке.

13. Докажите с помощью примеров из приведенного ниже текста (автор — В. А. Барабанщиков) его принадлежность к научному стилю; определите, в рамках какого подстиля он составлен, и аргументируйте свою точку зрения. Выделите языковые средства, с помощью которых достигается логичность изложения, укажите на лексические единицы, грамматические формы и синтаксические конструкции, свойственные научной речи.

На первый взгляд, между С. Л. Рубинштейном и Б. Ф. Ломовым мало общего: они принадлежат к разным поколениям, разным научным школам, занимались разработкой разных проблем, играли в развитии отечественной психологии различную роль. Вместе с тем, более внимательный анализ обнаруживает между ними глубокую внутреннюю связь. Во-первых, Рубинштейн и Ломов были яркими представителями смежных микроэпох развития отечественной науки: если поколение Рубинштейна разрабатывало основание, или фундамент новой психологии, то поколение Ломова возводило нижние этажи самого здания. Во-вторых, существует известная близость школ С. Л. Рубинштейна и Б. Г. Ананьева, к которой принадлежал Б. Ф. Ломов, а значит сходство развиваемых идей и подходов. В-третьих, и Рубинштейн, и Ломов в разные исторические периоды стояли перед необходимостью методологически осмыслить современное им состояние психологии, отразить тенденции и наметить пути ее развития. В-четвертых. Рубинштейна и Ломова роднит решение одной и той же научно-организационной задачи — создание в Академии Наук сильного психологического исследовательского центра. Открыв в 1971 году Институт психологии, Ломов, по существу, завершил работу, начатую Рубинштейном в 40-е годы. В-пятых, и Рубинштейн, и Ломов придерживались коллегиального способа исследовательской деятельности, собирая воедино и опираясь на все полезное и ценное, что сделано в мировой и отечественной науке. Наконец, в-шестых, в чисто человеческом плане их объединяет редкая способность брать на себя полноту ответственности за судьбы вверенных им людей. Добавлю, что ни Рубинштейн, ни Ломов не были кабинетными учеными, оторванными от событий реальной жизни; они всегда занимали ясную гражданскую позицию, принимая самое активное участие в построении современной им психологической науки, образования и практики. Вполне закономерным представляется также тот факт, что ученики и соратники Сергея Леонидовича вошли в состав созданного Б. Ф. Ломовым Института психологии и, опираясь на глубоко эвристичную концепцию Рубинштейна, успешно разрабатывают вопросы теории и методологии психологической науки, проблемы психологии познания, личности, социальной психологии и многие другие.

Б. Ф. Ломов неплохо знал труды Сергея Леонидовича, особенно «Основы общей психологии» (1946) и «Принципы и пути развития психологии» (1959). Наряду с Б. Г. Ананьевым и П. К. Анохиным Рубинштейн входил в группу авторов, на которых Ломов (1984) ссылался наиболее часто и идеи которых пытался конструктивно использовать и развивать.

В данной работе я кратко рассмотрю содержание психологических концепций Рубинштейна и Ломова, сделав акцент на главных линиях их соприкосновения, тех, которые характеризуют преемственность научных традиций.

Прежде всего, это глубокий интерес к проблеме человека, который устойчиво воспроизводится в российской науке и общественно-политической мысли с середины XIX века. Очевидно, что в психологии то или иное понимание человека — «альфа» и «омега» любых конкретных исследований.

Данная проблема разрабатывалась Рубинштейном на философско-методологическом уровне в терминах соотношения человека как реального практического существа и его бытия, и имела не только мировоззренческий, но и нравственный смысл. Отношения человека к бытию полагались как объективные, а сам человек выступал в качестве субъекта разнообразных форм и проявлений жизни. Поскольку специфика человеческого бытия виделась в общественном способе существования, в качестве центрального выделялось отношение человека к другим людям. Тем самым открывалась возможность наполнения понятия «человек» гуманистическим содержанием.

Предложенная концепция задавала принципиально новые пути конкретно-психологического исследования (во многом пока еще не пройденные позитивной наукой), а также определяла способ интеграции наук об обществе, природе и мышлении. Отмечу, что с проблемой интеграции, или синтеза гуманитарных и естественных наук Рубинштейн столкнулся еще в студенческие годы, обучаясь философии в Марбурге (эта проблема была центральной и у его знаменитых учителей — Г. Когена и П. Наторпа), и перманентно возвращался к ней на протяжении всего творчества, В частности, методологический принцип единства сознания и деятельности, разработанный в 30-е годы, предназначался в том числе и для организации разнородного психологического знания.

Б. Ф. Ломов рассматривал проблему человека в ином ключе: через призму позитивных наук, нередко в связи с решением практических задач. Согласно Ломову, феномен человека выражает единство законов природы и общества и в этом качестве является уникальным объектом исследования. Выявление основных свойств и отношений человека, закономерностей его организации и развития рассматривались Борисом Федоровичем в качестве важнейшей задачи научного познания. И антропоцентрический подход к анализу систем «человек — техника», и понятие «активного оператора», и шаги по активизации так называемого «человеческого фактора» — все это разные формы выражения гуманистической направленности его исследований. Однако, двигаясь по этому пути, исследователь рано или поздно оказывался перед проблемой места психологии в системе наук о человеке и связанной с ней проблемой синтеза знаний.

14. Исправьте максимально возможное количество ошибок в приведенном ниже стихотворении. Поставьте ударение в словах, выделенных полужирным курсивом.

Хоть я и не русский очень, но хочу любви.

Положил документ в портфель – стырили.

Сажаюсь на тубаретку, отложив дела.

Почто ты мне звонишь редко, где была ты?

Не понял, объясни, я ведь подросток в любви,

Как мне начать с тобой. Ты углубила мне боль:

Ножницами наскрозь режешь сердце мне до слёз,

Вот уже много дня не схожу я с билютня.

Я мышленье напрягаю, звоню в телефон,

Ложу тебе письма в ящик: полон он.

Тебе подарю я арбуз, грейфруктов куплю,

Кружовника насажаю – я тебя люблю.

Прецендент я создал, за транваем побежал,

Посклизнулся в снегу, поломал себе ногу.

И теперь я в пальте на инвалидном месте,

С костылем и в гипсу еду в лесополосу.

Там живешь ты в дому, адрес я храню в уму:

Квартал нужно пройтить, библиотеку найтить.

Мне достал родный брат зарубёжный мирмалат –

Прямо тает в роту, открывай, я накладу.

В колидор выходи, раболант моей любви,

Не застойся в дверях, жгёт огонь меня в нутрях.

Ты придёшь, я скажу, что словов не нахожу:

Так могуч, так велик этот русский твой язык.

ПРИ ВЫПОЛНЕНИИ КОНТРОЛЬНОЙ РАБОТЫ, А ТАКЖЕ ПРИ ПОДГОТОВКЕ К ЭКЗАМЕНУ (С ЦЕЛЬЮ ЕГО НЕЗАТРУДНИТЕЛЬНОЙ СДАЧИ) ПОЛЬЗУЙТЕСЬ СЛЕДУЮЩЕЙ ЛИТЕРАТУРОЙ:

1. Аванесов литературное произношение. М., 1984.

2. , Зарва ударений русского языка. М., 1993.

3. Введенская и искусство речи. Ростов-на-Дону, 1996.

4. , , Кашаева язык и культура речи. Ростов-на-Дону, 2003.

5. Головин культуры речи. М., 1980.

6. Голуб русского языка. М, 2001.

7. , , Ягубова язык и культура речи: Учебник для студентов-нефилологов / Под редакцией д. ф. н., проф. О. Б. Сиротининой. Саратов, 2001.

8. Горбачевич современного русского литературного языка. М., 1978.

9. К, , Катлинская правильность русской речи. Стилистический словарь вариантов. М., 1976.

10. Культура русской речи / Под ред. проф. и проф. . М., 1998.

11. , Шведова словарь русского языка. М., 1995.

12. Орфоэпический словарь русского языка. М., 2001.

13. Плещенко и культура речи: Учеб. Пособие / Т. П. Плещенко, , ; Под ред. . ¾ Мн.: «ТетраСистемс», 2001.

14. Розенталь стилистика русского языка. М., 1987.

15. Русский язык и культура речи: Учебно-методический комплекс для вузов / Под ред. доц. . Кемерово, 2002.

16. Русский язык и культура речи / Под ред. проф. . М., 2002.

17. Русский язык и культура речи / Под ред. проф. . М., 2000.

18. Русский язык и культура речи: Конспект лекций / , , Е. В Ваганова и др. – Челябинск: Изд-во ЮУрГУ, 2003.

1. Расставьте ударение в приведенных словах и выберите одно лишнее (с акцентологической точки зрения) слово из четырех. 1. Некролог, неприязнь, мытарство, духовник. 2. Банты, хвоя, торты, столяр. 3. Христианин, танцовщик, простолюдин, статут. 4. Жалюзи, догмат, средства, апостроф. 5. Свекла, километр, баловать, шасси. 6. Нефтепровод, туфля, каталог, эксперт.

2. Мягко или твердо произносятся в современном литературном языке согласные перед звуком [э] в следующих заимствованных словах? Бактерия, безе, деликатес, декан, депрессия, детектив, интервью, кофе, кодекс, кратер, кредо, крейсер, критерий, модель, нетто, одесский, орхидея, патент, сессия, свитер, синтез, темп, тент, терапевт.

3. В какой из этих пяти фраз нет плеоназма (употребления лишних слов, ничего не добавляющих к смыслу высказывания)? Объясните, в чем состоит лексическая избыточность остальных предложений. 1. Развод – это трагическая катастрофа для всей семьи. 2. Все бюджетники получат дополнительную надбавку к зарплате в связи с праздниками. 3. Власти ведут борьбу с незаконным оборотом наркотиков. 4. Семёнов неожиданно наткнулся на непредвиденные трудности. 5. Его диссертация посвящена натурфилософии природы.

4. Распределите книжную лексику по стилистическим группам (по сферам употребления): 1) лексика официально-деловая; 2) научная, 3) техническая, 4) публицистическая. 5) поэтическая; назовите ситуации, где может уместно употребляться каждая группа. Истец, славный, вышеуказанный, концепт, завет, константа, пакт, коварство, ореол, нижеподписавшиеся, вещий, препроводить, лазер, виртуальный, аккумулятор, осужденный, локальный, интервью, барокамера, приговор, макроэкономика, шествовать.

5. Исправьте ошибки, связанные с употреблением паронимов. 1. В зале ожидания находилось много командировочных. 2. Преподаватель представил студентам полную свободу в выборе темы для сочинения. 3. Наши воины совершили много геройских подвигов. 4. Поэт стоял у источников новой поэзии. 5. Писатель завоевал мировую признательность. 6. Члены делегации зашли по трапу в самолет.

6. Объясните и исправьте ошибки в образовании грамматических форм слов. 1. Их нужно соединить, а потом посмотреть, как они сочетутся. 2. Я в этом году в Сочах была, там так красиво! 3. По тому, как живет цветочек, можно определить, как к тебе относятся. Если начнет вять … сама понимаешь. 4. Я хожу без носок, босиком. 5. Сколько же платий тебе нужно? 6. Сколько доль наследства ты получил? 7. Только почетные гражданины города у нас ездят бесплатно. 8. Я обожаю романтическую музыку, поэтому мои любимые песни — это песни Элтон Джона и Джордж Майкла. 9. В той квартире, где она сейчас живет, раньше цыгане жили. И она после этих цыганей нашла у себя под половицей их карты.

7. Укажите, какая из двух параллельных форм вам представляется правильной. Если оба варианта, по вашему мнению, употребительны в современном литературном языке, подчеркните и тот, и другой и укажите, есть ли различие (какое?) в их использовании. Весь в пушистом снеге – весь в пушистом снегу; выговоры – выговора; живописный Капри – живописное Капри – живописные Капри; жираф – жирафа; из полутора метров – из полтора метров; купи творога – купи творогу – купи творог; лип – липнул; нет время – нет времени; обеих девочек – обоих девочек; пачка табака – пачка табаку; полощет – полоскает; пять килограмм – пять килограммов; Петя – такая умница или Петя – такой умница; работники ЖЭК – работники ЖЭКа; руководить молодого человека – руководить молодым человеком; с Иваном Гумберт – с Иваном Гумбертом; старый мозоль – старая мозоль; уделить внимание на этот вопрос – уделить внимание этому вопросу; чернило – чернила; кресло-кровать стояло – кресло-кровать стояла; множество людей любовались – множество людей любовалось; не бойся – не бойсь – не боись; не терять надежды – не терять надежду; острая боль – острый боль; под Бородином – под Бородиным; старая плащ-палатка – старый плащ-палатка; старинное бра – старинный бра; тарелка борща – тарелка борщу; удивлен поступком – удивлен поступку; я с сыновьями уеду – я с сыновьями уедем.

8. Допишите окончания и объясните свой выбор. 1. В зоопарке живет маленьк. кенгуру. 2. Детеныш выглядывал из сумки больш. кенгуру. 3. Какаду отложил. яйца и высиживал. птенцов. 4. В соседней клетке сидел. какаду. 5. Шуми. многолюдн. Осло. На главн. авеню – уютн. кафе «Розов. фламинго». В разнообразн. меню есть бутерброды с аппетитн. салями, салат из свеж. кольраби, черн. кофе, абрикосов. суфле, крепк. виски. 6. Крупнейш. цунами обрушил. сь на берег. 7. Эсперанто был… создан. доктором Л. Заменгофом.

9. Укажите ошибки в управлении. Предложите возможные варианты правки. 1. Начался обмен мнений. 2. Мы восторгались красоте летнего заката. 3. Я очень рада решением суда: все получилось так, как я хотела. 4. Дорогие друзья, Святославу Ещенко сегодня день рождения! 5. Он всегда удивлялся трудолюбием сына.

10. Исправьте ошибки, связанные с использованием деепричастного оборота. 1. Родившись в такой кинематографической семье, Наташино будущее было, в общем-то, предопределено. 2. Посмотрев этот фильм, мне стало очень грустно. 3. Придя домой, радио играло на всю громкость. 4. Выйдя из квартиры, у него появились слезы. 5. Приехав домой, мне стало значительно лучше.

11. Исправьте ошибки в построении предложений. 1. Полиция нередко безучастно созерцала на разгорающийся конфликт. 2. Вследствие предстоящей защиты я пятую ночь не сплю. 3. Крестьянство выступали против помещиков. 4. Прошу предоставить отпуск ввиду болезни. 5. Это же молодое поколение, которое растут. 6. Во многих странах мира действует правило, согласно которого все более-менее значительные объемы внешних заимствований отдельных рыночных субъектов данной страны гарантируются в той или иной степени государством. 7. Нашла у кого советоваться. 8. Благодаря его мы все успели вовремя. 9. Сегодня на меню картошка с котлетами. 10. Ты во сколько сегодня пришёл со школы?

12. Ответьте на поставленные вопросы, используя фразеологические обороты (например, «Как говорят о том, кого нетрудно уговорить сделать что-либо, пойти куда-либо?» — «Лёгок на подъём»). Как говорят 1) о чем-нибудь надоевшем, много раз слышанном; 2) об очень дальних родственниках; 3) о человеке, от которого ожидают успеха в какой-либо деятельности 4) о неразговорчивом человеке; 5) о том, кто никак не может прекратить смеяться; 6) о беспорядке, неразберихе, царящих где-либо; 7) о чем-нибудь необычайном, невероятном; 8) о человеке, имеющем большой опыт в чем-либо; 9) о предмете всеобщих разговоров, пересудов, сплетен.

13. Докажите с помощью примеров из приведенного ниже текста (автор — Л. В. Ухова) его принадлежность к научному стилю; определите, в рамках какого подстиля он составлен, и аргументируйте свою точку зрения. Выделите языковые средства, с помощью которых достигается логичность изложения, укажите на лексические единицы, грамматические формы и синтаксические конструкции, свойственные научной речи.

Особенностью публичной речи как разновидности речевого общения является также и то, что это вид прямой коммуникации, контролируемый обоими участниками общения. Это позволяет обеим сторонам корректировать характер взаимодействия, влиять на его результаты [Шмелева 1985], что обусловливает такую специфическую черту публичной речи, как спонтанность, которая, однако, носит ограниченный характер и прежде всего потому, что текст публичного выступления, имея устную форму презентации, как правило, готовится заранее и чаще фиксируется в письменном виде.

Диалог, будучи одной из форм существования языка, является едва ли не важнейшей областью проявления языковых закономерностей. Отличие диалога от других сфер функционирования языка заключается прежде всего в сложной картине взаимодействия интенций коммуникантов. Нормальный ход диалога предполагает согласование иллокутивных намерений (коммуникативных целей) участников диалога, которое заключается в удовлетворении их взаимных претензий.

Участники диалога вынуждены выполнять разнообразные речевые и неречевые действия, заставляя партнера реагировать на них определенным образом. Каждое такое речевое и неречевое вынуждение получает ответную реакцию со стороны другого участника (ср. у Бахтина представление о смене речевых субъектов и о выполнении взаимных обязательств участников как об основной особенности диалога в сравнении с монологом [Бахтин 1979: 251, 255]).

Говоря о взаимодействии участников общения в рамках диалога, А. Н. Баранов, Г. Е. Крейдлин вводят понятие иллокутивного вынуждения как «одного из проявлений законов сцепления, действующих на пространстве диалога» [Баранов, Крейдлин 1992, № 2]. Речевые акты, связанные в речевом контексте отношением иллокутивного вынуждения, авторы предлагают называть соответственно иллокутивно независимым и иллокутивно зависимым. Иллокутивно независимый речевой акт они определяют как речевой акт, иллокутивное назначение которого на данном шаге определяется интенциями самого говорящего, а иллокутивно зависимый речевой акт как речевой акт, иллокутивное назначение котого всецело определяется иллокутивным назначением какой-либо предшествующей реплики (из данного речевого отрезка), соответственно распределяя иллокутивно независимые и иллокутивно зависимые реплики. Структура диалога, по мнению авторов, опирается на отношение иллокутивного вынуждения, подобно тому как структура предложения формируется на основе синтаксических связей. «Между тем иллокутивное вынуждение не тождественно синтаксической связи. Если такая связь, как синтаксическая зависимость, основывается исключительно на категориальных свойствах языковых единиц, то вынуждение, действуя на пространстве речевых актов, формируется не только под влиянием иллокутивной функции речевых высказываний, но и находится под воздействием общих законов функционирования диалога» [Баранов, Крейдлин 1992: 88]. (К последним, в частности, принадлежат социально обусловленные законы-максимы Грайса и принцип вежливости Линча.)

Исследование закономерностей речевого взаимодействия, выявление путей оптимизации такого взаимодействия – задачи, которые на сегодняшний день лежат в области интересов различных направлений антрополингвистики [Седов 2000], и в частности, — лингвистической конфликтологии. О природе конфликтов много и достаточно интересно писали психологи [Берн, 1997; Шейнов, 1999]. В работах зарубежных и отечественных исследователей показаны причины конфликтов, даны рекомендации по их предотвращению и выходу из конфликтной ситуации. Языковеды делают лишь первые шаги в освоении этого объекта изучения [Горелов, Седов 1998; Енина 1999; Жельвис 1997; Седов 1996; Стернин 1995; Ширяев 2000].

Определение механизмов диалогического взаимодействия – задача далеко не решенная в современном языкознании, хотя область эта интенсивно изучается. О. С. Иссерс отмечает [Иссерс, 1999], что некоторые объяснения этих процессов могут быть получены через описание стратегических направлений и тактических приемов, реализуемых по ходу диалога. По мнению автора, стратегия речевого поведения охватывает всю сферу построения процесса коммуникации, когда ставится целью достижение определенных долговременных результатов. В общем смысле «речевая стратегия включает в себя планирование процесса речевой коммуникации в зависимости от конкретных условий общения и личностей коммуникантов, а также реализацию этого плана [Иссерс 1999: 54].

14. Исправьте максимально возможное количество ошибок в приведенном ниже стихотворении. Поставьте ударение в словах, выделенных полужирным курсивом.

Хоть я и не русский очень, но хочу любви.

Положил документ в портфель – стырили.

Сажаюсь на тубаретку, отложив дела.

Почто ты мне звонишь редко, где была ты?

Не понял, объясни, я ведь подросток в любви,

Как мне начать с тобой. Ты углубила мне боль:

Ножницами наскрозь режешь сердце мне до слёз,

Вот уже много дня не схожу я с билютня.

Я мышленье напрягаю, звоню в телефон,

Ложу тебе письма в ящик: полон он.

Тебе подарю я арбуз, грейфруктов куплю,

Кружовника насажаю – я тебя люблю.

Прецендент я создал, за транваем побежал,

Посклизнулся в снегу, поломал себе ногу.

И теперь я в пальте на инвалидном месте,

С костылем и в гипсу еду в лесополосу.

Там живешь ты в дому, адрес я храню в уму:

Квартал нужно пройтить, библиотеку найтить.

Мне достал родный брат зарубёжный мирмалат –

Прямо тает в роту, открывай, я накладу.

В колидор выходи, раболант моей любви,

Не застойся в дверях, жгёт огонь меня в нутрях.

Ты придёшь, я скажу, что словов не нахожу:

Так могуч, так велик этот русский твой язык.

ПРИ ВЫПОЛНЕНИИ КОНТРОЛЬНОЙ РАБОТЫ, А ТАКЖЕ ПРИ ПОДГОТОВКЕ К ЭКЗАМЕНУ (С ЦЕЛЬЮ ЕГО НЕЗАТРУДНИТЕЛЬНОЙ СДАЧИ) ПОЛЬЗУЙТЕСЬ СЛЕДУЮЩЕЙ ЛИТЕРАТУРОЙ:

1. Аванесов литературное произношение. М., 1984.

2. , Зарва ударений русского языка. М., 1993.

3. Введенская и искусство речи. Ростов-на-Дону, 1996.

4. , , Кашаева язык и культура речи. Ростов-на-Дону, 2003.

5. Головин культуры речи. М., 1980.

6. Голуб русского языка. М, 2001.

7. , , Ягубова язык и культура речи: Учебник для студентов-нефилологов / Под редакцией д. ф. н., проф. О. Б. Сиротининой. Саратов, 2001.

8. Горбачевич современного русского литературного языка. М., 1978.

9. К, , Катлинская правильность русской речи. Стилистический словарь вариантов. М., 1976.

10.Культура русской речи / Под ред. проф. и проф. . М., 1998.

11., Шведова словарь русского языка. М., 1995.

12.Орфоэпический словарь русского языка. М., 2001.

13.Плещенко и культура речи: Учеб. Пособие / Т. П. Плещенко, , ; Под ред. . ¾ Мн.: «ТетраСистемс», 2001.

14.Розенталь стилистика русского языка. М., 1987.

15.Русский язык и культура речи: Учебно-методический комплекс для вузов / Под ред. доц. . Кемерово, 2002.

16.Русский язык и культура речи / Под ред. проф. . М., 2002.

17.Русский язык и культура речи / Под ред. проф. . М., 2000.

18.Русский язык и культура речи: Конспект лекций / , , Е. В Ваганова и др. – Челябинск: Изд-во ЮУрГУ, 2003.

источники:

http://hr-portal.ru/article/kak-maksimalno-effektivno-i-pravilno-provesti-sobesedovanie

http://pandia.org/text/78/040/630.php

Понравилась статья? Поделить с друзьями:
  • Путь проб и ошибок на английском
  • Путь педагога это не ошибка
  • Путь ошибок трудных стихи
  • Путь ошибок разочарований и надежд
  • Путь к истине идет через ошибки сочинение